File:  [Local Repository] / db / baza / okmar16.txt
Revision 1.15: download - view: text, annotated - select for diffs - revision graph
Mon Jan 28 11:49:47 2019 UTC (5 years, 4 months ago) by rubashkin
Branches: MAIN
CVS tags: HEAD
*** empty log message ***

Чемпионат:
Окский марафон - 2016 (Пущино)

URL:
/znatoki/boris/reports/201601Puschino.html

Дата:
30-Jan-2016

Инфо:
В качестве первого тура марафона использовался синхрон "Карельские
петроглифы".

Тур:
2 тур. "20 000 сомелье под водой" (Тверь)

Редактор:
Станислав Попов (унтер-редактор), Максим Мерзляков (убер-редактор)

Инфо:
Команда, авторы и редакторы благодарят за тестирование и ценные
замечания по пакету Дмитрия Великова, Алису Хабирову, Марию Колосовскую,
Александра Коробейникова, Александра Либера, Дмитрия Андреева, а также
Nora Karu и Валерия Бьёрндалена.

Вопрос 1:
В Испании словом "cremallera" называют не только трехрельсовую горную
железную дорогу, но и некое приспособление. В некоторых языках слово,
обозначающее это приспособление, начинается с буквы Z. Назовите это
приспособление.

Ответ:
Застежка-молния.

Источник:
   1. http://es.wikipedia.org/wiki/Cremallera_(mecanismo)
   2. http://es.wikipedia.org/wiki/Cremallera_(indumentaria)

Автор:
Станислав Попов

Вопрос 2:
Греческие ученые изучили цветовые соотношения на пятистах пейзажах,
изображающих закат солнца, с начала XV по конец XIX века. Анализ
показал, что глобальные изменения находят четкое отражение в палитре, а
именно, максимальное преобладание красных тонов приходится на ПРОПУСК.
Заполните пропуск четырьмя символами или тремя словами.

Ответ:
1816.

Зачет:
Год без лета.

Источник:
http://www.atmos-chem-phys.net/7/4027/2007/acp-7-4027-2007.pdf

Автор:
Софья Вискова

Вопрос 3:
Владимир Маяковский знал, чем можно заняться на курорте. Прослушайте
отрывок из его стихотворения "Севастополь - Ялта":
   В авто насажали разных армян,
   рванулись - и мы в пути.
   Дорога до Ялты будто [ПРОПУСК 1]:
   всё время надо [ПРОПУСК 2].
   Восстановите оба пропущенных слова.

Ответ:
"... роман...", "... крутить".

Источник:
http://ru.wikisource.org/wiki/Севастополь_%E2%80%94_Ялта_(Маяковский)

Автор:
Роман Бережнов

Вопрос 4:
Спикер парламента Вануату, которого Верховный суд признал виновным во
взяточничестве, воспользовался временным отсутствием президента страны и
смог избежать наказания. Ответьте тремя словами: как именно?

Ответ:
Помиловал сам себя.

Зачет:
По смыслу.

Комментарий:
Согласно конституции Вануату, на время отсутствия президента страны
спикер парламента выполняет его функции, в том числе и помилование.

Источник:
https://meduza.io/news/2015/10/14/spiker-parlamenta-vanuatu-pomiloval-sam-sebya

Автор:
Александр Круглов

Вопрос 5:
Нос корабля "Васа" был украшен многочисленными статуями исторических
личностей. Мы не спрашиваем, ни как по-голландски звучит "нос корабля",
ни какое сооружение находилось на внутренней стороне борта в этой части.
Назовите имя того, кого изображала третья статуя справа.

Ответ:
Веспасиан.

Комментарий:
"Нос корабля" по-голландски - "гальюн", и именно там располагалось
отхожее место для команды. Забавно, что практически напротив туалета
была размещена статуя Веспасиана.

Источник:
ЛОАВ в музее корабля "Васа".

Автор:
Станислав Попов, Софья Вискова

Вопрос 6:
Гибель английского линкора "Бархэм" была засекречена, чтобы не допустить
упадка боевого духа. Тем не менее, известная в определенных кругах Хелен
Дункан разгласила полученную информацию, и в 1944 году ее осудили
согласно Акту о предательстве, изданному еще в XVIII веке. Какое слово
мы заменили на "предательство"?

Ответ:
Колдовство.

Комментарий:
Если быть точным, Хелен Дункан обвинили в "ложном утверждении об
обладании сверхъестественными способностями", но впоследствии появились
слухи, что обвинения были сфабрикованы, потому что руководство флота
боялось, что Дункан разгласит точную дату и место высадки союзников в
Европе.

Источник:
http://en.wikipedia.org/wiki/Helen_Duncan

Автор:
Станислав Попов

Вопрос 7:
(pic: 20160287.jpg)
   Внимание, в тексте вопроса слово "АЛЬФА" заменяет два слова.
   Перед вами фотография немецкого крейсера "Эмден". Однажды, чтобы в
сумерках пробраться в контролируемую британцами гавань, команда крейсера
решила сделать АЛЬФУ. Из книги известно, что другая АЛЬФА способна
уменьшить яркость светил на треть. Какие два слова мы заменили АЛЬФОЙ?

Ответ:
Четвертая труба.

Комментарий:
Британские военные суда были, в основном, четырехтрубными, и команда
"Эмдена" из дерева, брезента и какой-то материи собрала фальшивую
четвертую трубу. Рейд, кстати, удался. А звук четвертой трубы, согласно
Апокалипсису, погасил треть солнца, треть луны и треть звезд.

Источник:
   1. http://ru.wikipedia.org/wiki/Бой_у_Пенанга
   2. Откровение, 8:12.
http://ru.wikisource.org/wiki/Откровение_святого_Иоанна_Богослова#.D0.93.D0.BB.D0.B0.D0.B2.D0.B0_8

Автор:
Станислав Попов

Вопрос 8:
(pic: 20160288.jpg)
   Перед вами фотография линкора "Адмирал граф Шпее", который внешне
очень походил на линкор "Дойчланд". Немецкие моряки если и пытались
обмануть врага, то делали это честно. Ответьте одним словом: что мы
неоднократно использовали в этом блоке вопросов, а на раздатке убрали?

Ответ:
Кавычки.

Комментарий:
(pic: 20160289.jpg)
   Чтобы смутить иностранную разведку, вместо настоящего названия
корабля было написано "Дойчланд". В кавычках, по-честному.

Источник:
http://www.deutschland-class.dk/admiral_graf_spee/gallery/gallgrafspeeindisquise.html

Автор:
Станислав Попов

Вопрос 9:
В эпизоде сериала "Место преступления" патологоанатом пытается
определить время смерти по температуре печени убитого с помощью
специального зонда. Это получается не с первой попытки, так как
покойный, отличавшийся утонченным вкусом, многие годы ДЕЛАЛ ЭТО. ЭТО
ДЕЛАЛ и один детектив, что однажды спасло ему жизнь. Что мы заменили
словами "ДЕЛАТЬ ЭТО"?

Ответ:
Носить корсет.

Комментарий:
У жертвы в результате долгого ношения корсета сместились все внутренние
органы, и патологоанатом промахнулся датчиком. А Эрасту Петровичу
Фандорину корсет спас жизнь - о него сломался нож убийцы.

Источник:
   1. Сериал "CSI: Las Vegas", эпизод "Way to go".
   2. Б. Акунин. Азазель.

Автор:
Станислав Попов

Вопрос 10:
Изображение одного из персонажей спектакля на афише тверского театра
содержит аллюзию на известное полотно Рене Магритта. В принципе, аллюзия
сохранилась бы, если бы использовали практически любого из персонажей.
По произведению какого автора поставлен этот спектакль?

Ответ:
[Джанни] Родари.

Комментарий:
Спектакль поставлен по сказке "Приключения Чиполлино". Афиша частично
повторяет картину Магритта "Сын человеческий" - ту, где человек с
яблоком вместо лица. На афише изображен, очевидно, принц Лимон, но можно
было бы взять любого из фрукто-овощных персонажей.

Источник:
Афиша тверского Театра юного зрителя.

Автор:
Софья Вискова

Вопрос 11:
Единственной обязанностью первых жителей Ораниенбаума было
организовывать рыбалку на казенных прудах. Поэтому летом изо дня в день
они чинили снасти и приводили в порядок территорию, зато зимой делать
было совсем нечего. Ответьте одним словом: КЕМ прозвали жителей
Ораниенбаума? Ответьте одним словом: КЕМ прозвали жителей Ораниенбаума?

Ответ:
Сурки.

Источник:
Н.А. Синдаловский. Городские имена вчера и сегодня. Судьбы петербургской
топонимики в городском фольклоре. http://www.flibusta.is/b/362240/read

Автор:
Оксана Верпатова

Вопрос 12:
Многие рок-группы используют ЕГО, хотя на звук это обычно не влияет. У
одной немецкой группы ОН состоит из трех элементов. Назовите ЕГО словом
немецкого происхождения.

Ответ:
Умляут.

Комментарий:
Многие группы добавляют в логотип умляут, чтобы смотрелось
ГÖТИЧНÖ, но чаще всего произношение слова не меняется. У
группы "Die Ärzte" умляут в названии и так был, поэтому они его
удлинили до трех точек.

Источник:
http://tvtropes.org/pmwiki/pmwiki.php/Main/HeavyMetalUmlaut

Автор:
Станислав Попов

Вопрос 13:
В одном фильме рок-музыкант объясняет, почему группа использует умляут в
названии: "Он как пара глаз. Ты смотришь на умляут, а он смотрит на
тебя". Интересно, что известная группа при переиздании одного из своих
хитов 1970 года использовала умляут над буквой i. От какого греческого
слова происходит название этого хита?

Ответ:
Паранойя.

Комментарий:
В названии песни "Paranoïd" появились глаза, которые следят...

Источник:
http://en.wikipedia.org/wiki/Metal_umlaut

Автор:
Станислав Попов

Вопрос 14:
На англоязычном юмористическом коллаже изображено, как Роджер и Роберт,
так сказать, коллеги по цеху, работают в саду. Назовите фамилии Роджера
и Роберта.

Ответ:
Уотерс и Плант.

Комментарий:
А подпись гласит: "Watch Robert Plant while Roger Waters".

Источник:
http://9gag.com/gag/axGP74Y

Автор:
Павел Кадочников

Вопрос 15:
Слово "ОНА" является заменой.
   Википедия утверждает, что русское слово "ОНА", вероятно, восходит к
двум латинским словам, одно из которых - "inter" - означает "между".
Если это действительно так, то ЕЕ пренебрежительное название весьма
точно отражает этимологию. Какое слово мы заменили на "ОНА"?

Ответ:
Интеллигенция.

Комментарий:
Оскорбительное "прослойка" оказывается весьма точным отражением этого
варианта этимологии.

Источник:
http://en.wikipedia.org/wiki/List_of_English_words_of_Russian_origin

Автор:
Станислав Попов

Вопрос 16:
Один английский глагол имеет значение "использовать годные детали из
неисправного устройства". Не исключено, что к такой процедуре пришлось
прибегнуть членам полярной экспедиции Франклина, застрявшей во льдах. От
какого слова с удвоенной согласной происходит упомянутый глагол?

Ответ:
Каннибал.

Комментарий:
Глагол "to cannibalize" означает не только съесть себе подобного, но и
разобрать машину на запчасти. В принципе, похожие процессы. Как
известно, в ходе неудачной экспедиции Франклина были отмечены случаи
каннибализма.

Источник:
   1. http://www.merriam-webster.com/dictionary/cannibalize
   2. http://en.wikipedia.org/wiki/Franklin%27s_lost_expedition

Автор:
Станислав Попов, Софья Вискова

Вопрос 17:
Как известно, каннибализмом называют поедание особей своего вида. У
живородящих акул более сильные зародыши еще в утробе матери убивают
более слабых. Это явление Игорь Акимушкин назвал эмбриональным
калибанизмом. В одном из слов предыдущего предложения мы добавили
несколько лишних букв. Восстановите это слово в исходном виде.

Ответ:
Каинизм.

Комментарий:
Каинизм в биологии - убийство собственных братьев и сестер.

Источник:
И.И. Акимушкин. Мир животных. Том 4. Рассказы о змеях, крокодилах,
черепахах, лягушках, рыбах. http://www.flibusta.is/b/325787/read

Автор:
Станислав Попов

Вопрос 18:
Выход первого фильма фантастической саги "Звездные войны" произвел
настоящий фурор. Джордж Лукас рассказывал, что для озвучки Дарта Вейдера
хотел пригласить МИХАИЛА ЕФРЕМОВА, но решил, что его голос будет слишком
узнаваемым. Какие имя и фамилию мы заменили на "МИХАИЛ ЕФРЕМОВ"?

Ответ:
Орсон Уэллс.

Комментарий:
Орсон Уэллс в свое время тоже произвел фурор, использовав фантастическое
произведение, так что его голос явно был знаком американцам.

Источник:
http://www.imdb.com/title/tt0076759/trivia?item=tr0742832

Автор:
Станислав Попов

Вопрос 19:
Дуплет.
   1. Одна туристическая компания в Ненецком автономном округе
предлагает отвязное приключение с национальным колоритом под названием
"ТАКИЕ выходные в Арктике". Какое слово мы заменили на "ТАКИЕ"?
   2. Журнал "Автопилот" в ноябре 2009 года в рубрике "Стиль"
опубликовал подборку модной одежды и аксессуаров из Италии и написал,
что "Российские ОНИ автомобилей Ferrari и Maserati не останутся
равнодушными". Какое слово мы заменили на "ОНИ"?

Ответ:
   1. Чумовые.
   2. Тиффози.

Источник:
   1. http://tourism.interfax.ru/ru/news/articles/29481/
   2. http://www.autopilot.ru/issues/auto/2009/11/22.html

Автор:
Максим Алексеев, Станислав Попов

Вопрос 20:
В спецпроекте журнала "Арзамас" английский король Генрих V, отличавшийся
стилем и импозантностью, получил титул "ПРОПУСК года - 1415". Заполните
пропуск коротким словом.

Ответ:
Лук.

Комментарий:
Не только выглядел красиво, но и битву при Азенкуре выиграл.

Источник:
http://arzamas.academy/mag/233-new-year-poll

Автор:
Софья Вискова

Вопрос 21:
(pic: 20160290.jpg)
   Перед вами фотография Джессики Смит. Примерно за двадцать лет до
этого снимка Джессика сыграла свою единственную звездную роль в
популярном сериале. Назовите имя любого из главных персонажей этого
сериала.

Ответ:
Любой из персонажей: Тинки-Винки, По, Ляля, Дипси.

Комментарий:
А Джессика сыграла солнышко.

Источник:
http://www.telegraph.co.uk/news/newstopics/howaboutthat/11310883/The-baby-from-Teletubbies-reveals-herself.html

Автор:
Александр Круглов

Вопрос 22:
(pic: 20160291.jpg)
   На розданной вам картинке мы закрыли ИКС. Интересно, что в 2013 году
канал "Discovery" назвал ИКС "королем селфи". Напишите ИКС по-английски.

Ответ:
Curiosity.

Комментарий:
Как известно, любопытство убило кошку. А марсоход "Curiosity" сделал
несколько десятков селфи на фоне марсианских пейзажей.

Источник:
   1. http://www.condenaststore.com/-sp/It-s-curiosity-Cartoon-Prints_i8642768_.htm
   2. http://en.wikipedia.org/wiki/Space_selfie

Автор:
Станислав Попов

Вопрос 23:
   <раздатка>
   ТЯРУРИДЗУ
   </раздатка>
   В эпизоде романа Бориса Акунина "Статский советник" камердинер
Фандорина, японец Маса в Петросовских банях отмечает, что тела
европейцев некрасивы и вспоминает мудреца по имени Тяруридзу. А как
фамилия этого мудреца?

Ответ:
Дарвин.

Источник:
Б. Акунин. Статский советник.

Автор:
Станислав Попов

Вопрос 24:
(pic: 20160292.jpg)
   Перед вами узор "персидский огурец", который один блогер назвал
"хипстерская ОНА". Одним из первых ЕЕ описал Христиан Гюйгенс,
прославившийся астрономическими открытиями. Назовите ЕЕ составным
словом.

Ответ:
Инфузория-туфелька.

Источник:
   1. http://dil.diary.ru/p193546920.htm
   2. http://en.wikipedia.org/wiki/Paramecium

Автор:
Софья Вискова, Станислав Попов

Тур:
3 тур. "Hangover" (Архангельск - Москва - Санкт-Петербург)

Редактор:
Валерия Комаровская (Коломна)

Вопрос 1:
Дуплет.
   1. Александр Никонов пишет, что в общежитии, где жил его друг, в ЭТОМ
ГОДУ так и не смогли запустить лифт. Назовите ЭТОТ ГОД.
   2. Память святого Юлия празднуется в ЭТОТ ДЕНЬ, хотя обычно с этой
датой связано другое имя. Напишите упомянутые месяц и день.

Ответ:
   1. 1961.
   2. 12 апреля.

Комментарий:
В год запуска человека в космос в общежитии несколько месяцев не могли
запустить лифт. Первым человеком в космосе был Юрий Гагарин.

Источник:
   1. А.П. Никонов. За фасадом империи. Краткий курс отечественной
мифологии. http://www.flibusta.is/b/297525/read
   2. http://ru.wikipedia.org/wiki/12_апреля

Автор:
Денис Никитенко

Вопрос 2:
По словам Игоря Киселёва, всё законодательство самопровозглашенной
Ичкерии производило "эффект АЛЬФЫ": для внешнего применения -
конституция чуть ли не европейского образца, однако на самом деле
террористический анклав. В фильме 1971 года АЛЬФЕ дали своеобразное
социолектическое определение. Напишите это определение двумя словами.

Ответ:
Нехороший человек.

Комментарий:
Политический эффект редиски, согласно "Толковому словарю
обществоведческих терминов", - "попытки некоторых политических деятелей
скрыть свою реакционную сущность за прогрессивной фразеологией".
Согласно фильму "Джентльмены удачи", "редиска - нехороший человек".

Источник:
   1. http://ru.wikipedia.org/wiki/Чеченская_Республика_Ичкерия#.D0.A6.D0.B8.D1.82.D0.B0.D1.82.D1.8B
   2. http://www.slovarnik.ru/html_tsot/p/politi4eskiy-6ffekt-rediski.html
   3. Х/ф "Джентльмены удачи" (1971), реж. Александр Серый.

Автор:
Денис Никитенко

Вопрос 3:
За всю свою жизнь Пушкин написал ТУДА писем больше, чем отцу, матери,
сестре и брату вместе взятым. Герой Пелевина, шутя, говорит пациенту,
что хоть тот и не Пушкин, его вернут ТУДА. Ответьте тремя словами: куда?

Ответ:
В Третье отделение.

Зачет:
По словам "Третье отделение".

Комментарий:
В самом деле, Пушкин написал Бенкендорфу как начальнику Третьего
отделения Собственной Его Императорского Величества канцелярии 58 писем.
Врач Тимур Тимурович из романа "Чапаев и Пустота" любил пошутить.

Источник:
   1. Ю.И. Дружников. Узник России. http://www.flibusta.is/b/161941/read
   2. В.О. Пелевин. Чапаев и Пустота.
http://pelevin.nov.ru/romans/pe-pust/2.html

Автор:
Денис Никитенко

Вопрос 4:
Парадоксально, но основную свою функцию эта организация стала исполнять
не после события, случившегося 14 апреля, а после аналогичного события в
1901 году. Создание же ее было одобрено президентом именно утром 14
апреля. Назовите упомянутого президента.

Ответ:
[Авраам] Линкольн.

Комментарий:
Утром 14 апреля 1865 года Авраам Линкольн одобрил создание Секретной
службы США, а вечером того же дня был смертельно ранен в театре Форда.
Охрана первых лиц была вменена в обязанности Секретной службе США лишь
после убийства президента Мак-Кинли (до этого она занималась, в
основном, борьбой с фальшивомонетчиками и подчинялась Министерству
финансов). При этом лишь в 2003 году она вошла в состав Министерства
внутренней безопасности.

Источник:
http://ria.ru/spravka/20150705/1111618727.html

Автор:
Денис Никитенко

Вопрос 5:
Герой романа Гарри Гаррисона "Стальная Крыса отправляется в ад"
описывает надпись на вратах Вальхаллы и упоминает ИХ. ОНИ считаются
плохой приметой, и собирать ИХ в таком случае лучше, берясь за "ножку",
а не за "головку". Назовите ИХ двумя словами.

Ответ:
Рассыпанные спички.

Зачет:
Просыпанные спички и т.п.

Комментарий:
Руны напоминают герою Гаррисона рассыпанные спички. Считается, что
просыпанные спички - такая же плохая примета, как и просыпанная соль.

Источник:
Г. Гаррисон. Стальная Крыса отправляется в ад.
http://www.loveread.ec/read_book.php?id=1295&p=37

Автор:
Денис Никитенко

Вопрос 6:
В фильме "Статский советник" персонаж Никиты Михалкова, вербуя
информатора, упоминает героя пушкинской сказки. А какой эпитет он
применяет по отношению к третьей ипостаси этого героя?

Ответ:
Мохнатый.

Комментарий:
Информатора сравнивают с царевичем Гвидоном, который будет то комаром,
то мухой, то шмелем "летать" в Охранное отделение. Эпитет "мохнатый" -
отсылка к знаменитому романсу "А цыган идет..." из фильма Рязанова
"Жестокий романс", который исполняет герой Михалкова Паратов.

Источник:
Х/ф "Статский советник" (2005), реж. Филипп Янковский.

Автор:
Екатерина Макоед

Вопрос 7:
На современном юмористическом рисунке один ИКС говорит другому: "Видишь,
какие хвосты были у наших предков?" - и указывает на картину, где
изображена ИКСОВАЯ АЛЬФА. Какие два слова, начинающиеся на одну букву,
мы заменили на "ИКСОВАЯ АЛЬФА"?

Ответ:
Телефонная трубка.

Источник:
https://www.adme.ru/zhizn-nostalgiya/11-kartinok-o-tom-kak-silno-izmenilsya-mir-1077860/

Автор:
Сергей Коновалов

Вопрос 8:
В статье на российском таможенном портале утверждается, что этот
музыкальный инструмент - едва ли не главный солист в отечественной
торговле. Назовите этот инструмент.

Ответ:
Контрабас.

Комментарий:
К сожалению, значительную часть торгового оборота составляют
контрабандные товары, на сленге таможенников - "контрабас".

Источник:
http://www.tks.ru/reviews/2015/10/26/03

Автор:
Сергей Коновалов

Вопрос 9:
Дуплет.
   1. В одном из эпизодов книги Бернарда Корнуэлла викинги оставляют
экипаж захваченного судна без денег. Какие два слова мы заменили в
вопросе?
   2. Маргарет Грайвер едва не погибла во время крушения "Лузитании". Ее
спас вырывающийся воздух и то, что она осталась без денег. Какие три
слова мы заменили в вопросе?

Ответ:
   1. На мели.
   2. Вылетела в трубу.

Комментарий:
Маргарет засосало в трубу тонущей "Лузитании", но вырвавшийся изнутри
воздух буквально "выстрелил" ее на поверхность, и она спаслась.

Источник:
   1. Б. Корнуэлл. Властелин Севера.
http://www.flibusta.is/b/255403/read
   2. http://ru.wikipedia.org/wiki/Потопление_%C2%ABЛузитании%C2%BB

Автор:
Сергей Коновалов

Вопрос 10:
Прослушайте цитату: "Год назад мы с другом путешествовали по Дагестану.
Да, это то самое далекое место, где постоянно ловят бородатых дядек. Там
ПРОПУСК - это не доктор, а режим". Заполните пропуск.

Ответ:
КТО.

Комментарий:
"Доктор Кто" - известный британский сериал. КТО расшифровывается как
"контртеррористическая операция".

Источник:
http://lenta.ru/articles/2015/10/09/dagestan/

Автор:
Екатерина Макоед

Вопрос 11:
Герой Хэмфри Богарта, прошедший Вторую мировую войну, знает, что в виски
подмешано снотворное, но выпивает его. Перед этим он произносит тост из
одного слова. Воспроизведите этот тост.

Ответ:
Джеронимо.

Комментарий:
Герой Богарта - бывший десантник. Чувствуя опасность, он произносит клич
десантников-парашютистов. Небольшой подсказкой может служить то, что
клич "Джеронимо!" часто использовал одиннадцатый Доктор Кто, герой
предыдущего вопроса.

Источник:
Х/ф "Рассчитаемся после смерти" (1947), реж. Джон Кромвелл.

Автор:
Денис Никитенко

Вопрос 12:
В цикле произведений Джорджа Мартина действуют островитяне, очень
похожие на исландских викингов. Один из их кораблей назывался
"Серебряный ИКС". Поскольку в книге не было проставлено ударение, автор
вопроса не понял, является ли ИКС частью тела или строительным
материалом. Какое слово мы заменили на ИКС?

Ответ:
Плавник.

Источник:
Дж. Мартин. Пир стервятников. http://www.flibusta.is/b/308110/read

Автор:
Сергей Коновалов

Вопрос 13:
Герой американского сериала учится играть на музыкальном инструменте.
Жена называет его прозвищем, которое одной буквой отличается от названия
известной группы. Назовите эту группу.

Ответ:
"Bon Jovi".

Комментарий:
Герой учится играть на банджо, жена называет его банджови (banjovi).

Источник:
Сериал "Modern Family", 6 сезон.

Автор:
Екатерина Макоед

Вопрос 14:
Ричард Бернстайн пишет, что китайские женщины видят в иностранцах
возможность изменить свою жизнь, поэтому называют их "ИКС". Когда автор
вопроса набрал "ИКС" в поисковике Yandex, он не нашел ссылку на
популярную песню из-за обилия онлайн-сервисов. Какие три слова мы
заменили на ИКС?

Ответ:
Билет на самолет.

Комментарий:
"Билет на самолет с серебристым крылом" - строчка из песни Виктора Цоя
"Пачка сигарет". Из-за обилия онлайн-сервисов, предоставляющих
возможность купить билеты, песня в поисковике на первых страницах не
появляется.

Источник:
   1. Р. Бернстайн. Восток, Запад и секс. История опасных связей.
http://www.loveread.ec/read_book.php?id=45789&p=2
   2. https://yandex.ru/search/?text=билет%20на%20самолет&lr=2

Автор:
Сергей Коновалов

Вопрос 15:
Звено Кларк Гиллис - Брайан Троттье - Майк Босси было одним из
величайших в истории НХЛ. От болельщиков оно получило прозвище РЕКА
ВЕЛИКАЯ. Какие слова мы заменили на "РЕКА ВЕЛИКАЯ"?

Ответ:
Трио-Гранде.

Зачет:
Trio Grande.

Источник:
   1. http://ru.wikipedia.org/wiki/Босси,_Майк
   2. http://en.wikipedia.org/wiki/The_Trio_Grande

Автор:
Сергей Коновалов

Вопрос 16:
Внимание, добрый дуплет!
   1. В боевике "Ронин" 1998 года ЭТОГО не происходит. Назовите ЭТО.
   2. В серии "Лагерь для толстяков", вышедшей 6 декабря 2000 года,
ЭТОГО не происходит. Назовите ЭТО.

Ответ:
   1. Смерть [персонажа] Шона Бина.
   2. Смерть Кенни [Маккормика].

Зачет:
   1. Убийство [персонажа] Шона Бина.
   2. Убийство Кенни [Маккормика].

Комментарий:
   1. Шон Бин считается одним из самых "убиваемых" актеров.
   2. Кенни Маккормика убивают практически в каждой серии "Южного
Парка".

Источник:
   1. http://ru.wikipedia.org/wiki/Ронин_(фильм)
   2. http://ru.wikipedia.org/wiki/Лагерь_для_толстяков

Автор:
Сергей Коновалов

Вопрос 17:
Комментируя назначение официальным представителем МИД РФ Марии
Захаровой, Владимир Соловьёв назвал Марию "пианистка". Какое слово мы
заменили в вопросе?

Ответ:
Антипсаки.

Комментарий:
Анаграмма.

Источник:
http://lenta.ru/news/2015/08/10/mid/

Автор:
Сергей Коновалов

Вопрос 18:
Недавно автору вопроса попалась разновидность ЕЕ, которая начиналась с
Ленина и Сталина, а заканчивалась якобы вторым сроком Путина. Автор
хотел проверить, но как-то не сложилось. Назовите ЕЕ, используя четыре
символа.

Ответ:
2048.

Комментарий:
В этой вариации игры "2048" два Ленина превращаются в Сталина, два
Сталина - в Хрущёва и т.п.

Источник:
http://putin2048.ru/

Автор:
Сергей Коновалов

Вопрос 19:
Мы знаем, как вы любите, когда в вопросах что-нибудь убрано, заменено
или переставлено. Внимание, блиц!
   1. Статья на Горнопромышленном портале России об опасностях отрасли
озаглавлена "АЛЬФА". Какие два слова мы заменили на "АЛЬФА"?
   2. Статья на портале pravda.ru о заполонивших Нью-Йорк насекомых
называется "БЕТА на штаб-квартиру ООН". Какие два слова мы заменили на
"БЕТА"?
   3. Прослушайте четверостишие из фанфика под названием "ГАММА":
   По всей Галактике великий плач звучит.
   "На Корускант напали!" - Голонет кричит.
   Генерал Гривус канцлера схватил
   И с собою на крейсер утащил.
   Какие два слова мы заменили на "ГАММА"?

Ответ:
   1. Срытая угроза.
   2. Атака клопов.
   3. Месть стихов.

Комментарий:
Каждая замена - немного измененное название первых по фабуле трех частей
киносаги "Звездные войны", в котором буквы убраны, заменены и
переставлены соответственно. Автор приносит свои извинения за то, что
вам пришлось внимательно прослушать это ужасное четверостишие. :-)

Источник:
   1. http://www.miningexpo.ru/news/12483
   2. http://www.pravda.ru/science/planet/environment/29-10-2010/1055236-cimexlectularius-0/
   3. https://ficbook.net/readfic/2681116/7145189#part_content

Автор:
Денис Никитенко

Вопрос 20:
Отрывок из статьи про одного предпринимателя: "Массимо вскоре расширил
географию поставок зерна и кукурузы, в том числе заключив очень выгодные
во всех смыслах контракты со странами Северной Африки". Какое слово мы
заменили в предыдущем предложении?

Ответ:
Хлебные.

Источник:
http://www.sports.ru/tribuna/blogs/lateral/850561.html

Автор:
Сергей Коновалов

Вопрос 21:
(pic: 20160293.jpg)
   На розданной вам картинке героиня в момент наивысшего эмоционального
напряжения предупреждает Мюнхгаузена, завершая фразу "Они положили сырой
порох" одним словом после запятой. Назовите это слово.

Ответ:
Карл.

Комментарий:
На розданной картинке Марта, вторая жена барона Карла Фридриха Иеронима
фон Мюнхгаузена. В наивысший по накалу страстей момент в фильме она
кричит: "Они положили сырой порох, Карл! Они хотят помешать тебе!".
Многие интернет-ресурсы называют мем про Карла самым эмоциональным.

Источник:
   1. Х/ф "Тот самый Мюнхгаузен" (1979), реж. Марк Захаров.
   2. https://coub.com/view/6sery

Автор:
Денис Никитенко

Вопрос 22:
(pic: 20160294.jpg)
   В конце одного из клипов группы "Мельница" есть кадр, где музыканты
используют электровиолончель в качестве ЭТОГО. Назовите ЭТО по-русски
или по-английски.

Ответ:
Палка для селфи.

Зачет:
Selfiestick; монопод; палка Нарцисса; narcisstick.

Источник:
   1. http://www.youtube.com/watch?v=P_g3sYGuuEA
   2. http://ru.wikipedia.org/wiki/Палка_для_селфи

Автор:
Сергей Коновалов

Вопрос 23:
Герой Симмонса Джон Бридженс, моряк, отождествлял себя в разные периоды
своей жизни с различными героями литературных произведений. В зрелые
годы, когда Джон поступил на службу в Королевский флот, один из его
близких друзей в приватной беседе усомнился в существовании ЕЕ, после
чего Джон понял, что его хитроумие больше не приносит плодов, и
дальнейшая карьера под вопросом. Назовите ЕЕ.

Ответ:
Пенелопа.

Источник:
Д. Симмонс. Террор.

Автор:
Иван Беляев

Вопрос 24:
В вопросе словами "СДЕЛАТЬ ЭТО" заменены два слова.
   Говоря о кадровых проблемах "Динамо", комментатор футбольного матча
сказал, что в защите сегодня вышли играть все, кто был способен СДЕЛАТЬ
ЭТО. Назовите Героя Советского Союза, который якобы не очень хорошо
ДЕЛАЛ ЭТО.

Ответ:
[Юрий Алексеевич] Гагарин.

Источник:
   1. Трансляция матча "Динамо" - "Спартак" на телеканале "Спорт",
25.10.2015 г.
   2. http://www.retroportal.ru/gagarin_14_04_1961.shtml

Автор:
Сергей Коновалов

Тур:
4 тур. "Конец перспективы" (Москва - Санкт-Петербург)

Редактор:
Владислав Король

Инфо:
Редактор благодарит за помощь в работе над пакетом Дмитрия Великова,
Александра Иванова, Тимура Кафиатуллина, Наталию Новыш, Константина
Сахарова, Ирину Устину, Владимира Цвингли, Дмитрия Богданова и команду
"Хомяки" (Белгород), Евгения Машерова и команду "16 тонн" (Москва).

Вопрос 1:
Внимание, в тексте вопроса два слова заменены одним.
   В рассказе "Конец перспективы" описывается альтернативная история, в
которой основным топливом является уголь, а о полезных свойствах нефти
никому не известно. При этом война 1914 года длилась лишь четыре месяца,
а Второй мировой вовсе не было. В финале рассказа один из героев
собирается найти способ получения топлива из нефти и разбогатеть.
Восстановите название рассказа.

Ответ:
"Конец прекрасной эпохи".

Комментарий:
Второй герой отказывается от предприятия, чувствуя недоброе, но первый
всё же решает попробовать. Слова "конец перспективы" взяты из известного
стихотворения Иосифа Бродского "Конец прекрасной эпохи". Кстати, есть
документальный фильм о Бродском с тем же названием.

Источник:
   1. http://haritonov.kulichki.net/stories/end.htm
   2. http://www.world-art.ru/lyric/lyric.php?id=7608

Автор:
Владислав Король

Вопрос 2:
   <раздатка>
   За шесть недель до этой безобразной сцены журналистке впервые
расскажут о проекте "Мобиле кооперато" и его гениальном творце,
чернорабочем ленинградской фабрики.
   </раздатка>
   Розданный вам немного измененный фрагмент одного произведения
нехарактерен для творческой манеры писателя. Так, вместо слова "творец"
легко можно было употребить слово "автор" - однако этого сделано не
было. Какое слово в тексте заменено словом "ленинградской"?

Ответ:
Таллиннской.

Зачет:
Таллинской.

Комментарий:
Сергей Довлатов старался не начинать слова в предложении с одной и той
же буквы, поэтому предложения в его текстах обычно не очень длинные. В
розданном тексте 21 слово, а в оригинале вообще 23 (вместо слов
"ленинградской фабрики" сказано "одной из таллиннских фабрик", так что
нарушений в этом предложении три: творца/таллиннских, и/из, о/одной).
Употребление слова "автор" вместо "творец" и небольшое сокращение фразы
позволило бы соблюсти порядок. Творческая жизнь Довлатова прошла в
Ленинграде, Таллине и Нью-Йорке. Документальный фильм "Конец прекрасной
эпохи" рассказывает не только о Бродском, но и о Довлатове, а недавно на
экраны вышел одноименный художественный фильм, снятый как раз по
довлатовскому "Компромиссу".

Источник:
С.Д. Довлатов. Компромисс. http://www.flibusta.is/b/133524/read

Автор:
Владислав Король

Вопрос 3:
В 1915 году юная англичанка Винифред Уильямс вышла замуж за
сорокашестилетнего Зигфрида, помогая тому скрыть нетрадиционную
сексуальную ориентацию. Через несколько лет по просьбе друга семьи
Винифред передала тому большое количество бумаги. Для чего друг
использовал эту бумагу?

Ответ:
Для написания "Моей борьбы".

Зачет:
По смыслу.

Комментарий:
Супруга единственного сына Рихарда Вагнера передала Адольфу Гитлеру в
тюрьму бумагу, на ней и был написан первый вариант "Mein Kampf". За 15
лет брака супруги родили четверых детей, их потомки сейчас занимаются
вагнеровским фестивалем в Байройте. Правда, утверждают, что Зигфрид не
перевоспитался.

Источник:
http://www.vokrugsveta.ru/vs/article/6149/

Автор:
Владислав Король

Вопрос 4:
Газета "Полезно для пенсионера" советует не брать в супермаркеты детей -
дескать, малыши могут незаметно подложить в тележку несколько ИХ,
которые вы обнаружите только расплачиваясь. Назовите ИХ сложносоставным
словом.

Ответ:
Киндер-сюрпризы.

Зачет:
Киндерсюрпризы.

Комментарий:
Так автор газеты "Полезно для пенсионера" назвал "мелкие вкусняшки,
которые любят дети". "Киндер-сюрприз" может оказаться "киндерсюрпризом"
от "киндер-сюрприза".

Источник:
Газета "Полезно для пенсионера", 2015, N 22.

Автор:
Владислав Король

Вопрос 5:
(pic: 20160295.jpg)
   Альбом словенской группы "Laibach" [лАйбах], название которого
закрыто черным прямоугольником, содержит своеобразные интерпретации
гимнов разных стран. Для жителя Любляны название диска является омонимом
названия животного. Назовите это животное по-словенски, по-немецки или
по-русски.

Ответ:
Volk.

Зачет:
Wolf; волк.

Комментарий:
Laibach - немецкое название Любляны, столицы Словении. По-немецки "volk"
[фольк] - "народ", а по-словенски "volk" [воук] - "волк", слова пишутся
одинаково.

Источник:
http://ru.wikipedia.org/wiki/Volk

Автор:
Владислав Король

Вопрос 6:
(pic: 20160295.jpg)
   Альбом словенской группы "Laibach" [лАйбах], название которого
закрыто черным прямоугольником, содержит своеобразные интерпретации
гимнов разных стран. По шуточному предположению автора вопроса, на
обложке диска пасутся ОНИ. Назовите ИХ двумя словами одинаковой длины,
начинающимися на соседние буквы алфавита.

Ответ:
Мирные народы.

Комментарий:
В стихотворении "Свободы сеятель пустынный..." Пушкин обращается к
пасущимся мирным народам. По мнению автора вопроса, обложка диска "Volk"
является своеобразной аллюзией на это стихотворение.

Источник:
   1. http://ru.wikipedia.org/wiki/Volk
   2. http://www.rvb.ru/pushkin/01text/01versus/0423_36/1823/0335.htm

Автор:
Владислав Король

Вопрос 7:
Французская группа "Peste Noire" [пэст нуАр] широко использует в своем
творчестве националистические идеи, что, впрочем, не помешало им долгое
время играть с барабанщиком индийского происхождения. Кстати говоря, в
одной из их песен ИКСЫ рифмуются с АЛЬФОЙ. Назовите ИКСОВ и АЛЬФУ
по-русски или на языке оригинала.

Ответ:
Арийцы, Марианна.

Зачет:
Aryan, Marianne.

Источник:
   1. http://www.metal-archives.com/board/viewtopic.php?p=1357837
   2. http://lyrics.wikia.com/wiki/Peste_Noire:Casse,_P%C3%AAches,_Fractures_Et_Traditions

Автор:
Дмитрий Великов

Вопрос 8:
   <раздатка>
   выдал бы тебе по пятое число семнадцать раз ремнем с армейской
пряжкой
   </раздатка>
   Перед вами цитата из композиции отечественной музыкальной группы.
Напишите слово, которое пропущено в цитате.

Ответ:
Двадцать.

Комментарий:
"По двадцать пятое число". Цитата из песни группы "25/17" "Будь белым
2".

Источник:
http://rap-text.ru/25_17/1748-iezekiil-2517-bud-belym-2-tekst-pesni.html

Автор:
Владислав Король

Вопрос 9:
Газета "Наш ИКС" включена в Федеральный список экстремистских
материалов. Все слова в названии газеты начинаются с одних и тех же двух
букв. Назовите ИКСА двумя словами.

Ответ:
Народный наблюдатель.

Комментарий:
Газета "Фёлькишер Беобахтер", печатный орган НСДАП, выходила с 1920 по
1945 год. Немецкое слово "Volk" вам хорошо известно. В Серпухове,
кстати, выходит городская газета "Народный наблюдатель", в Федеральный
список экстремистских материалов не включенная.

Источник:
   1. Федеральный список экстремистских материалов, пункты 83, 135, 136.
   2. http://ru.wikipedia.org/wiki/V%C3%B6lkischer_Beobachter

Автор:
Владислав Король

Вопрос 10:
   <раздатка>
   Съехал оттуда давно с переменою адреса,
   Чтобы забыть, словно сон, и начать жить по-новому.
   &nbsp;
   F4Band
   </раздатка>
   Перед вами фрагмент текста песни "АЛЬФА двадцать лет спустя" и
название группы, которая ее исполняла. Назовите АЛЬФУ двумя словами.

Ответ:
Улица Ленина.

Комментарий:
F4 - намек на Ф.Ч., инициалы Федора Чистякова. Спустя много лет Чистяков
написал продолжение своей знаменитой песни "Улица Ленина". Размер у
обеих песен одинаковый и довольно своеобразный (если прочитать цитату
вслух, должно помочь). Кстати, образ жизни Чистякова за это время
действительно сильно изменился.

Источник:
http://megalyrics.ru/lyric/fiodor-chistiakov/ulitsa-lienina-20-liet-spustia.htm

Автор:
Владислав Король

Вопрос 11:
В одной песне упоминаются сиреневая ветка московского метро и ТАКАЯ
ветка казанского метро. Какая - ТАКАЯ?

Ответ:
Единственная.

Комментарий:
Эквиритмичные слова.

Источник:
Текст песни "Эсер по жизни" ВИА "Кобыла и Трупоглазые Жабы Искали Цезию,
Нашли Поздно Утром Свистящего Хна".

Автор:
Владислав Король

Вопрос 12:
   <раздатка>
   Stra&szlig;enschlact - уличная битва
   Schwuchteln lieben - любить пидора
   machten "Safer-Sex" - заниматься "безопасным сексом"
   </раздатка>
   На раздатке вы видите слова, которыми оканчиваются третья, пятая и
седьмая строчки песни неонацистской группы "Landser" [ландсер] о
"свинках-коммунистах". Назовите произведение, вдохновившее группу на
создание этой песни.

Ответ:
"Десять негритят".

Комментарий:
От всего перечисленного персонажи поочередно умирают. Слова на раздатке
рифмуются соответственно с "acht" [ахт] (восемь), "sieben" [зИбен]
(семь) и "sechs" [зэкс] (шесть).

Источник:
http://www.jooov.net/text/1113235/Landser-10_kleine_Kommi-Schweine.htmls

Автор:
Владимир Цвингли

Вопрос 13:
(pic: 20160296.jpg)
   Хельмут Херцфельд родился в июне 1891 года в Берлине. Письмо, в
котором неоднократно упоминается топор, было опубликовано задолго до
рождения Хельмута Херцфельда. Назовите адресата этого письма.

Ответ:
[Александр] Герцен.

Комментарий:
В 1916 году "национал-предатель" Херцфельд сменил имя и фамилию -
фамилию он просто перевел на язык противника. Кстати, Винифред Уильямс
сменила фамилию с английской на немецкую примерно в то же время. После
прихода к власти нацистов художник эмигрировал в Прагу, а затем и в
Лондон, где провел всю войну. "Письмо из провинции" 1860 года призывало
"Колокол" "не благовестить к молебну, а звонить набат" и звать Русь к
топору. Герцен опубликовал письмо, впрочем, отвергнув призывы "к
топору", "пока останется хоть одна разумная надежда на развязку без
топора", а также сочтя невозможным и безнравственным "звать к топорам"
из Лондона. Фамилия Герцен также происходит от немецкого слова "Herz"
[херц] - сердце.

Источник:
   1. http://de.wikipedia.org/wiki/John_Heartfield
   2. http://lawbooks.news/istoriya-jurnalistiki_852/pismo-provintsii.html

Автор:
Владислав Король

Вопрос 14:
В июне 2014 года в печати появилась информация о бразильской семье, все
члены которой обладают характерной редкой особенностью. Пресса отмечала,
что это семейство наглядно олицетворяет мечту всех бразильцев. Назовите
писателя, один из заглавных героев произведения которого обладал той же
особенностью.

Ответ:
[Виктор] Пелевин.

Комментарий:
У всех членов семьи по шесть пальцев на руке. Однако выиграть шестой
титул на домашнем чемпионате бразильцы не смогли. Затворник и Шестипалый
- герои повести Виктора Пелевина.

Источник:
"Метро", 25.06.2014 г.

Автор:
Владислав Король, Владимир Цвингли

Вопрос 15:
Июньское наступление русской армии на ИКС потерпело неудачу, и вскоре
ИКС подал в отставку. Что мы заменили на ИКС?

Ответ:
Львов.

Комментарий:
Наступление в Галиции летом 1917 года. Георгий Львов возглавлял
Временное правительство до 7 июля. Город ни разу не переименовывался,
просто в разных языках его называют по-разному (так, на русских картах
1914 года он Львовъ, да и существовавшая в 1914-1915 годах губерния была
Львовской).

Источник:
http://ru.wikipedia.org/wiki/Июньское_наступление

Автор:
Владислав Король

Вопрос 16:
   <раздатка>
   .....in6
   </раздатка>
   Перед вами фрагмент адреса интернет-сообщества почитателей одной
группы. Напишите пять букв, замененных точками.

Ответ:
Death.

Комментарий:
Сообщество посвящено группе "Death In June" (которая, как и "Laibach",
широко использует тоталитарные образы и символику), шестерка означает
"июнь". С упоминания июня начинаются первые три вопроса этого блока.

Источник:
http://vk.com/deathin6

Автор:
Владислав Король

Вопрос 17:
Дуплет.
   Два вопроса, для обсуждения первого из них будет дано меньше времени,
чем обычно. Однако суммарно на оба вопроса у вас будет 60 секунд.
   1. Разноцветные кресла, показанные в этом российском фильме, - явный
анахронизм. Назовите этот фильм.
   2. Компактную пудру, которой пользовалась героиня этого фильма,
иногда считают анахронизмом. Но это не так - изобретена она была задолго
до войны. Назовите этот фильм.

Ответ:
   1. "Легенда N 17".
   2. "Тегеран-43".

Комментарий:
   1. Разноцветные кресла, создающие впечатление аншлага, впервые
появились на стадионах, построенных к Евро-2004 в Португалии, в
Советском Союзе их тем более не было. Некоторые сцены фильма "Легенда N
17" снимались без зрителей в современном ледовом дворце.
   2. "Movie 43" - незачет.
   На первый вопрос дуплета было дано 17 секунд, на второй - 43.

Источник:
   1. Х/ф "Легенда N 17" (2012), реж. Николай Лебедев.
   2. Х/ф "Тегеран-43" (1980), реж. Александр Алов, Владимир Наумов, 2-я
серия, 18-я минута.
   3. http://shy.in.ua/blog/istoriya-poyavleniya-na-svet-kompaktnoj-pudry/
   4. http://kosmetista.ru/blog/concilium/3880.html

Автор:
Владислав Король

Вопрос 18:
В надписи на пьедестале памятника пропавшим без вести солдатам была
допущена орфографическая ошибка. Надежда Конобеевская пишет, что
разрешение чиновников на замену таблички затерялось где-то в череде
согласований - короче говоря, "СДЕЛАЛО ЭТО". Ответьте абсолютно точно -
какие два слова заменены на "СДЕЛАЛО ЭТО"?

Ответ:
Пропало безвести.

Комментарий:
На табличке слова "без вести" были написаны слитно.

Источник:
http://rusrep.ru/article/2014/11/06/yazyik-dobeshenstva-dovedet

Автор:
Владислав Король

Вопрос 19:
Автор одного стихотворения опечалена, что из девочки в кроссовках
превратилась в женщину в тУфлях, с которыми ничего... Закончите это
высказывание двумя словами.

Ответ:
Не рифмуется.

Комментарий:
В песне, написанной по мотивам этого стихотворения, упоминается мужчина
в ботинках. Впрочем, это художественное преувеличение, рифмы к словам
"туфли" и "ботинки" подобрать можно.

Источник:
http://naddoo.livejournal.com/30280.html

Автор:
Владислав Король

Вопрос 20:
Песня "Будь белым" группы "25/17" включена в Федеральный список
экстремистских материалов. На одном из постеров группы изображены четыре
скрещенных ИКСА. Назовите ИКС одним словом.

Ответ:
Топор.

Комментарий:
Скрещенные, правда, не в виде свастики, как на антифашистском плакате
Джона Хартфилда "Кровь и железо" (который, кстати, был использован в
оформлении одного из альбомов группы "Laibach"). Песня была признана
экстремистской решением Эжвинского районного суда города Сыктывкара в
2012 году. Интересно, что такой песни у группы нет, есть песня "Будь
белым 2", процитированная ранее, а песню "Будь белым" исполняла группа
"Отрицательное влияние", в которой играл один из участников "25/17".
Также интересно, что ничего экстремистского в обеих песнях нет, они в
шуточной форме призывают молодых рэп-исполнителей не подражать
темнокожим коллегам, а делать что-то свое.

Источник:
   1. Федеральный список экстремистских материалов, пункт 1730.
   2. http://via-midgard.info/news/2517-topory-2013.htm
   3. http://blednyi-2517.livejournal.com/242348.html

Автор:
Владислав Король

Вопрос 21:
(pic: 20160297.jpg)
   Подпись к этой картинке состоит из двух глаголов, начинающихся на
одну и ту же букву. Воспроизведите эту подпись.

Ответ:
"Покушать принес".

Зачет:
В любом порядке; также засчитывать синонимичные ответы, удовлетворяющие
форме вопроса.

Комментарий:
Что принес в шкатулочке зеленый слоник, автору вопроса неизвестно.

Источник:
https://www.instagram.com/p/28Y8W4KXh4/

Автор:
Владислав Король

Вопрос 22:
Дуплет.
   (pic: 20160298.jpg)
   1. После выхода диска "Саломея" недоброжелатели дали Елене Ваенге
восьмибуквенное прозвище. Какое?
   <раздатка>
   Мовсисян
   </раздатка>
   2. После перехода в московский "Спартак" недоброжелатели дали Юре
Мовсисяну восьмибуквенное прозвище. Какое?

Ответ:
   1. Сраломея.
   2. Мясосвин.

Зачет:
   1. Ссаломея.
   2. Свиномяс.

Комментарий:
   1. Не самый удачный шрифтовой дизайн, чего уж там.
   2. Анаграмма к фамилии игрока.

Источник:
   1. http://pikabu.ru/story/kovarnaya_bukva_2489598
   2. http://eagle-r.livejournal.com/383911.html

Автор:
Владислав Король

Вопрос 23:
---

Ответ:
---

Вопрос 24:
---

Ответ:
---

Тур:
5 тур. "Захер нам Мазох" (Тула)

Вопрос 1:
Цитата из поста под названием "Настоящая забота" с сайта pikabu.ru:
"Первый день у меня дома. Он серый и невзрачный. Я дал ему сахарок.
Через пару часов он уже стал таким игривым! Я закутал его в пушистое
полотенце и разместил у батареи. Сейчас он уже иногда попискивает".
Назовите то, о чем заботится автор поста.

Ответ:
Квас.

Источник:
http://pikabu.ru/story/nastoyashchaya_zabota_3106414

Автор:
Алексей Борисов (Тула)

Вопрос 2:
В комментариях к статье сайта meduza.io, повествующей об эстонском
госслужащем, которого обвинили в растрате 140 евро на покупку обуви,
один из пользователей пошутил, изменив одну букву в названии известного
произведения. Воспроизведите измененное название.

Ответ:
"Коп в сапогах".

Источник:
https://vk.com/meduzaproject?w=wall-76982440_413557_r413569

Автор:
Алексей Борисов (Тула)

Вопрос 3:
Действие повести Виктора Пелевина "День бульдозериста" происходит в
мирное время в обществе, близком к советскому. Один из главных героев,
жалуясь на громкую музыку у соседей, спрашивает: "Кто это так
трудячит?". Назовите группу двумя словами, начинающими на соседние буквы
алфавита.

Ответ:
"Ласковый май".

Комментарий:
Если есть труд и мир, то куда же без мая.

Источник:
В.О. Пелевин. День бульдозериста.
http://pelevin.nov.ru/pov/pe-buld/2.html

Автор:
Павел Сенин (Тула)

Вопрос 4:
В XVI веке ИХ делили на три типа. Первый - люди с богатым воображением -
художники, поэты, ремесленники. Второй - люди, у которых рассудок
преобладает над чувством, - ученые, государственные деятели. Третий -
люди, у которых преобладает интуиция, - богословы и философы. Альбрехт
Дюрер относил себя к первому типу. Назовите ИХ.

Ответ:
Меланхолики.

Комментарий:
Надпись на гравюре - "MELENCOLIA I".

Источник:
http://ru.wikipedia.org/wiki/Меланхолия_(гравюра_Дюрера)

Автор:
Павел Сенин (Тула)

Вопрос 5:
   <раздатка>
   Не так лих ПРОПУСК, как его малюют.
   </раздатка>
   После одной из уверенных побед сборной России футбольный комментатор
Василий Уткин оставил у себя в твиттере розданное вам сообщение. Какие
восемь букв мы заменили на ПРОПУСК на раздаточном материале?

Ответ:
тенштейн.

Комментарий:
Лихтенштейн действительно оказался не так уж и лих. Наша сборная
обыграла его со счетом 7:0.

Источник:
https://twitter.com/radioutkin/status/641518595051266048/

Автор:
Александр Жабров (Тула)

Вопрос 6:
На иллюстрации к новости о том, что списанный океанский лайнер превратят
в офисное здание, присутствует голодное млекопитающее, готовое съесть
корабль. При этом упоминается ОН. ЕГО название происходит от слова со
значением "блуждающий". Назовите ЕГО одним словом.

Ответ:
Планктон.

Комментарий:
Млекопитающее - это кит, который питается планктоном (в данном случае,
видимо, офисным). Планктон - разнородные, в основном мелкие организмы,
свободно дрейфующие в толще воды и неспособные сопротивляться течению.

Источник:
   1. https://vk.com/obrazovach?w=wall-74404187_248460
   2. http://ru.wikipedia.org/wiki/Планктон

Автор:
Александр Жабров (Тула)

Вопрос 7:
В вопросе есть замены.
   ОНА и ОНО - части отложенного налога на прибыль. ОНА уменьшает налог,
ОНО - увеличивает. Напишите, что мы заменили на "ОНА" и на "ОНО", в
сокращенной форме или полностью.

Ответ:
ОНА и ОНО.

Зачет:
Отложенный налоговый актив и отложенное налоговое обязательство.

Источник:
Положение по бухгалтерскому учету "Учет налогов на прибыль организаций"
ПБУ 18.02.

Автор:
Елена Кибанова (Тула)

Вопрос 8:
(pic: 20160299.jpg)
   Перед вами хозяйственная сумка, разработанная компанией "Tisch5". Для
какой цели у этой сумки три ручки?

Ответ:
Чтобы ребенок мог держаться за ручку и не потеряться.

Зачет:
По смыслу, с упоминанием ребенка.

Комментарий:
(pic: 20160300.jpg)

Источник:
http://www.tisch5.de/

Автор:
Елизавета Феофилова (Тула)

Вопрос 9:
(pic: 20160301.jpg)
   На розданном рисунке от вас скрыто название группы, основанной в 1994
году. Напишите это название.

Ответ:
"Спайс Гёрлз".

Зачет:
"Spice Girls".

Источник:
https://vk.com/photo19014119_341482868

Автор:
Владислав Волошин (Тула)

Вопрос 10:
В романе Дугласа Адамса бармен ожидает начала матча с участием
"Арсенала", а рабочие собираются расчистить место для нового шоссе.
Намеченным планам не суждено сбыться. Возможно потому, что случился
конец света, а возможно потому, что вышеописанные события должны были
произойти именно тогда. Когда?

Ответ:
В четверг после дождя.

Зачет:
По смыслу, с упоминанием четверга и дождя.

Комментарий:
В романе планета Земля была уничтожена в четверг. В городе, где проживал
главный герой, накануне прошел дождь.

Источник:
Д. Адамс. Автостопом по Галактике. http://www.flibusta.is/b/155848/read

Автор:
Владислав Волошин (Тула)

Вопрос 11:
Прослушайте отрывок из стихотворения Джона Апдайка:
   Но в крепость кристаллической структуры
   Закрыт был вход и не было ключей,
   Покуда не проник сквозь амбразуры
   Туда ИКС.
   Назовите ИКС тремя словами.

Ответ:
Пучок рентгеновских лучей.

Комментарий:
ИКС должен служить подсказкой, указывающей на принятое в западных
странах обозначение рентгеновских лучей.

Источник:
Дж. Апдайк. Танцы твердых тел. http://www.flibusta.is/b/284731/read

Автор:
Павел Сенин (Тула)

Вопрос 12:
Автора вопроса трудно упрекнуть в суеверности, но когда черниговская
девушка предложила ему встретиться на валу у тринадцатой пушки, он
воспринял это как дурной знак. По какой причине?

Ответ:
Их там всего двенадцать.

Зачет:
По смыслу.

Источник:
ЛОАВ.

Автор:
Владислав Волошин (Тула)

Вопрос 13:
Джон Стейнбек сравнивает новорожденную ЕЕ с вытянутой хирургической
иглой. Назовите ЕЕ.

Ответ:
Луна.

Источник:
Дж. Стейнбек. Зима тревоги нашей. http://www.flibusta.is/b/276529/read

Автор:
Павел Сенин (Тула)

Вопрос 14:
В описании одной игры говорится, что она основана на реальных событиях и
повествует: об эпической гонке, которая произошла задолго до того
момента, который вы могли бы запомнить; о миллионах участников и всего
одном победителе; о призе ценою в жизнь. Ответьте одним словом: что
происходит в результате этого "соревнования"?

Ответ:
Зачатие.

Зачет:
Оплодотворение.

Комментарий:
Речь идет о гонке сперматозоидов.

Источник:
http://www.spermania.com/

Автор:
Александр Жабров (Тула)

Вопрос 15:
Прослушайте шутку Джимми Карра: "Стивен Хоккинг - практически
получеловек-полукомпьютер. Уверен: когда он умрет, это будет из-за...".
Закончите шутку одним словом.

Ответ:
"... вируса".

Источник:
https://vk.com/british_prikol?w=wall-88497493_91240

Автор:
Владислав Волошин (Тула)

Вопрос 16:
"Девушка <...> была хороша: щеки - как розы, губы - алые гвоздики, глаза
цвета васильков, а волосы - золотистые пряди льна". Так Елена Коровина
писала про ЕГО возлюбленную Каролину. Назовите ЕГО.

Ответ:
[Джузеппе] Арчимбольдо.

Источник:
Е.А. Коровина. Великие загадки мира искусства. 100 историй о шедеврах
мирового искусства. http://www.flibusta.is/b/278810/read

Автор:
Алексей Морозов (Тула)

Вопрос 17:
[Ведущему: читать первое предложение медленно, под запись.]
   В фантастическом фильме 2014 года персонаж, обладающий
сверхчеловеческой скоростью, играет в настольный теннис. Какие три
слова, начинающиеся на одну и ту же букву, мы пропустили в предыдущем
предложении?

Ответ:
Сам с собой.

Комментарий:
Мутант Ртуть из "Людей Икс" настолько быстр, что играет в настольный
теннис сам с собой, успевая поочередно отбивать мячик по обе стороны
стола.

Источник:
Х/ф "Люди Икс: Дни минувшего будущего" (2014), реж. Брайан Сингер.

Автор:
Алексей Морозов (Тула)

Вопрос 18:
Дуплет.
   1. Свои клубные цвета эта российская футбольная команда получила
благодаря торговым партнерам из Италии, предоставившим игровую форму.
Назовите эту команду.
   2. По одной из версий, свои клубные цвета этот европейский футбольный
гранд получил благодаря его основателю, швейцарцу по происхождению, на
родине являвшемуся бессменным капитаном другой команды. Напишите
название этого гранда.

Ответ:
   1. "Амкар".
   2. "Барселона".

Комментарий:
   1. В ответ на просьбу предоставить игровую форму пермской команде
итальянские партнеры из Милана предложили игровые комплекты одноименного
клуба. Как известно, традиционные цвета "Милана" и "Амкара" - красный и
черный.
   2. Основатель "сине-гранатовых" Жоан Гампер был капитаном "Базеля",
имеющего схожую клубную цветовую гамму.

Источник:
   1. http://ru.wikipedia.org/wiki/Амкар
   2. http://ru.wikipedia.org/wiki/Гампер,_Жоан

Автор:
Владислав Волошин (Тула)

Вопрос 19:
После одного скандала с группой школьниц в Интернете появилось шуточное
двустишие: "Сегодня ты ПРОПУСК, // А завтра Родину отверг". Какие два
слова, начинающиеся на одну и ту же букву, мы пропустили?

Ответ:
"... танцуешь тверк...".

Комментарий:
Двустишие пародирует аналогичный известный стишок про джаз и посвящено
танцу оренбургских школьниц "Пчелки и Винни-Пух", которым зачем-то
заинтересовался Следственный комитет.

Источник:
http://lenta.ru/articles/2015/04/14/bees/

Автор:
Алексей Морозов (Тула)

Вопрос 20:
Отвечая на вопрос "Чем пахнет ваш город?", жители областного центра
упоминают сладости, горячий напиток и порох. Назовите этот город.

Ответ:
Тула.

Комментарий:
Тульские пряники, чай из самоваров и оружие - бренды города, известные
далеко за пределами региона. Чтобы вы убедились в правдивости, автор
вопроса приглашает всех в гости. Только без самовара. :-)

Источник:
http://tula.aif.ru/society/chem_pahnet_tula_v_regione_mogut_poyavitsya_duhi_s_mestnym_aromatom

Автор:
Антонина Позднякова (Тула)

Вопрос 21:
По мнению Марины Цветаевой, женщины говорят о НЕЙ и молчат о НИХ, а
мужчины - наоборот. И всё же многие современники Цветаевой посвящали ЕЙ
свои стихи, тогда как стихи о НИХ малоизвестны. Назовите ЕЕ и ИХ
однокоренными словами.

Ответ:
Любовь и любовники.

Зачет:
Любовь и любовницы.

Источник:
http://ru.wikiquote.org/wiki/Марина_Ивановна_Цветаева

Автор:
Антонина Позднякова (Тула)

Вопрос 22:
Общий сюжет "Властелина колец" не чужд и нашей отечественной
мультипликации. В мультфильме есть и само кольцо, и братство кольца,
изображенное в виде пары домашних животных, и свой безумный самодержец.
Роль же Саурона исполняет вполне обычная, охочая до счастья за чужой
счет девушка. Назовите ее имя.

Ответ:
Ульянка.

Источник:
Мультфильм "Волшебное кольцо" (1979).
http://www.youtube.com/watch?v=-LqD-W9kzYw

Автор:
Иван Гусаров (Тула)

Вопрос 23:
Как известно, женщины дадут мужчинам тысячу очков форы в том, сколько о
них выдумано стереотипов. Однако, вопреки одному из самых популярных
стереотипов, такие девушки, как Сьюзи Вольф, Джованна Амати, Дезире
Уилсон и другие, отлично проявили себя в ЭТОМ наравне с мужчинами.
Назовите ЭТО.

Ответ:
Автогонки.

Комментарий:
Перечисленные девушки - успешные гонщицы. Слово "фора" - маленькая
подсказка.

Источник:
http://veddro.com/2015/03/samyie-izvestnyie-gonshhitsyi/

Автор:
Иван Гусаров (Тула)

Вопрос 24:
   <раздатка>
   "Ein kleines bisschen Horrorschau"
   </раздатка>
   На розданном вам материале - название одного из альбомов немецкой
панк-рок-группы "Die Toten Hosen" [ди тОтен хОзен], вероятно
пользующейся особенной популярностью в среде тинейджеров. Какому
произведению посвящен этот альбом?

Ответ:
"Заводной апельсин".

Зачет:
"A Clockwork Orange".

Комментарий:
Одно из слов на раздаточном материале могло вам напомнить русское слово
"Хорошо". Из подобных слов состоял вымышленный язык подростков из романа
- "надсат". Название языка - модифицированное окончание русских
числительных от "одиннадцати" до "девятнадцати". Объясняется это тем,
что носителями надсата в "Заводном апельсине" были подростки
(nadtsatyje) - "тинейджеры" (teenagers, буквально "надцатилетние"; или
сокращенно - "тины", teens).

Источник:
http://ru.wikipedia.org/wiki/Ein_kleines_bisschen_Horrorschau

Автор:
Владислав Волошин (Тула)

Тур:
6 тур. "Белый шум" (Санкт-Петербург)

Редактор:
Данила Аладин (Пущино)

Вопрос 1:
(pic: 20160302.jpg)
   Мы не спрашиваем, какое название мы скрыли от вас на розданном
фрагменте карты. Какой объект мы заменили на данной раздатке?

Ответ:
Далек.

Комментарий:
В песне поется: "Издалека долго течет река Волга". Далеки - герои
сериала "Доктор Кто". Получим: из далека долго течет река Волга. Тардис
- космический корабль Доктора Кто. Далек заменен на ТАРДИС. А скрыт
исток Волги - деревня Волговерховье.

Источник:
   1. Яндекс.карты.
   2. http://ru.wikipedia.org/wiki/ТАРДИС

Автор:
Александр Маничев (Санкт-Петербург)

Вопрос 2:
Один из видов общественного транспорта в странах Юго-Восточной Азии и
Океании - автобусы, ходящие по специально выделенной для них полосе.
Название маршрутной сети таких автобусов в Джакарте и Аделаиде является
анаграммой другого общественного транспорта, исправно работающего уже
почти полтора столетия. Как называется вышеупомянутая автобусная сеть?

Ответ:
"Busway".

Комментарий:
Subway - название метрополитена Нью-Йорка, начавшего работу в 1868 году.

Источник:
   1. http://forum.awd.ru/viewtopic.php?f=868&t=210249&start=340
   2. http://ru.wikipedia.org/wiki/О-Бан
   3. http://en.wikipedia.org/wiki/O-Bahn_Busway
   4. http://ru.wikipedia.org/wiki/Нью-Йоркский_метрополитен

Автор:
Александр Маничев (Санкт-Петербург)

Вопрос 3:
Внимание, в вопросе есть замены.
   Логично, что доктор из произведения середины 1990-х годов, согласно
Женевским конвенциям, является докторантом. Какие слова мы заменили в
предыдущем предложении?

Ответ:
Комбат, комбатант.

Комментарий:
Песня "Комбат" группы "ЛЮБЭ" вышла в 1995 году. Комбатант - лицо,
принимающее непосредственное участие в боевых действиях в составе
вооруженных сил одной из сторон международного вооруженного конфликта.

Источник:
   1. http://ru.wikipedia.org/wiki/Комбат_(песня)
   2. http://ru.wikipedia.org/wiki/Комбатант
   3. http://ru.wikipedia.org/wiki/Четвёртая_женевская_конвенция

Автор:
Александр Маничев (Санкт-Петербург)

Вопрос 4:
Дуплет.
   1. Милую жительницу Японии, проживающую вдали от города, можно
назвать словом, полученным из названия произведения XIX века путем
удаления из него пяти букв. Назовите эти буквы.
   2. Милую жительницу благородного происхождения, проживавшую в Бресте
или Львове в 1920-х - 1930-х годах, можно назвать словом, полученным из
названия произведения XIX века путем удаления из него двух букв.
Назовите эти буквы.

Ответ:
   1. барыш.
   2. ть.

Комментарий:
   1. Милая женщина, красивая, японка - просто ня! Произведение -
"Барышня-крестьянка".
   2. Брест и Львов входили в состав Польши, так называемые "Восточные
кресы". Произведение - "Барышня-крестьянка".

Источник:
   1. http://ru.wikipedia.org/wiki/Ня
   2. http://ru.wikipedia.org/wiki/Восточные_кресы
   3. http://ru.wikipedia.org/wiki/Барышня-крестьянка

Автор:
Александр Маничев (Санкт-Петербург)

Вопрос 5:
(pic: 20160303.jpg)
   Внимание, в вопросе есть замены.
   Перед вами список ИКСОВ, проводившихся в 2015 году. Как видно из
раздатки, каждый ИКС можно назвать ТАКИМ. Мы не спрашиваем, что мы
заменили на ИКС. Назовите любой из ИКСОВ, которые мы заменили знаком
вопроса.

Ответ:
Савёловский.

Зачет:
Павелецкий.

Комментарий:
ИКС - это ЧГК-фестиваль. Проводится параллель с вокзалами Москвы:
Петербургский (Ленинградский) фестиваль "Белые ночи", Ярославский -
"Нехрустальная сова", Казанский - Открытый Кубок КНИТУ, Курский -
Фестиваль памяти Сергея Некрасова, Киевский - Открытый Молодежный Кубок
Киева, Белорусский - "Нестерка", Рижский - "Знатокиада. Рижские
каникулы".

Источник:
   1. http://ru.wikipedia.org/wiki/Железнодорожные_вокзалы_Москвы
   2. http://rating.chgk.info/tournaments.php

Автор:
Александр Маничев (Санкт-Петербург)

Вопрос 6:
На граффити в одном из московских переулков дым, идущий из труб,
образует лицо. В предыдущем предложении мы пропустили несколько слов, в
том числе две фамилии. Назовите любую из пропущенных фамилий.

Ответ:
Потёмкин.

Зачет:
Эйзенштейн.

Комментарий:
(pic: 20160304.jpg)
   Дым, идущий из труб "Потёмкина", образует лицо режиссера Сергея
Эйзенштейна.

Источник:
http://cr2.livejournal.com/375428.html

Автор:
Александр Маничев (Санкт-Петербург)

Вопрос 7:
Дуплет.
   1. В песне на стихи Беллы Ахмадулиной ОНИ оправляют голубенькие
перья. Назовите ИХ двумя словами на две одинаковых буквы.
   2. Молодых героев картины "Завтрак на траве" можно назвать ИМИ.
Назовите ИХ двумя словами на две одинаковых буквы.

Ответ:
   1. Девочки Дега.
   2. Мальчики Мане.

Комментарий:
Девочками Дега Белла Ахмадулина назвала танцовщиц в голубых одеждах на
картине Эдгара Дега "Голубые танцовщицы".

Источник:
   1. http://www.world-art.ru/lyric/lyric.php?id=3810
   2. http://ru.wikipedia.org/wiki/Голубые_танцовщицы
   3. http://ru.wikipedia.org/wiki/Завтрак_на_траве_(картина_Мане)

Автор:
Александр Маничев (Санкт-Петербург)

Вопрос 8:
В одном из своих произведений Василий Звягинцев пишет: "Начиная с
1969-го... человечество словно подменили. Во всём мире сразу как бы
исчез "творческий запал", кураж, стремление к новым рубежам, вообще к
творчеству любого рода... Осталась только страсть к потребительству и
идея, что "права личности выше прав государства, нации, вообще земной
цивилизации"". Можно сказать, что человечество сделало неправильный ИКС.
Назовите ИКС коротким словом.

Ответ:
Шаг.

Источник:
В.Д. Звягинцев. Величья нашего заря.
https://books.google.ru/books?id=vKcyBQAAQBAJ&pg=PT122#v=onepage&q&f=false

Автор:
Александр Маничев (Санкт-Петербург)

Вопрос 9:
   <раздатка>
   СНАШЕ
   СНАШИВАТЬ
   СНАЩАТЬСЯ
   СНЕВОЛИВАТЬ
   СНЕДЬ
   СНЁМ
   СНЕСТИ
   СНЕТ
   СНЕТИКУ
   </раздатка>
   Внимание, в вопросе есть замена.
   Перед вами фрагмент из "Толкового словаря живого великорусского
языка" Владимира Ивановича Даля. Заметно, что из списка СДЕЛАЛ ЭТО ОН.
По свидетельству знакомой со школьной скамьи известного племянника, ОН
СДЕЛАЛ ЭТО с задержкой. Что мы заменили на "ОН СДЕЛАЛ ЭТО"?

Ответ:
Снег выпал.

Комментарий:
   "Снег выпал только в январе
   На третье в ночь. Проснувшись рано,
   В окно увидела Татьяна
   Поутру побелевший двор".

Источник:
   1. В.И. Даль. Толковый словарь живого великорусского языка.
   2. А.С. Пушкин. Евгений Онегин.
http://www.rvb.ru/pushkin/01text/04onegin/01onegin/0836.htm

Автор:
Александр Маничев (Санкт-Петербург)

Вопрос 10:
Назовите фамилию художественного персонажа, сына Израиля, родившегося,
как следует из слов его свояченицы, в 1860 или 1859 году.

Ответ:
Шниперсон.

Комментарий:
Как следует из личного дела Василия Кроликова, он родился в 1953 году.
По словам его тети Прасковьи Алексеевны, отец Кроликова, Иван Израилевич
Шниперсон, погиб в день его рождения в возрасте 93 лет. Следовательно,
год рождения И.И. Шниперсона - 1860 или 1859. Свояченица - сестра жены.

Источник:
Х/ф "Ширли-мырли" (1995), реж. Владимир Меньшов.

Автор:
Александр Маничев (Санкт-Петербург)

Вопрос 11:
Внимание, в вопросе есть замена.
   Для путешествий АЛЬФы в пространстве и времени необходимо огромное
количество энергии. Однако это неудивительно, ведь АЛЬФА содержит
звезду. Напишите латиницей или кириллицей, что мы заменили на АЛЬФУ.

Ответ:
TARDIS.

Зачет:
ТАРДИС.

Комментарий:
Слово "tardis" содержит в себе "star" - "звезду".

Автор:
Александр Маничев (Санкт-Петербург)

Вопрос 12:
Можно сказать, что употребление таких слов, как "насекомое", "кислород"
и "внутримышечный", - это ОНО. Для части НЕГО также требуется ОНО.
Назовите ЕГО шестнадцатибуквенным словом.

Ответ:
Импортозамещение.

Комментарий:
Слова "насекомое", "кислород" и "внутримышечный" получены поморфемным
переводом иностранного слова на русский язык. Сейчас принято говорить об
импортозамещении. Нетрудно заметить, что в самом слове
"импортозамещение" один из корней иностранный. Кстати, в самом вопросе
нет иностранных слов.

Источник:
   1. http://ru.wikipedia.org/wiki/Калька_(лингвистика)
   2. http://ru.wikipedia.org/wiki/Кислород

Автор:
Александр Маничев (Санкт-Петербург)

Вопрос 13:
   <раздатка>
|   Цельсий    14.12.2015
|   Фаренгейт  10.09.2014
|   Кельвин    ----------
|   Реомюр     14.01.2016
   </раздатка>
   Внимание, вопрос задает Джон Рокфеллер!
   Числовые показатели данной таблицы соответствует некой норме. Мы
надеемся, что будут данные и по Кельвину. Значениям чего соответствует
эта норма?

Ответ:
Цена на нефть марки Brent в долларах США.

Комментарий:
Норма - температура человеческого тела: 36.6 по Цельсию, 97.88 по
Фаренгейту, 309.6 по Кельвину, 29.28 по Реомюру. Цена барреля нефти
принимала указанные значения в даты, указанные в таблице. Данных
значений цена достигала в ходе своего падения.

Источник:
   1. http://www.neftonline.com
   2. http://ru.wikipedia.org/wiki/Standard_Oil
   3. http://ru.wikipedia.org/wiki/Рокфеллер,_Джон_Дэвисон

Автор:
Александр Маничев (Санкт-Петербург)

Вопрос 14:
Внимание, в вопросе есть замена.
   В названии одного из районов Москвы можно найти два продукта. Один из
них производится преимущественно в Испании, второй - в Италии, Новой
Зеландии и Чили. Назовите оба этих продукта.

Ответ:
Хамон, киви.

Комментарий:
Из букв слова "Хамовники" можно составить слова "хамон" и "киви".

Источник:
   1. http://ru.wikipedia.org/wiki/Хамовники_(район_Москвы)
   2. http://ru.wikipedia.org/wiki/Хамон_(блюдо)
   3. http://ru.wikipedia.org/wiki/Киви_(фрукт)

Автор:
Александр Маничев (Санкт-Петербург)

Вопрос 15:
Можно сказать, что главной целью алхимиков были метаморфозы. Какие три
буквы мы пропустили в предыдущем предложении?

Ответ:
лло.

Комментарий:
Например, превращение свинца в золото.

Автор:
Александр Маничев (Санкт-Петербург)

Вопрос 16:
Дуплет.
   1. Этот крейсер, участник Русско-японской войны, наличием трех
дополнительных букв отличается от марки автомобиля отечественного
производства. Напишите эти три буквы.
   2. Этот крейсер, участник Русско-японской войны, одной буквой
отличается от марки автомобиля японского производства. Назовите эту
марку.

Ответ:
   1. пал.
   2. Ниссан.

Комментарий:
   1. Крейсер "Паллада" и "Лада".
   2. Крейсер "Ниссин" и "Ниссан".

Источник:
   1. http://ru.wikipedia.org/wiki/Паллада_(бронепалубный_крейсер)
   2. http://ru.wikipedia.org/wiki/Lada
   3. http://ru.wikipedia.org/wiki/Ниссин_(крейсер)
   4. http://ru.wikipedia.org/wiki/Nissan

Автор:
Александр Маничев (Санкт-Петербург)

Вопрос 17:
(pic: 20160305.jpg)
   Перед вами мозаичное панно, сюжет которого - индустриальный пейзаж
Петербурга начала XX века. Многие видят на нем ЕЕ. О коллегах ЕЕ вы
узнали из предыдущих вопросов. Назовите ЕЕ.

Ответ:
"Аврора".

Зачет:
Крейсер "Аврора".

Комментарий:
Действительно, силуэты заводов напоминают военный корабль. В предыдущих
вопросах упоминались броненосец "Потёмкин" (тоже причастный к
революционным событиям корабль) и крейсер "Паллада" (систершип "Авроры",
также участник Русско-японской войны).

Источник:
http://www.skyscrapercity.com/showthread.php?t=587108&page=215

Автор:
Александр Маничев (Санкт-Петербург)

Вопрос 18:
Внимание, в вопросе есть замена.
   Говоря о разделе Афганистана на сферы русского и английского влияния,
герои детектива "Туркестан" писателя Николая Свечина упоминают ЕЕ.
Назовите год, в котором на политической карте мира появилась более
известная ОНА.

Ответ:
1945.

Комментарий:
ОНА - 38-я параллель. Корея была разделена на советскую и американскую
зоны влияния по 38-й параллели.

Источник:
   1. Н. Свечин. Туркестан.
   2. http://ru.wikipedia.org/wiki/Разделение_Кореи

Автор:
Александр Маничев (Санкт-Петербург)

Вопрос 19:
Внимание, в вопросе есть замены.
   В современном обществе решение многих проблем часто ложится на плечи
женщин, а мужчины при этом остаются в стороне. В связи с этим автор
вопроса упоминает ПЕРВУЮ АЛЬФУ и ВТОРОЕ РО. Назовите двумя словами,
начинающимися на соседние буквы алфавита, того, в обращении к которому
упоминается ВТОРАЯ АЛЬФА и ПЕРВОЕ РО.

Ответ:
Младший лейтенант.

Комментарий:
В песне Ирины Аллегровой "Младший лейтенант" есть строки "Если бы ты
знал женскую тоску по сильному плечу". Применительно к первой части
вопроса имеем сильную тоску по женскому плечу.

Источник:
Песня Ирины Аллегровой "Младший лейтенант".

Автор:
Александр Маничев (Санкт-Петербург)

Вопрос 20:
К примеру, в дни защиты дипломов в толпе можно увидеть ЛАКУНЫ. ЛАКУНУ
упоминает Валерий Меладзе в одной из своих песен. Какие четыре слова мы
заменили на ЛАКУНУ?

Ответ:
Юных лет хмельная дева.

Комментарий:
Кстати, лакуна - пропуск женского рода.

Источник:
   1. Песня Валерия Меладзе "Не тревожь мне душу, скрипка".
   2. Наблюдения автора вопроса.

Автор:
Александр Маничев (Санкт-Петербург)

Вопрос 21:
Если бы некая финская фирма была отечественной, то неизбежно из-за
анаграммы возник бы вопрос: почему она производит не ПЕРВОЕ, а ВТОРОЕ?
Какое слово мы пропустили в предыдущем предложении?

Ответ:
Масло.

Комментарий:
Анаграмма слова "Валио" - "Олива". Логично, что фирма "Олива" должна
выпускать оливковое, а не сливочное масло.

Источник:
http://ru.wikipedia.org/wiki/Valio

Автор:
Александр Маничев (Санкт-Петербург)

Вопрос 22:
Внимание, вопрос задает Иван Васильевич!
   Многие населенные пункты были переименованы в честь своих знаменитых
уроженцев или уроженцев близлежащих мест. Так, Тверь получила имя
Калинин, Гжатск - Гагарин. Как могла быть переименована Вязьма, если бы
карьера одного из уроженцев Вяземского уезда сложилась более удачно?

Ответ:
Годунов.

Комментарий:
Будущий царь Борис Годунов родился на территории Вяземского уезда, в
молодости проживал в Вязьме в доме дяди.

Источник:
http://ru.wikipedia.org/wiki/Борис_Годунов

Автор:
Александр Маничев (Санкт-Петербург)

Вопрос 23:
[Ведущему: кавычки не озвучивать!]
   С 1938 по 1957 год оренбургский пуховый платок формально был ТАКИМ.
"ТАКИЕ" станции метро открылись в Москве и Санкт-Петербурге с разницей в
два года. Какое слово мы заменили на "ТАКОЙ"?

Ответ:
Чкаловский.

Комментарий:
С 1938 по 1957 год Оренбург носил имя Чкалов. Станция метро "Чкаловская"
Люблинско-Дмитровской линии Московского метро открылась в 1995 году,
станция метро "Чкаловская" Правобережной линии Санкт-Петербургского
метро - в 1997 году.

Источник:
   1. http://ru.wikipedia.org/wiki/Оренбургский_пуховый_платок
   2. http://ru.wikipedia.org/wiki/Оренбург
   3. http://ru.wikipedia.org/wiki/Чкаловская_(станция_метро,_Москва)
   4. http://ru.wikipedia.org/wiki/Чкаловская_(станция_метро,_Санкт-Петербург)

Автор:
Александр Маничев (Санкт-Петербург)

Вопрос 24:
Википедия сообщает, что 7 сентября 1501 года войска Ливонского ордена
разорили Крым. Какое слово мы заменили в предыдущем предложении?

Ответ:
Остров.

Комментарий:
Мы заменили одно слово из названия романа Василия Аксёнова другим.

Источник:
   1. http://ru.wikipedia.org/wiki/Остров_(город)
   2. http://ru.wikipedia.org/wiki/Остров_Крым

Автор:
Александр Маничев (Санкт-Петербург)

Тур:
7 тур. "Сэр Пухов" (Серпухов)

Редактор:
Максим Евланов

Вопрос 1:
Внимание, в вопросе есть замена.
   Впервые ААРОНОВОЕ БЛАГОСЛОВЕНИЕ появилось в 1967 году. С того времени
ААРОНОВОЕ БЛАГОСЛОВЕНИЕ прочно вошло в мировую культуру - например,
активно используется в сериале "Теория Большого взрыва" и даже
воплотилось в виде эмодзи. Итальянский космонавт Саманта Кристофоретти
воспроизвела ЕГО на борту МКС. Какие два слова мы заменили словами
"ААРОНОВОЕ БЛАГОСЛОВЕНИЕ"?

Ответ:
Вулканский салют.

Комментарий:
Приветствуем всех участников марафона!

Источник:
http://ru.wikipedia.org/wiki/Вулканский_салют

Автор:
Павел Семёнов

Вопрос 2:
В ходе перфоманса в парижской галерее художнику Свену Захзальберу
понадобилось на поиски целых два дня. С объектом поисков художника
писатель Николай Бирюков сравнивал в своем романе партизан. Назовите
объект поиска четырьмя словами.

Ответ:
Иголка в стоге сена.

Источник:
   1. http://lenta.ru/news/2014/11/16/needle/
   2. http://phraseology.academic.ru/4798/

Автор:
???

Вопрос 3:
Известно, что "Южный парк" начинается с АЛЬФЫ. В списке самых креативных
АЛЬФ упоминаются АЛЬФЫ, которые можно использовать как качели, весы,
футбольные ворота, а также АЛЬФА в виде слова из трех букв. Напишите это
слово.

Ответ:
Bus.

Комментарий:
АЛЬФА - автобусная остановка.

Источник:
   1. http://www.youtube.com/watch?v=S8p22rtNMoM
   2. http://www.bugaga.ru/interesting/1146723383-17-originalnyh-sposobov-ukrasit-avtobusnye-ostanovki.html
   3. http://www.mirkrasiv.ru/articles/avtobusnaja-ostanovka-v-vide-ogromnyh-bukv-slova-bus-baltimor-ssha.html

Автор:
Олег Холодов

Вопрос 4:
Не сразу, но всё же стало известно, что этот хорошо известный в России
отрицательный тип приложил свою "руку" к созданию олбанского языка, а
также стал одним из пионеров селфи. Назовите этого типа.

Ответ:
Волк [из "Ну, погоди!"].

Комментарий:
В четвертой серии мультфильма "Ну, погоди!" Волк пишет на экране
стадиона "ПрЮвет Волку!"; в той же серии волк фотографирует себя.

Источник:
http://www.youtube.com/watch?v=sHjVIJ5FfOs

Автор:
Святослав Холодов, Виктория Холодова

Вопрос 5:
Вам, скорее всего, знаком "клеточный" вариант ЕГО. Другой вариант ЕГО
выпускался на Серпуховском радиотехническом заводе с 1974 года.
Любопытно, что специальная версия этого варианта устанавливалась на
советских подводных лодках. Назовите ЕГО двумя словами.

Ответ:
"Морской бой".

Комментарий:
Помимо традиционного варианта игры "Морской бой" на листиках бумаги "в
клеточку", существовал и специальный электромеханический автомат
"Морской бой".

Источник:
   1. http://ru.wikipedia.org/wiki/Морской_бой_(игра)
   2. http://ru.wikipedia.org/wiki/Морской_бой_(игровой_автомат)

Автор:
Святослав Холодов

Вопрос 6:
ОН является прекрасным защитником от змей и с удовольствием охотится на
молодых кобр. В средневековых "Бестиариях" ОН уподобляется одной из
жертв Гермеса. На логотипе телекомпании NBC ОН появился в 1950-х годах,
чтобы продемонстрировать богатство новых технических возможностей.
Назовите ЕГО.

Ответ:
Павлин.

Комментарий:
В средневековых бестиариях он, как и Аргус, именуется "стооким".

Источник:
   1. http://ianimal.ru/topics/pavliny
   2. http://www.symbolarium.ru/index.php/Павлин
   3. http://www.blogotv.ru/?p=1093

Автор:
Святослав Холодов

Вопрос 7:
При желании скрытый смысл можно придать чему угодно. В рамках программы
года России - Франции 2010 организация "Amnesty International" запустила
так называемую социальную рекламу, посвященную России. Слоганом этой
рекламы стала фраза, которую можно перевести словами: "Мы не должны
позволить обаянию России скрыть ее злодеяния". Назовите снимавшуюся в
этой рекламе.

Ответ:
Русская матрешка.

Комментарий:
Традиционный российский символ на этот раз представлен в весьма
необычном виде. Руки кукольника собирают, вкладывая одна в одну,
мертвых, окровавленных, плачущих кукол с заклеенными ртами и в потёках
крови. И лишь последняя, традиционно нарядная, красивая и улыбающаяся,
выставлена на всеобщее обозрение. Уже тогда готовилось общественное
мнение...

Источник:
   1. http://www.audi-club.ru/forum/showthread.php?t=227412
   2. http://www.youtube.com/watch?v=bgSUJLRGNnA

Автор:
Олег Холодов

Вопрос 8:
Закончите стихотворение-порошок одним глаголом:
   их спросишь что такое совесть
   и тут же опускают взгляд
   не обольщайтесь им не стыдно
   ...

Ответ:
гуглят

Источник:
https://vk.com/perawki?w=wall-28122932_25366

Автор:
Татьяна Соломенникова

Вопрос 9:
[Ведущему: отточие не озвучивать.]
   Внимание, укороченная цитата из статьи сайта "Спорт-Экспресс": "За
последние годы Денисова так часто <...> куда-нибудь ссылали, что
складывалось впечатление: нынешняя ссылка вот-вот превратится в НЕЕ
длиною в вечность". Известно, что ОНА имеет два якоря и иногда бывает
битой. Назовите ЕЕ.

Ответ:
Гиперссылка.

Источник:
   1. http://www.sport-express.ru/football/rfpl/reviews/955422/
   2. http://www.programming-lang.com/html/html/html401/links.html

Автор:
Святослав Холодов

Вопрос 10:
Блиц.
   В Интернете опубликовали придуманные современные вариации обложек для
произведений русских классиков.
   1. Воспроизведите состоящее из одного слова название произведения, на
обложке которого напечатаны, в частности, следующие фразы: "Превышен
интервал ожидания для запроса. Ответ от 192.168.2.1: Заданная сеть
недоступна".
   2. Воспроизведите состоящее из одного слова название произведения, на
обложке которого изображен Элайджа Вуд в роли Фродо Бэггинса.
   3. Воспроизведите состоящее из двух слов название произведения, на
обложке которого изображена верхняя часть женской фигуры в красном
купальнике.

Ответ:
   1. "Обрыв".
   2. "Недоросль".
   3. "Алые паруса".

Источник:
(pic: 20160306.jpg)

Автор:
Виктория Холодова

Вопрос 11:
По некоторым данным, "ИКСАМИ" прозвали тех альпинистов, которые
предпочитали первым делом позировать фотографам, упирая одну руку в бок,
а другую отставляя в сторону, опираясь на ледоруб, лыжную палку и тому
подобное. Какое слово мы заменили на ИКС?

Ответ:
Чайник.

Комментарий:
Их силуэт сильно напоминает чайник.

Источник:
http://new.gramota.ru/spravka/buro/search-answer?s=254453

Автор:
Святослав Холодов

Вопрос 12:
В одной недавней статье говорится, что, не имея других аргументов,
бывший министр иностранных дел Польши Радослав Сикорский сравнил в свое
время ИКС с пактом Молотова - Риббентропа. А в конце этой статьи
приводится ссылка на материал, заголовок которого завершается словами:
"... Польша снова бежит по лезвию грабель", Какие два слова мы заменили
на ИКС?

Ответ:
"Северный поток".

Источник:
http://regnum.ru/news/international/2042870.html

Автор:
Олег Холодов

Вопрос 13:
В интервью с игроком московского "Локомотива" Дмитрием Тарасовым
журналист задает по поводу травмы футболиста вопрос: "Первая мысль при
столкновении: "Всё, ОНИ?"". Если верить Википедии, ОНИ - это разговорное
название языка программирования C++ [си плюс плюс]. Назовите ИХ одним
словом.

Ответ:
Кресты.

Комментарий:
"Кресты" - жаргонное название передней и задней крестообразных связок
коленного сустава.

Источник:
http://football.sport-express.ru/reviews/42242/

Автор:
Олег Холодов

Вопрос 14:
На карикатуре, посвященной злободневной теме, изображены: мальчик,
кошачье, медведь, непарнокопытное и зайцеобразное. Они стоят на берегу,
смотря на плот... Ответьте максимально точно: с кем?

Ответ:
С Пятачком.

Комментарий:
Посвящается свиному гриппу.

Источник:
http://pikabu.ru/story/svinoy_gripp___11057

Автор:
Михаил Березин

Вопрос 15:
По одной из версий, Пушкин использовал ИКС для вскрытия писем, а
британский живописец Уильям Тёрнер - для удаления ненужной краски.
Некрасов никогда не садился играть в карты с людьми, у которых был ИКС.
Назовите ИКС двумя словами.

Ответ:
Длинный ноготь.

Зачет:
Масонский ноготь.

Комментарий:
Длинный ноготь на мизинце использовался шулерами как закладка для
разделения карт при съеме.

Источник:
   1. http://interesource.ru/raznoe/zachem-otrashhivayut-dlinnye-nogti-na-mizincax/
   2. http://impressionnisme.narod.ru/TURNER/epizode_turner2.htm
   3. http://i-fakt.ru/interesnye-fakty-o-nekrasove/

Автор:
Олег Холодов

Вопрос 16:
Эта американская песня рассказывала о тяжелом труде шахтеров, жалующихся
на высокие нормы добычи угля. А после Карибского кризиса в СССР
появилась интерпретация этой песни, которая начиналась со слов: "Сидим
мы в баре в поздний час". Впрочем, от оригинала в этой интерпретации
осталась и строчка: "А в каждой бомбе...". Закончите эту строчку двумя
словами.

Ответ:
"... шестнадцать тонн".

Комментарий:
"Шестнадцать тонн - умри, но дай. / Всю жизнь работай - век страдай" -
гласит один из переводов оригинала; интерпретация времен Карибского
кризиса содержала и куплет "Летит по небу мой "фантом", / А в каждой
бомбе - шестнадцать тонн. / Шестнадцать тонн - опасный груз, / А мы
летим бомбить Союз!".

Источник:
   1. http://belaveda.moy.su/blog/shestnadcat_tonn/2012-10-08-1012
   2. http://ru.wikipedia.org/wiki/Sixteen_Tons

Автор:
Ника Холодова

Вопрос 17:
Французские журналисты вручили награду "ИКС" Евгению Кафельникову за его
резкие высказывания и необщительность. Статью, посвященную борьбе
российских вузов за правительственные гранты в "Московском комсомольце"
назвали "ИКС раздора". Какое слово мы заменили на ИКС?

Ответ:
Лимон.

Источник:
   1. http://lenta.ru/sport/2000/05/31/lemon/
   2. http://www.mk.ru/editions/daily/article/2006/03/27/184585-limon-razdora.html

Автор:
Виктория Холодова

Вопрос 18:
В СССР обязательное употребление ЕЕ было введено 24 декабря 1942 года
приказом N 1825 наркома Потёмкина. В современном стихотворении-пирожке
корректор из креативного агентства украшает ЕЕ запятыми. Напишите ЕЕ.

Ответ:
Ё.

Источник:
   1. http://ru.wikipedia.org/wiki/Ё
   2. http://poetory.ru/content/view?id=64032

Автор:
Павел Семёнов

Вопрос 19:
Это слово пришло к нам из уголовного жаргона. В современном Серпухове
этим словом стали называть охранников городской администрации, после
того как по решению нового мэра были установлены некие устройства.
Назовите эти устройства одним словом.

Ответ:
Вертушки.

Источник:
ЛОАВ.

Автор:
Павел Семёнов

Вопрос 20:
В игре "Warcraft III" [варкрАфт три] для разведки территории Жрица Луны
может использовать АЛЬФУ. Поговорка "везти АЛЬФУ в Афины" означает
примерно то же, что и "ехать в Тулу со своим самоваром". Какое слово мы
заменили на АЛЬФУ?

Ответ:
Сова.

Комментарий:
Сова - священный символ богини Афины.

Источник:
   1. Игра "Warcraft III. The Frozen Throne".
   2. http://old.kpfu.ru/fil/kn2/index.php?sod=11

Автор:
Олег Холодов

Вопрос 21:
Одними из первых Новый год в России встречает команда из пяти человек,
обслуживающая маяк на косе под названием "ОНА". Селекция ЕЕ голубой
началась в 1893 году. Назовите ЕЕ двумя словами.

Ответ:
Русская Кошка.

Источник:
   1. http://ru.wikipedia.org/wiki/Русская_Кошка
   2. http://ru.wikipedia.org/wiki/Русская_голубая_кошка

Автор:
???

Вопрос 22:
Вслед за программой "Ревизорро" на канале "Пятница" вышел проект
"Магаззино". Владельцы магазинов, которые не прошли тяжелое испытание
новой программой, получают... Что именно?

Ответ:
Молоко.

Зачет:
Молоко за вредность.

Комментарий:
Ведущий программы - Александр Молочко.

Источник:
http://magazzino.friday.ru/

Автор:
Татьяна Соломенникова

Вопрос 23:
Внимание, в вопросе есть замены.
   Бержерак - очень одиозная личность. В одном европейском языке даже
появился глагол "siranera", означающий что-то вроде "доминировать,
делать что-то с силой". По мнению журналистов, на Бержерака, подобно Ди
Каприо, наложено проклятие. Впрочем, по тому же мнению, Бержерак не
бежит от проклятия, меняя одно место работы на другое, а просто
воспроизводит профиль своего носа. Какую фамилию мы заменили на
"Бержерак"?

Ответ:
Ибрагимович.

Комментарий:
Как Ди Каприо не может выиграть "Оскар", так Ибрагимович не может
выиграть "Золотой мяч" и Лигу чемпионов. Глагол "zlatanera" появился в
шведском языке.

Источник:
http://www.youtube.com/watch?v=mCPg-uqDzBE

Автор:
Святослав Холодов

Вопрос 24:
Изначально ЭТО СЛОВО появилось в русском языке при Екатерине II как
наименование трефового туза в карточной игре ломбер. С вас хватит!
Назовите ЭТО СЛОВО.

Ответ:
Баста.

Комментарий:
Баста - на этом наш тур закончен!

Источник:
http://dic.academic.ru/dic.nsf/enc2p/205954/

Автор:
Олег Холодов

Тур:
8 тур. "Львята отца Симбы" ("Лоси") (Северодвинск)

Редактор:
Валерия Комаровская (Коломна)

Вопрос 1:
(pic: 20160307.jpg)
   Мы НАЧИНАЕМ наш тур с раздаточного материала.
   Интернет-поиск в Яндекс-картинках по определенному слову часто выдает
среди прочих изображение игрушки, изредка - кнопки. Еще шесть лет назад
изображение игрушки в ответ на такой запрос не появилось бы. Перед вами
еще один результат поиска по этому слову - картинка, которая вполне
могла бы стать иллюстрацией к песне Гарика Сукачёва или группы
"Мураками". Мы не спрашиваем, по какому слову осуществлялся поиск. Какие
три символа мы скрыли на картинке?

Ответ:
0 км.

Зачет:
0 km.

Комментарий:
(pic: 20160308.jpg)
   По поиску на слово "начало" часто выдается изображение кнопки
(красной, со словом "Start" и т.п.), и очень часто - волчка из фильма
"Начало". Премьера фильма состоялась в Лондоне 13 июля 2010 года, т.е.
более шести лет назад. На картинке скрыто "0 км" - так называемый
"нулевой километр", начало отсчета расстояния. "Нулевой километр" -
название песни Гарика Сукачёва и песни группы "Мураками".

Источник:
   1. https://yandex.ru/images/search?text=начало
   2. http://ru.wikipedia.org/wiki/Начало_(фильм,_2010)
   3. http://www.gl5.ru/s/sukachev-garik/sukachev-garik-nulevoj-kilometr.html
   4. http://www.gl5.ru/m/murakami/murakami-nulevoj-kilometr.html

Автор:
Елена Ганькова (Северодвинск)

Вопрос 2:
В вопросе словом "ОНА" заменены другие слова.
   Через некоторое время после выхода альбома "Вендетта" певица Земфира
выпустила песню "Выкинуть тебя из головы", в которой лирической героине
обещана ОНА. ОНА стала называнием книги американского писателя Роберта
Лоуренса Стайна, изданной в серии "Ужастики". Но вам, вероятно, более
известна другая "ОНА", опубликованная в 1831 году. Назовите ЕЕ двумя
словами.

Ответ:
Страшная месть.

Комментарий:
Строки Земфиры: "Ты обещаешь мне страшную месть, // А я оставляю свой
паспорт тебе". Альбом "Вендетта" упомянут не случайно, т.к. вендетта -
это тоже месть. В 1831 году опубликована повесть Н.В. Гоголя "Страшная
месть", строки из которой "Чуден Днепр при тихой погоде..." и "Редкая
птица долетит до середины Днепра" известны нам со школьных лет.

Источник:
   1. https://music.yandex.ru/album/2032899/track/18307527
   2. http://ru.wikipedia.org/wiki/Z-Sides
   3. http://ru.wikipedia.org/wiki/Вендетта_(альбом)
   4. http://ru.wikipedia.org/wiki/Ужастики_(список_книг)
   5. http://ru.wikipedia.org/wiki/Страшная_месть

Автор:
Елена Ганькова (Северодвинск)

Вопрос 3:
Внимание, в вопросе есть замена.
   Один ЖЖ-пользователь вспоминает дефицит 80-х: "... Стоит длинная
очередь... Подходят две тетки, одна становится в очередь, а другая идет
посмотреть, что дают. Возвращается: "Там такие МОЗАИКИ замечательные,
красные, синие, зеленые... Какие брать будем?"". Ответьте абсолютно
точно, за чем стояла очередь.

Ответ:
За кубиками Рубика.

Источник:
http://dubikvit.livejournal.com/331305.html?thread=19093545#t19093545

Автор:
Дмитрий Гевель (Северодвинск)

Вопрос 4:
Мультсериал "Маша и Медведь" содержит множество аллюзий к различным
событиям и произведениям. К примеру, в серии "Ловись, рыбка!" Медведь
жонглирует по просьбе Маши и представляет себе рыбку, как и лев в
мультфильме "Каникулы Бонифация". Ситуация, когда Маша ловит золотую
рыбку и загадывает желания, напрямую отсылает к "Сказке о рыбаке и
рыбке" А.С. Пушкина. А в финале серии уставший и почти отчаявшийся
Медведь наконец-то успешно вытаскивает из воды целую и невредимую рыбу.
А какую именно?

Ответ:
МарлИна.

Зачет:
Копьеносца. Незачет: Меч-рыбу.

Комментарий:
Рыба, которую в конце ловит Медведь, - это марлИн - та самая, которую
поймал Старик из повести Э. Хемингуэя "Старик и море". "Вся она горела
на солнце, голова и спина у нее была темно-фиолетовая, а полосы на боках
казались при ярком свете очень широкими и нежно-сиреневыми. Вместо носа
у нее был меч, длинный, как бейсбольная клюшка, и острый на конце, как
рапира". Меч-рыба - это рыба другого семейства, которую часто путают с
марлином, ее старик не ловил, и Медведь тоже не ловил, соответственно.

Источник:
   1. http://ru.wikipedia.org/wiki/Маша_и_Медведь
   2. http://www.youtube.com/watch?v=xMGgU4zxDbo
   3. http://www.liveinternet.ru/users/3701366/post235033777
   4. http://ru.wikipedia.org/wiki/Марлиновые
   5. http://ru.wikipedia.org/wiki/Меч-рыба

Автор:
Елена Ганькова (Северодвинск)

Вопрос 5:
(pic: 20160309.jpg)
   29 ноября 2015 года одно из сообществ ВКонтакте поздравило этой
картинкой своих подписчиков с праздником. Ответьте как можно точнее: с
каким?

Ответ:
С днем рождения буквы Ё.

Комментарий:
29 ноября (18 ноября по старому стилю) 1783 года в доме директора
Петербургской Академии наук княгини Екатерины Романовны Дашковой
состоялось одно из первых заседаний недавно созданной Российской
академии, на котором присутствовали Г.Р. Державин, Д.И. Фонвизин, И.И.
Лепёхин, Я.Б. Княжнин, митрополит Гавриил и др. Дашкова предложила
использовать новую букву "ё" "для выражения слов и выговоров, с сего
согласия начинающихся, как матiорый, iолка, iож, iол". Доводы Дашковой
показались убедительными. 18 ноября 1784 года буква "ё" получила
официальное признание.

Источник:
   1. http://vk.com/guruandroid?w=wall-47710541_154059
   2. http://www.opoccuu.com/291111.htm

Автор:
Елена Ганькова, Анна Дуплищева (Северодвинск)

Вопрос 6:
В одном из номеров журнала "Brain" за 1956 год была опубликована статья
о человеке с визуальной агнозией: он не узнавал себя и других людей и
вынужден был отыскивать одну-две заметные черты и на их основании
строить догадки. Не узнавая жену, больной просил, чтобы она помогала
ему, используя в одежде "какую-нибудь заметную деталь" - например,
большой головной убор. А какой автор приводит эту историю в своем
произведении 1971 года?

Ответ:
[Оливер] Сакс.

Комментарий:
У пациента, о котором нейропсихолог и невролог Оливер Сакс рассказывает
в книге "Человек, который принял жену за шляпу", были сходные симптомы.
Их роднит и то, что пациент - герой статьи - просил жену надевать
большую шляпу.

Источник:
О. Сакс. Человек, который принял жену за шляпу.
http://www.flibusta.is/b/95975/read

Автор:
Елена Ганькова (Северодвинск)

Вопрос 7:
С этим процессом связано множество заблуждений. К примеру, он был
распространен среди древних скифов Евразии и во время войн между
вестготами, франками и англосаксами в IX веке. Да и позднее европейцы им
не брезговали. На его основе развился выгодный бизнес: так, в 1703 году
в Пенсильвании можно было получить 50 долларов или 124 доллара... Мы не
спрашиваем, за что именно. Ответьте, в зависимости от чего варьировалась
эта цена?

Ответ:
Женщине или мужчине принадлежит скальп.

Зачет:
По смыслу с упоминанием скальпа или скальпирования.

Комментарий:
Снятие скальпа - не прерогатива индейских племен, оно было
распространено и в Европе. Причем именно европейцы-колонизаторы основали
бизнес на скальпах индейцев: в 1703 году в Пенсильвании можно было
получить 50 долларов за скальп женщины индейского племени или 124
доллара - за скальп мужчины.

Источник:
http://ru.wikipedia.org/wiki/Скальпирование

Автор:
Елена Ганькова (Северодвинск)

Вопрос 8:
Внимание, в вопросе есть замена.
   В фильме "Уильям Тёрнер" английский художник Уильям Тёрнер в порыве
вдохновения смотрел свысока на свои картины и подправлял мелкие детали,
добавляя штрихи и мазки, чтобы довести полотно до ума. В предыдущем
предложении мы заменили одно слово на два других. Восстановите
замененное слово.

Ответ:
Плевал.

Комментарий:
Тёрнер в прямом смысле плевал на картины, разводя краски слюной.
"Смотреть свысока" - один из синонимов слова "плевать" в переносном
смысле.

Источник:
Х/ф "Уильям Тёрнер" (2014), реж. Майк Ли.

Автор:
Алексей Нехлебаев (Северодвинск)

Вопрос 9:
(pic: 20160310.jpg)
   На розданном вам логотипе одной компании мы изменили пару букв.
Восстановите оригинальную надпись.

Ответ:
"Kinguin".

Комментарий:
(pic: 20160311.jpg)
   Основанная в 2013 году компания, платформа для продажи игр, которая
активно спонсирует киберспорт.

Источник:
https://www.kinguin.net/

Автор:
Алексей Нехлебаев (Северодвинск)

Вопрос 10:
В вопросе есть замена.
   Оливер Сакс в книге "Человек, который принял жену за шляпу"
рассказывает о женщине, которая утратила идею "левой стороны", т.е. не
видела ничего, что находилось слева, - например, еду на левой стороне
тарелки. Она выработала стратегии, позволяющие обойти дефект: так, если
ей казалось, что на тарелке не хватает еды, она начинала вертеться на
кресле-каталке вправо. Доехав по кругу до недостающей половины, она
съедала половину еды, и если всё еще была голодна, то делала еще один
оборот, и т.д. Таким образом, пациентка Сакса сравнила себя с бочкой
Диогена. Какие слова мы заменили в этом вопросе?

Ответ:
Стрела Зенона.

Комментарий:
"Абсурд, - говорит она. - Я как стрела Зенона - никогда не долетаю до
цели. Выглядит это, наверно, как в цирке, но куда же денешься?".

Источник:
О. Сакс. Человек, который принял жену за шляпу.
http://www.flibusta.is/b/95975/read

Автор:
Елена Ганькова (Северодвинск)

Вопрос 11:
Герой стихотворения из книги Владимира Шинкарёва "Митьки" оказался в
ситуации, аналогичной той, в которую однажды, по легенде, попал Платон.
Цитата:
   Японский друг принес кувшин сакэ.
   Максиму с Федором с учтивою улыбкой
   для закуси велел сакуры принести.
   А те, японским языком владея не изрядно,
   ему несут не сакуру, но [ПРОПУСК].
   Что же принесли Максим и Федор?

Ответ:
Куру.

Зачет:
Курицу.

Комментарий:
Существует легенда, согласно которой Диоген Синопский, в ответ на слова
Платона "Человек есть животное о двух ногах, лишенное перьев", общипал
петуха и принес его философу, объявив: "Вот платоновский человек!".

Источник:
   1. http://www.marussia.ru/shinkarev.html
   2. http://ru.wikipedia.org/wiki/Диоген

Автор:
Алексей Нехлебаев (Северодвинск)

Вопрос 12:
В вопросе словом "ИКС" заменены другие слова.
   Знакомый автора вопроса в ходе своей профессиональной деятельности по
итогам одной проверки провел исследование и выяснил, что лидирующие
места занимают Стамбул, Нью-Йорк, Сидней, Кейптаун, которые на самом
деле ИКСАМИ не являются. Назовите ИКСЫ тремя словами, начинающимися на
одну и ту же букву.

Ответ:
Столицы своих стран.

Комментарий:
На самом деле они не являются столицами своих стран, которыми их считают
распространенные заблуждения и люди, не знающие географию.

Источник:
Тест на уроке географии в средней школе.

Автор:
Анна Дуплищева (Северодвинск)

Вопрос 13:
(pic: 20160312.jpg)
   Внимание, в вопросе есть замены.
   Автор вопроса, увидев это изображение, воскликнул: "Какой классный
ИКС!". В ответ ему заметили: "Это же не ИКС, а АЛЬФА!". Назовите ИКС и
АЛЬФУ, которые лишь немного отличаются друг от друга.

Ответ:
Косплей, коспЛея.

Зачет:
Косплей, Косплея.

Комментарий:
Косплей - переодевание в костюмы известных персонажей. В данном случае
это принцесса Лея из "Звездных войн". :-)

Источник:
   1. ЛОАВ.
   2. http://ru.wikipedia.org/wiki/Косплей

Автор:
Анна Дуплищева (Северодвинск)

Вопрос 14:
Внимание, в вопросе есть замена.
   После того как состав группы "Queen" стал постоянным, Фредди Меркьюри
решил нарисовать ее герб. По одной из версий, за основу был взят герб
Великобритании, с латинской буквой Q, вокруг которой "вплетены" два
льва, краб, вылезающий из огня, и две феи с крыльями. Как следует из
герба, Джон Дикон, Роджер Тэйлор и Брайан Мэй СДЕЛАЛИ ЭТО. Можно
подумать, что и Меркьюри СДЕЛАЛ ЭТО, но на самом деле он, так сказать,
немного "опоздал". Оба автора вопроса также СДЕЛАЛИ ЭТО. Какие два слова
мы заменили словосочетанием "СДЕЛАТЬ ЭТО"?

Ответ:
Родиться летом.

Комментарий:
В герб заключены зодиакальные знаки членов "Queen": два Льва - Джон
Дикон и Роджер Тэйлор, краб, вылезающий из огня, - знак Рака - Брайан
Мэй. Две феи с крыльями - зодиакальный знак Фредди - Дева. Можно
подумать, что Фредди родился летом, но он чуть "опоздал" и родился в
осенние дни знака - 5 сентября. Авторы вопроса родились летом - 24
августа и 21 июля.

Источник:
   1. http://ru.wikipedia.org/wiki/Queen#.D0.9B.D0.BE.D0.B3.D0.BE.D1.82.D0.B8.D0.BF
   2. http://ru.wikipedia.org/wiki/Фредди_Меркьюри
   3. ЛОАВ.

Автор:
Елена Ганькова, Анна Дуплищева (Северодвинск)

Вопрос 15:
По его собственным словам, Питер де Врайз стал ИМ, потому что терпеть не
мог бумажной работы. Писатель Тристан Бернар признался, что стал ИМ,
потому что ему не с кем было поговорить. Айрис Мёрдок сравнила ИХ
занятие с женитьбой, ибо ни в коем случае не стоит делать решительного
шага, пока сам не изумишься своему счастью. Назовите это занятие.

Ответ:
Писательство.

Зачет:
По смыслу.

Источник:
http://citaty.su/aforizmy-i-citaty-o-pisatelyax

Автор:
Елена Ганькова (Северодвинск)

Вопрос 16:
Дуплет.
   1. В вопросе есть замена.
   Интернет-шутка: "Сегодня целый час чистил чайник. Достало". Какое
слово мы заменили в этом вопросе?
   2. Закончите интернет-шутку тремя словами: "Все знают барона
Мюнхгаузена. Но мало кто знает, как его звали. А ведь его звали...".

Ответ:
   1. Накипело
   2. "... Карл. Карл, Карл!".

Зачет:
   2. С любыми знаками препинания.

Комментарий:
   2. Известный интернет-мем про Карла.

Источник:
   1. http://pikabu.ru/story/nakipelo_3784109
   2. http://www.yaplakal.com/forum2/topic1099639.html

Автор:
   1. Анна Дуплищева (Северодвинск)
   2. Елена Ганькова (Северодвинск)

Вопрос 17:
(pic: 20160313.jpg)
   Перед вами портрет известного уроженца Парижа. Назовите это
изображение так, как сделал это автор вопроса, - изменив лишь
капитализацию в названии произведения 1903 года.

Ответ:
"Вишневый Сад".

Комментарий:
На портрете Донасьен Альфонс Франсуа, граф де Сад.

Источник:
   1. http://ru.wikipedia.org/wiki/Маркиз_де_Сад
   2. http://ru.wikipedia.org/wiki/Вишнёвый_сад

Автор:
Елена Ганькова (Северодвинск)

Вопрос 18:
Внимание, в вопросе слова "ПЕРВЫЙ" и "ВТОРОЙ" - замены.
   Факт общения ПЕРВОГО и ВТОРОГО в эпистолярном жанре достаточно
известен благодаря источнику первой трети XX века. Причем
интернет-пользователи часто вместо ПЕРВОГО ошибочно упоминают русского
или еврея, а также считают, что ОНИ разговаривали о бабах, ругались или
играли в шахматы по переписке. В шахматы по переписке, Карл! Назовите
ПЕРВОГО и ВТОРОГО в любом порядке.

Ответ:
[Фридрих] Энгельс и [Карл] Каутский.

Зачет:
В любом порядке.

Комментарий:
В 1925 году была написана повесть М.А. Булгакова "Собачье сердце".
Некоторые интернет-пользователи уверены, что Шариков читал переписку
Ленина или Троцкого с Каутским, а не Фридриха Энгельса, как было на
самом деле. Кстати, немца Каутского звали Карлом (не путать с Марксом).

Источник:
   1. М.А. Булгаков. Собачье сердце.
http://www.flibusta.is/b/327279/read
   2. http://ru.wikipedia.org/wiki/Каутский,_Карл
   3. https://otvet.mail.ru/question/174668418
   4. https://otvet.mail.ru/question/25261392

Автор:
Елена Ганькова (Северодвинск)

Вопрос 19:
Дуплет.
   1. Комедийно-детективный сериал "Ясновидец" полон аллюзий и пародий
на известные кинофильмы и телевизионные штампы. Так, главный герой
сериала получает e-mail с приглашением посетить затерянный в сосновых
лесах небольшой городок. По его приезду выясняется, что тело школьницы,
любимицы всех горожан Полы Меррал найдено на берегу реки. Детективу
приходит sms, содержащее четыре слова. Воспроизведите текст sms.
   2. Статья в "Московском комсомольце" от 8 сентября 2014 года
рассказывает, что компания "Оренбургские авиалинии" могла стать жертвой
недобросовестности своих партнеров по турбизнесу. По мнению экспертов,
банкротящиеся туроператоры попытались свалить свои проблемы на
авиакомпанию. Статья озаглавлена вопросительной фразой из четырех слов.
Воспроизведите это название.

Ответ:
   1. Кто убил Полу Меррал?
   2. Кто подставил "Оренбургские авиалинии"?

Комментарий:
   1. Аллюзия на сериал "Твин Пикс"; Пола Меррал - анаграмма Лоры
Палмер.
   2. Аллюзия на другой известный киновопрос - "Кто подставил кролика
Роджера?".

Источник:
   1. Сериал "Ясновидец", s05e12.
   2. http://www.mk.ru/economics/2014/09/08/kto-podstavil-orenburgskie-avialinii.html

Автор:
Валерия Комаровская (Коломна)

Вопрос 20:
И еще один вопрос.
   Отвечая на определенный вопрос, специалисты упоминают условия
проживания, особенности рациона и строения тела. В игре "Сто к одному"
шестым по популярности предлагаемым ответом на этот вопрос является
"Холодно". А опрос, проведенный школьником-исследователем среди учеников
1-3 классов, дал и такие варианты ответа, как "Боятся" и "Не хотят".
Чтобы дать правильный ответ, воспроизведите этот вопрос.

Ответ:
Почему пингвины не летают?

Зачет:
Почему пингвины не могут летать? и т.п. по смыслу.

Комментарий:
Пингвинам пришлось приспосабливаться к жизни в условиях постоянного
холода и необходимости добывать себе пищу в воде. Основная еда пингвинов
- рыба. Чтобы успешно ее ловить, пингвинам необходимо было стать
хорошими пловцами. Так их крылья, слишком короткие, толстые и тяжелые
для того, чтобы летать, стали прекрасными плавниками.

Источник:
   1. https://vk.com/vkwhy?w=wall-45938293_49290
   2. http://www.bolshoyvopros.ru/questions/276630-100-k-1-pochemu-pingviny-ne-letajut.html
   3. http://nsportal.ru/ap/library/drugoe/2015/11/12/issledovatelskaya-rabota-pochemu-pingviny-ne-letayut

Автор:
Елена Ганькова (Северодвинск)

Вопрос 21:
(pic: 20160314.jpg)
   На картинке изображен ОН. Назовите ЕГО двумя словами и не
промахнитесь!

Ответ:
Десерт игл.

Комментарий:
Обыграно название пистолета "Desert Eagle". "Не промахнитесь" - намек. А
этот вопрос открывает последние четыре вопроса от нашей команды - так
сказать, десерт нашего пакета.

Источник:
(pic: 20160315.jpg)

Автор:
Дмитрий Гевель (Северодвинск)

Вопрос 22:
Дуплет от Дмитрия Глуховского.
   1. Во многих онлайн-играх войны между кланами проходят круглосуточно.
В одной из такой игр после объявления войны командир пишет в игровом
чате одно из слов. К примеру, "школьный предмет", или "детский
праздник", или "бытовой прибор". Назовите этот прибор.
   2. В клановых войнах онлайн-игр временами полезны не только победы,
но и поражения. Иногда про такие кланы говорят, что они сидят на
фруктовой диете. Что мы заменили словом "фруктовая"?

Ответ:
   1. Ночник.
   2. Сливовая.

Комментарий:
   1. Речь идет об онлайн-игре "Метро-2033" ВКонтакте. После объявления
войны командир объявляет, когда война: днем - "дневник", утром -
"утренник", ночью - "ночник".
   2. Проигрыш - слив.

Источник:
ЛОАВ - чат клана "Метролли" в игре "Метро-2033".

Автор:
Анна Дуплищева (Северодвинск)

Вопрос 23:
Во время Второй мировой войны немецкие саперы особо охотились за
английскими офицерами. Отступая, они минировали в домах криво повешенные
картины. Хорошо воспитанные джентльмены приводили в действие взрывной
механизм. Одна из интернет-статей, рассказывающих об этом, была
озаглавлена популярным в сети словосочетанием. Напишите это
словосочетание.

Ответ:
Ад перфекциониста.

Источник:
   1. https://vk.com/publicheretics?w=wall-99950980_36
   2. https://www.adme.ru/zhizn-marazmy/ad-perfekcionista-648305/

Автор:
Елена Ганькова, Анна Дуплищева (Северодвинск)

Вопрос 24:
Ну и, наконец, последний вопрос.
   Первый ОН в столичном метро появился на станции "Проспект мира" в
сентябре 2015 года. В ходе проекта предстояло определить, будет ли ОН
востребован среди пассажиров. РБК отмечает, что ОН был закрыт после 55
дней работы, но не потому, что не востребован, а из-за возникших
замечаний. "Мы выяснили, что 50 рублей - это та стоимость, которую люди
готовы платить", - отметил замглавы метрополитена Юрий Дегтярёв. В
данном вопросе мы неоднократно пропустили 16 букв. Напишите их.

Ответ:
б, щ, е, с, т, в, е, н, ы, й, т, у, а, л, е, т.

Источник:
http://www.rbc.ru/rbcfreenews/5656e6ad9a79473f98796fc7

Автор:
Елена Ганькова, Анна Дуплищева (Северодвинск)

Тур:
9 тур. "Я выгнал Арамиса" (Москва)

Редактор:
Серафим Шибанов (Москва)

Инфо:
Редактор благодарит за помощь в подготовке пакета Дмитрия Дягилева,
Максима Мерзлякова, Галину Пактовскую, Дмитрия Соболева и Александра
Телкова.

Вопрос 1:
Один пользователь Живого Журнала вспоминает, как задал
первокурснику-историку вопрос о Николае, а тот не смог вспомнить его
фамилию. Напишите эту фамилию.

Ответ:
Чаушеску.

Комментарий:
Дети стали забывать румынского диктатора. Вы, надеюсь, его не забыли.

Источник:
http://samir-befish.livejournal.com/1085.html

Автор:
Серафим Шибанов (Москва)

Вопрос 2:
   <раздатка>
   Какой суровый путь он прошел с патриотической самоотверженностью во
имя защиты социалистической Родины, недвусмысленно свидетельствует его
единственная вечная куртка, которую он носил до последних дней своей
жизни.
   </раздатка>
   Перед вами цитата о Ким Ир Сене, в одном из слов которой мы заменили
две буквы. Напишите это слово в исходном виде.

Ответ:
Ватная.

Комментарий:
Не вечная, а ватная. Недаром говорится о патриотической
самоотверженности.

Источник:
http://lurkmore.to/Ватник

Автор:
Серафим Шибанов (Москва)

Вопрос 3:
К лидеру Танзании Джулиусу НьерЕре население страны всегда относилось с
уважением, называя его за заслуги и образованность "мвАлиму". Слово
"мвАлиму" переводится на русский фамилией деятеля культуры. Напишите имя
этого деятеля культуры.

Ответ:
Алексей.

Комментарий:
Учитель.

Источник:
   1. http://ru.wikipedia.org/wiki/Ньерере,_Джулиус
   2. http://ru.wikipedia.org/wiki/Учитель,_Алексей_Ефимович

Автор:
Серафим Шибанов (Москва)

Вопрос 4:
Диктатор Габона АльбЕр БернАр БОнго в 1973 году стал раком - вероятно, в
честь халифа. Какое слово мы заменили в предыдущем предложении?

Ответ:
Омаром.

Комментарий:
БОнго принял ислам и взял себе имя Омар, которое носил второй Праведный
халиф.

Источник:
http://en.wikipedia.org/wiki/Omar_Bongo

Автор:
Серафим Шибанов (Москва)

Вопрос 5:
   <раздатка>
   Шарифходжаев, Титов, Гусаков, Екимов, Пикуль
   </раздатка>
   В один ряд с обладателями этих фамилий можно поставить советскую
певицу, родившуюся в 1932 году. Напишите ее фамилию.

Ответ:
Кристалинская.

Комментарий:
В фамилиях "Шарифходжаев", "Титов", "Гусаков", "Екимов", "Пикуль" можно
найти фамилии "ХОджа", "ТИто", "ГУсак", "Ким", "Пик", которые носили
лидеры стран соцлагеря. В фамилии "Кристалинская" можно найти "Сталин".

Источник:
http://ru.wikipedia.org/wiki/Кристалинская,_Майя_Владимировна

Автор:
Серафим Шибанов (Москва)

Вопрос 6:
В 1972 году президент Уганды Иди Амин своим указом изгнал из страны
проживавших там азиатов. ПЕРВЫЙ не остался в стороне и впоследствии
получил медаль ВТОРОГО, бывшего нобелевским лауреатом. Фамилии ПЕРВОГО и
ВТОРОГО начинаются на одну букву. Напишите обе фамилии.

Ответ:
Ньерере, Нансен.

Комментарий:
Изгнанные из Уганды азиаты (а именно индийцы) нашли пристанище в
соседней Танзании, где правил НьерЕре. За гуманное обращение с беженцами
лидер Танзании получил премию Нансена, также известного своими делами на
данной почве.

Источник:
   1. http://ru.wikipedia.org/wiki/Амин,_Иди
   2. http://en.wikipedia.org/wiki/Julius_Nyerere
   3. http://ru.wikipedia.org/wiki/Нансен,_Фритьоф

Автор:
Серафим Шибанов (Москва)

Вопрос 7:
Леонид Брежнев в начале 80-х пообещал подарить эфиопскому диктатору
МенгИсту ХАйле МариАму новый авиалайнер. Когда спустя некоторое время
лайнер прибыл в Эфиопию, МенгИсту не только не поблагодарил Москву, но
даже не поехал взглянуть на самолет. Татьяна Синицына объясняет
произошедшее тем, что МенгИсту посчитал лайнер подарком от НЕГО. В
названии какой повести 1972 года упоминается ОН?

Ответ:
"Что сказал покойник".

Комментарий:
Когда лайнер изготовили, Брежнев уже умер, поэтому МенгИсту испугался,
что ему достался подарок от покойника. В Эфиопии такое считается дурным
знаком, предвестником неудач. Африканские диктаторы тоже бывают
суеверны.

Источник:
   1. http://ria.ru/analytics/20061218/57105386.html
   2. http://pl.wikipedia.org/wiki/Ca%C5%82e_zdanie_nieboszczyka

Автор:
Серафим Шибанов (Москва)

Вопрос 8:
Мексиканский диктатор ПорфИрио ДИас управлял страной на протяжении 27
лет подряд и был свергнут, когда ему было за восемьдесят. На поздней
фотографии Диаса можно увидеть множество ИХ, в том числе персидский,
прусский и цинский. Назовите ИХ.

Ответ:
Ордена.

Комментарий:
В отличие от другого политического долгожителя, Леонида Ильича Брежнева,
ПорфИрио в свои восемьдесят держался лучше, чем Брежнев в семьдесят. А
вот ордена любили и тот, и другой. У Диаса даже был орден королевства
Гавайи.

Источник:
http://en.wikipedia.org/wiki/Porfirio_D%C3%ADaz

Автор:
Серафим Шибанов (Москва)

Вопрос 9:
Рассказывают, что султан АбдУл-ХамИд II опасался попыток отравления и
повелел держать ВТОРУЮ, с которой сам имел дело, прямо во дворце.
Назовите ВТОРУЮ.

Ответ:
Корова.

Комментарий:
Султан сам доил корову, чтобы никто не успел отравить молоко. "Корова" -
вторая сура в Коране.

Источник:
   1. http://istorfakt.ru/2010/08/13/opasnaya-rabota/
   2. http://ru.wikipedia.org/wiki/Сура_(Коран)

Автор:
Серафим Шибанов (Москва)

Вопрос 10:
   <раздатка>
   - Did Mugabe eat people like Bokassa?
   - No, people like Bokassa give him heartburn. His doctor has told him
to lay off the rich food so he is on a diet of peas at the moment.
   </раздатка>
   Перед вами анекдот про диктатора Зимбабве РОберта МугАбе, в котором,
можно сказать, мы пропустили ИХ. ОНИ и сами могут быть пищей. Назовите
ИХ.

Ответ:
Ants.

Зачет:
Муравьи.

Комментарий:
На самом деле диета МугАбе не из бобов (peas), а из крестьян (peasANTS).
Крестьяне в Зимбабве не слишком жирные и не вызывают изжогу. Приятного
аппетита всем, кто сейчас что-нибудь ест. :-)

Источник:
http://lurkmore.to/Зимбабве

Автор:
Серафим Шибанов (Москва)

Вопрос 11:
"Петрушечная" резня - это массовое убийство лиц гаитянского
происхождения, осуществленное в Доминиканской Республике по приказу
РафаЭля ЛеОнидаса ТрухИльо МолИны. Название объясняется тем, что
"петрушка" была ИМ. Назовите ЕГО правильно.

Ответ:
Шибболет.

Зачет:
Сибболет; сиболет.

Комментарий:
Кто не мог правильно выговорить слово "perejil" [перехИль] (петрушка),
того убивали мачете. Слава логопедам!

Источник:
http://ru.wikipedia.org/wiki/Этнические_чистки_в_Доминиканской_Республике_(1937)

Автор:
Серафим Шибанов (Москва)

Вопрос 12:
К угнетению никарагуанского народа диктатор АнастАсио СомОса подходил с
огоньком. Так, он запретил импорт некоторых предметов, поскольку владел
ЕЮ. "Феникс" и "Искра" - это российские ОНИ. Назовите ЕЕ двумя словами.

Ответ:
Спичечная фабрика.

Зачет:
Фабрика спичек.

Комментарий:
СомОса запретил импорт зажигалок, так как владел спичечной фабрикой.
"Феникс" и "Искра" - подходящие названия для огненной продукции.

Источник:
   1. И.А. Мусский. 100 великих диктаторов.
http://www.flibusta.is/b/159499/read
   2. http://www.feniksmatch.ru/
   3. http://www.mojgorod.ru/amursk_obl/blagovschsk/firms.html

Автор:
Серафим Шибанов (Москва)

Вопрос 13:
Во время правления АнастАсио СомОсы ДебайлЕ третьей статьей экспорта
Никарагуа - после кофе и хлопка - была ОНА. Какая ОНА, как пишет
Википедия, хорошо подходит к пёркёльту?

Ответ:
"Бычья кровь".

Зачет:
"Эгерская бычья кровь"; бычья.

Комментарий:
ОНА - это кровь. Многие никарагуанцы, чтобы не умереть с голоду, сдавали
кровь, которую потом продавали в США. Пёркёльт - блюдо венгерской кухни,
к нему хорошо подходит вино "Бычья кровь".

Источник:
   1. http://www.screen.ru/Tarasov/Nicaragua.htm
   2. http://ru.wikipedia.org/wiki/Эгерская_бычья_кровь
   3. http://ru.wikipedia.org/wiki/Пёркёльт

Автор:
Серафим Шибанов (Москва)

Вопрос 14:
Из ЕГО названия вполне ясно, для чего ОН должен применяться.
Рекордсменом по ЕГО использованию считается ПЕтар БрзИца. По
специальному заказу партия ИХ была изготовлена в годы Второй мировой
войны в немецком городе. Назовите этот город.

Ответ:
Золинген.

Комментарий:
Речь идет о сербосЕке (или серборЕзе) - специальном ноже для казни
сербов, применявшемся устАшами. ПЕтар БрзИца с помощью сербосека за ночь
убил то ли 1300, то ли 1360 сербов. Золинген известен в том числе и
своими ножами.

Источник:
http://ru.wikipedia.org/wiki/Сербосек

Автор:
Серафим Шибанов (Москва)

Вопрос 15:
В детстве ЕГО звали просто ТАчо. В испанском языке это слово имеет и
негативное значение - "ублюдок". Более известно, как о НЕМ якобы
отозвался 32-й. Какие два слова, начинающиеся на одну и ту же букву,
фигурировали в этом отзыве?

Ответ:
Сукин сын.

Зачет:
Но, наш.

Комментарий:
Тачо - это уменьшительный вариант от АнастАсио. Считается, что именно
про АнастАсио СомОсу ГарсИя 32-й президент США Франклин Делано Рузвельт
якобы сказал: "Да, это сукин сын, но это наш сукин сын". Впоследствии,
правда, выяснилось, что эта история апокрифична.

Источник:
   1. И.А. Мусский. 100 великих диктаторов.
http://www.flibusta.is/b/159499/read
   2. http://www.scepsis.net/library/id_993.html
   3. http://ru.wikipedia.org/wiki/Рузвельт,_Франклин

Автор:
Серафим Шибанов (Москва)

Вопрос 16:
   "Я знаю народ Гаити, потому что народ Гаити - это я".
   "Я гаитянский флаг".
   "Тот, кто мой враг, тот становится врагом отечества".
   Что Игорь Мусский назвал образцом для "Мыслей ДювальЕ"?

Ответ:
Красный цитатник Мао Цзэдуна.

Зачет:
По имени Мао и указанию цитатника или красной книжечки. Незачет: "Слова,
изменившие мир".

Комментарий:
Каждый гаитянин мог причаститься мудрости из этой книжечки. Франсуа
ДювальЕ, он же Папа-Док, был долгое время диктатором Гаити.

Источник:
   1. http://echo.msk.ru/blog/daniel_kotsubinsky/851834-echo/
   2. И.А. Мусский. 100 великих диктаторов.
http://www.flibusta.is/b/159499/read

Автор:
Серафим Шибанов (Москва)

Вопрос 17:
В своем выступлении, посвященном ЕМУ, министр иностранных дел АндрЕ
РаймОн напомнил, что Цезарь стал императором в девятнадцать, в том же
возрасте Уильям Питт стал премьер-министром Великобритании, а Хусейн
стал королем так и вовсе в семнадцать. Напишите ЕГО прозвище.

Ответ:
Бэби-Док.

Зачет:
Baby Doc.

Комментарий:
Жан-Клод ДювальЕ стал президентом Гаити в неполные двадцать лет, что по
тем временам было мировым рекордом, и "придворные" начали петь ему
дифирамбы в первый же день. Прозвище "Бэби-Док" было сконструировано по
аналогии с прозвищем его отца.

Источник:
http://www.vokrugsveta.ru/vs/article/4848/

Автор:
Серафим Шибанов (Москва)

Вопрос 18:
В одной стране во второй четверти двадцатого века действовал лозунг
"Rectitud, Libertad, Trabajo, Moralidad" [ректитУд, либертАд, трабАхо,
моралидАд], т.е. "Честность, свобода, труд, нравственность". Назовите
эту страну.

Ответ:
Доминиканская республика.

Зачет:
Доминикана.

Комментарий:
Лозунг обыгрывал инициалы РафаЭля ЛеОнидаса ТрухИльо МолИны, правившего
страной с 1930 по 1961 год.

Источник:
http://es.wikipedia.org/wiki/Partido_Dominicano

Автор:
Серафим Шибанов (Москва)

Вопрос 19:
По мнению автора вопроса, ОН пил кровь из народа своей страны вплоть до
31 мая 1911 года, когда сбежал в Испанию. Назовите ЕГО.

Ответ:
ХосЕ де ла Крус ПорфИрио ДИас МОри.

Зачет:
По фамилии ДИас.

Комментарий:
Имя мексиканского диктатора родственно названию болезни (порфирия),
которой, как считалось раньше, страдали вампиры. Мы не случайно в
восьмом вопросе сказали вам про гавайский, цинский и прусский ордена.
Это могло намекнуть на то, когда жил ДИас.

Источник:
   1. http://en.wikipedia.org/wiki/Porfirio_D%C3%ADaz
   2. http://ru.wikipedia.org/wiki/Порфирия

Автор:
Серафим Шибанов (Москва)

Вопрос 20:
Барракуда питается мелкой рыбой, а вот на человека может напасть только
по ошибке, приняв за мелкую рыбу его конечности. Операция по свержению
какого правителя называлась "Барракуда"?

Ответ:
Бокасса I.

Зачет:
Жан-Бедель Бокасса; Бокасса.

Комментарий:
Барракуда целенаправленно не ест человеческое мясо - в отличие от
Бокассы.

Источник:
   1. http://ru.wikipedia.org/wiki/Барракуды
   2. http://lurkmore.to/Бокасса

Автор:
Серафим Шибанов (Москва)

Вопрос 21:
Прослушайте список: Сисоват Кетарарак, Сисоват Путсара, Сисоват Дуонг
Кхара, Сисоват Ием Мария, Сисоват Понвираквонг, Сисоват Норакнорин
Дарит, Мавия. При описании судьбы этих семерых автор паблика "Цитатник
Пол Пота" в своем сообщении от 16 июля упоминает русскую фамилию.
Напишите ее.

Ответ:
Романовы.

Комментарий:
Семь перечисленных человек - члены дома СисовАтов, одного из двух
правящих домов Камбоджи до прихода к власти красных кхмеров. Все они в
итоге были убиты. 16 июля упомянуто не случайно - именно в этот день
была расстреляна семья Романовых, которых также было семеро.

Источник:
https://vk.com/saloth_sar?w=wall-28682458_418

Автор:
Серафим Шибанов (Москва)

Вопрос 22:
Жан-Бедель Бокасса во всём подражал своему кумиру. В статье энциклопедии
"Луркоморье" о Бокассе один из разделов даже называется "Неудавшиеся
ОНИ". ОНИ входят в название альбома, записанного в 1999 году. Какой
группой?

Ответ:
"Неигрушки".

Комментарий:
Бокасса ни в чем не хотел уступать Наполеону и даже после свержения
решил замутить возвращение в страну. Альбом "100 дней до приказа"
известен в основном титульной песней.

Источник:
   1. http://lurkmore.to/Бокасса
   2. https://traditio.wiki/Неигрушки_(группа)

Автор:
Серафим Шибанов (Москва)

Вопрос 23:
Во время правления Сапармурата Ниязова в центре Ашхабада возвели
огромную статую Туркменбаши, которая за счет своей конструктивной
особенности была схожа с растением. Каким?

Ответ:
Подсолнухом.

Зачет:
Гелиотропным [растением].

Комментарий:
Статуя медленно вращалась вокруг оси - "чтобы лицо Туркменбаши всегда
было обращено к солнцу".

Источник:
В.В. Мирошникова, И.Я. Вагман, Н.В. Вукина. 100 знаменитых тиранов.
http://www.flibusta.is/b/310734/read

Автор:
Серафим Шибанов (Москва)

Вопрос 24:
Рассказывают, что в Средневековье с пойманными леопардами поступали
аккуратно, чтобы не портить шкуру. Поэтому можно сказать, что к
леопардам смерть приходила ТАК. Автор надеется, что этот пакет написан
не ТАК. Назовите любого из правителей, к которым смерть, по некоторым
версиям, пришла ТАК.

Ответ:
[Муаммар] Каддафи.

Зачет:
Эдуард II; Эдуард Карнарвонский; Эдуард Плантагенет.

Комментарий:
Леопардов убивали раскаленным прутом, вонзая его в задний проход.
Рассказывают, что над Каддафи его враги также издевались похожим
образом. Более известна история про короля Англии Эдуарда II из династии
Плантагенетов, которого покарали именно раскаленной кочергой.

Источник:
   1. http://istorfakt.ru/2010/09/07/gryaznoe-ubijstvo/
   2. http://www.globalpost.com/dispatch/news/regions/middle-east/111024/gaddafi-sodomized-video-gaddafi-sodomy
   3. http://ru.wikipedia.org/wiki/Эдуард_II

Автор:
Серафим Шибанов (Москва)

Тур:
10 тур. "Кровлю Кромвеля снесло" (Москва)

Редактор:
Мария Киселёва, Александр Берелехис (Уфа)

Вопрос 1:
В стихотворении автора вопроса сосед после прогулки с псом приносит
домой ЕГО. Назовите ЕГО тремя словами.

Ответ:
Пакет с г...ном.

Комментарий:
Такой вот воспитанный человек. И не говорите после тура о том, что мы
вас не предупреждали о состоянии пакета. :-)

Источник:
Стихотворение автора вопроса.

Автор:
Сергей Вознесенский (Москва)

Вопрос 2:
На картине Виктора Пузо, созданной по мотивам известного изображения, в
бутылке водки разлагается ОН. Назовите ЕГО коротким словом.

Ответ:
Луч.

Комментарий:
Как на обложке альбома "Pink Floyd".

Источник:
http://victorpuzo.livejournal.com/3042931.html

Автор:
Андрей Алдашев (Москва)

Вопрос 3:
По легенде, казна Таллина хранилась в сундуке с тремя замками. Горожане
выбирали казначеев так, чтобы они были ИКСАМИ, и доверяли каждому лишь
один из ключей. Где находится ИКС в названии фильма 2001 года?

Ответ:
У ворот.

Комментарий:
"Враг у ворот" - голливудская военная драма, действие которой происходит
в Сталинграде. Жители эстонской столицы разумно предполагали, что вражда
и взаимное недоверие не позволят казначеям сговориться и совершить
кражу.

Источник:
   1. Э. Тополь. Московский полет. http://www.flibusta.is/b/402609/read
   2. http://ru.wikipedia.org/wiki/Враг_у_ворот

Автор:
Андрей Алдашев (Москва)

Вопрос 4:
   <раздатка>
   Ст. 11. Нарсудам подсудны гражданские дела между _____________, на
рассмотрение которых стороны выразят свое письменное согласие в особых
подписках или заявлениях, кроме дел, подсудных сельнарсуду, по коим
достаточно заявление истца.
   </раздатка>
   Перед вами фрагмент постановления 1921 года, касающегося судебной
системы в одной из советских республик. Догадавшись, какую букву мы в
нем дважды заменили, восстановите пропущенное слово.

Ответ:
Мусульманами.

Комментарий:
В Дагестанской ССР в начале Советской власти действовали не только
народные, но и шариатские суды. Естественно, в шарсуд обращались только
мусульмане.

Источник:
http://www.dargo.ru/publ/28-1-0-759

Автор:
Андрей Алдашев (Москва)

Вопрос 5:
Герой англоязычного фильма 2013 года приходит, переодевшись врачом, в
больницу за сосудами с кровью. Отмечая, что пристрастия героя довольно
необычны, продавец, перед тем как выдать кровь, называет героя ИКСОМ.
Назовите ИКСА двумя словами.

Ответ:
Доктор Стрейнджлав.

Комментарий:
В английском получается игра слов: любовь героя к крови довольно
странная. :-)

Источник:
Х/ф "Выживут только любовники" (2013), реж. Джим Джармуш.

Автор:
Никита Коровин (Москва)

Вопрос 6:
Блиц.
   1. По-чешски "окурки" - это "огурцы". А каким словом чехи называют
окурки?
   2. В английском варианте известной идиомы вместо НИХ упоминается
спица, что довольно странно - спицы там и так, скорее всего, есть.
Назовите ИХ.
   3. Название комедии Карло Гольдони дали ОНИ между жительницами
Кьоджи, отличавшимися злыми языками. Назовите ИХ.

Ответ:
   1. Недопалки.
   2. Палки.
   3. Перепалки.

Комментарий:
   1. От "недопаленный", т.е. не сожженный до конца.
   2. По-английски "to put a spoke in someone's wheel" - "вставлять
спицу в колеса".

Источник:
   1. http://mi3ch.livejournal.com/2987178.html
   2. http://translate.academic.ru/put%20a%20spoke%20in%20someone%27s%20wheel/en/ru/
   3. К. Гольдони. Кьоджинские перепалки.
http://www.flibusta.is/b/237163/read

Автор:
Андрей Алдашев, Никита Коровин (Москва)

Вопрос 7:
Плиний Старший пишет, что фессалийцы в определенное время поднимали
жуткий шум, чтобы не дать СДЕЛАТЬ ЭТО. Забавно, что книжный магазин
"Плиний Старший" провел 1 апреля 2009 года акцию в честь человека,
который описал, как СДЕЛАЛИ ЭТО. Назовите этого человека.

Ответ:
[Николай Васильевич] Гоголь.

Комментарий:
Во время лунных затмений фессалийцы поднимали шум, чтобы не дать
нечистой силе похитить Луну. В произведении Николая Васильевича месяц с
неба похитить все-таки удалось. :-) 1 апреля 2009 года исполнилось 200
лет со дня рождения Гоголя.

Источник:
   1. Дж. Грант. Отвергнутая наука. Самые невероятные теории, гипотезы,
предположения. http://www.flibusta.is/b/335577/read
   2. http://www.pressagenda.com/novosibirsk/7074

Автор:
Никита Коровин (Москва)

Вопрос 8:
В романе Рэймонда Чандлера об одном из героев - враче по профессии,
говорится, что тот приезжает к посетителям ночью и не дает им
"позавтракать" ТАКИМИ ИМИ. В меню одного из завтраков Дэвида Ливингстона
приводятся пиво и ОН, правда обыкновенный. Какие слова мы заменили на
"ТАКОЙ ОН"?

Ответ:
Розовый слон.

Комментарий:
"Видеть розовых слонов/слоников" - эвфемизм для обозначения
галлюцинаций, вызванных белой горячкой, ну а у Чандлера ими можно и
позавтракать. Ливингстон тоже любил алкоголь, но вот слон на его
завтраке был вполне реальным и питательным. :-)

Источник:
   1. http://ae-lib.org.ua/texts-c/chandler__the_lady_in_the_lake__en.htm
   2. http://neobychno.com/20493/shashlyk-iz-begemot/

Автор:
Никита Коровин (Москва)

Вопрос 9:
Описывая развитие медицины, Говард Хаггард говорит, что в XVIII веке
представителей некой профессии пренебрежительно прозвали "ТАКИМИ
дедами". Назовите современное наименование этой профессии.

Ответ:
Акушер.

Комментарий:
В XVIII веке акушерство было исключительно женским занятием, и
мужчин-акушеров пренебрежительно называли "повивальными дедами" по
аналогии с "повивальными бабками".

Источник:
Г. Хаггард. От знахаря до врача. История науки врачевания.

Автор:
Никита Коровин (Москва)

Вопрос 10:
Персонаж фильма "Газонокосильщик" - умственно отсталый юноша. При помощи
погружения в киберпространство он за считанные дни становится гением.
Один из русскоязычных комментаторов замечает, что такое вряд ли
возможно, и предполагает, что персонаж попросту ДЕЛАЛ ЭТО. В январе
прошлого годя многие смогли СДЕЛАТЬ ЭТО благодаря предложению Владимира
Путина. Что мы заменили на "ДЕЛАЛ ЭТО"?

Ответ:
Косил от армии.

Зачет:
Косил от призыва.

Комментарий:
Комментатор предположил, что герой попросту симулировал отставание в
развитии, чтобы не служить в армии. Предложение президента России
Владимира Путина увеличить срок пребывания в России украинцев
способствовало тому, что многие украинцы уезжали в Россию косить от
армии.

Источник:
   1. Х/ф "Газонокосильщик" (1992), реж. Бретт Леонард.
   2. http://www.kinopoisk.ru/user/1172604/comment/2113809/
   3. https://meduza.io/news/2015/01/28/ukraintsam-prizyvnogo-vozrasta-prodlili-srok-prebyvaniya-v-rf

Автор:
Никита Коровин (Москва)

Вопрос 11:
Варвары старались извлечь максимальную выгоду из покорения других
народов. Современный исследователь пишет, что они пополняли не только
свою казну и армию, но и ЕГО. Назовите ЕГО словом с двумя корнями.

Ответ:
Генофонд.

Комментарий:
Женщин чаще всего брали в наложницы и таким образом пополняли генофонд
племени.

Источник:
Ж. Ле Гофф. Рождение Европы. http://www.flibusta.is/b/265799/read

Автор:
Никита Коровин (Москва)

Вопрос 12:
Дуплет.
   На ИКС в обеих частях вопроса заменено одно и то же слово.
   1. В числе ИКСОВ, входивших в ЕГО состав, были Озерный, Речной,
Луговой, Таежный. Назовите ЕГО пятибуквенным словом.
   2. В числе ИКСОВ, входящих в ЕГО состав, имеются Озерный, Речной,
Полевой, Морской. Назовите ЕГО пятибуквенным словом.

Ответ:
   1. ГУЛАГ.
   2. Артек.

Источник:
   1. http://ru.wikipedia.org/wiki/Список_лагерей_ГУЛага
   2. http://www.artek.org/ob-arteke/obshaya-informaciya/

Автор:
Андрей Алдашев (Москва)

Вопрос 13:
Герои фильма "Машина времени в джакузи" попадают в далекое будущее. Один
из персонажей, надев очки, вспоминает "Короля". Какое слово в предыдущем
предложении мы заменили?

Ответ:
Газонокосильщика.

Зачет:
"Газонокосильщика".

Комментарий:
Очки позволяют перенестись в виртуальную реальность. Совсем как в фильме
"Газонокосильщик", о котором мы вам уже рассказали. :-) У Кинга есть
рассказ "Газонокосильщик" (англ. The Lawnmower Man).

Источник:
Х/ф "Машина времени в джакузи" (2010), реж. Стив Пинк.

Автор:
Никита Коровин (Москва)

Вопрос 14:
Можно сказать, что причиной сумасшествия героя рассказа 1869 года стала
неудачная шутка сослуживцев, однако герой получил прозвище "ОН". В
русском переводе 1995 года название "ОН" получило... Что?

Ответ:
Детективное агентство.

Комментарий:
До прихода современной психиатрии в депрессиях, в плохом сне и порой в
психических заболеваниях винили Луну. Герой, ставший жертвой неудачной
ночной шутки, получил прозвище Лунный Свет, а в переводе зарубежного
сериала детективное агентство "Голубая Луна" решили назвать "Лунным
светом", чтобы избежать ассоциаций.

Источник:
   1. http://ru.wikisource.org/wiki/Джак_Лунный_Свет
   2. http://ru.wikipedia.org/wiki/Детективное_агентство_%C2%ABЛунный_свет%C2%BB

Автор:
Никита Коровин (Москва)

Вопрос 15:
Во время одного из своих путешествий Дэвид Ливингстон оказался на борту
старого парохода. Называя пароход ужасающе жалким судном, Ливингстон
отметил, что двигатель годится только для ЭТОГО. На сайте kivahan.ru
[кивахан точка ру] говорится, что степень ЭТОГО - не менее важный
фактор, чем, например, сорт. Назовите ЭТО двумя словами.

Ответ:
Помол кофе.

Комментарий:
Ливингстон известен как исследователь Африки, славящейся, в том числе, и
своим кофе.

Источник:
   1. П. Брендон. Упадок и разрушение Британской империи. 1781-1997.
http://www.flibusta.is/b/347591/read
   2. http://kivahan.ru/sposoby-pomola-kofe/

Автор:
Никита Коровин (Москва)

Вопрос 16:
Герой фильма называет себя Волчестай, поясняя, что родители ошиблись с
именем. Какую профессию они ему предрекали?

Ответ:
Композитор.

Комментарий:
Волчестай - вольный (и довольно ошибочный) вариант имени Вольфганг.
Родители хотели назвать его в честь Моцарта, надеясь, что сын тоже
станет композитором.

Источник:
Х/ф "Звездный ворс" (2012), реж. Андрей Кагадеев, Николай Копейкин.

Автор:
Андрей Алдашев (Москва)

Вопрос 17:
Отчаявшаяся пациентка психолога Мэг Джей - девственница, которая не
может завести отношения с мужчинами. У психолога складывается
впечатление, что она постоянно видит вышитую на груди девушки ЕЕ. Что
значит ОНА в произведении 2006 года?

Ответ:
Вендетта.

Комментарий:
Пациентка Мэг Джей - девственница, которая не может завести отношения, и
ей кажется, что на ее груди вышита буква "V" (virgin - девственница).
Фильм "V - значит вендетта" вышел в 2006 году.

Источник:
   1. М. Джей. Важные годы. Почему не стоит откладывать жизнь на потом.
http://www.flibusta.is/b/426210/read
   2. http://www.kinopoisk.ru/film/86208/

Автор:
Анатолий Шкатов (Липецк)

Вопрос 18:
В фильме студии "Дисней" тренер обращается за помощью к богачу, после
чего талисманом команды становятся не самые героические животные. Из
какого города профессиональная команда, в названии которой они также
присутствуют?

Ответ:
Анахайм.

Комментарий:
На волне успеха фильма "Могучие утки" компания "Дисней" создала команду
НХЛ "Mighty Ducks of Anaheim". Кроме того, известный диснеевский богач
Скрудж МакДак - тоже утка.

Источник:
http://ru.wikipedia.org/wiki/Анахайм_Дакс

Автор:
Андрей Алдашев (Москва)

Вопрос 19:
Среди прочих вещей, которые по закону не могли находиться в ИКСЕ, в
списке 2014 года можно было увидеть еду, напитки и даже лекарства. В
другом "ИКСЕ" упоминаются надежды, тревоги, подсчеты и ожидания.
Назовите ИКС двумя словами, начинающимися на одну и ту же букву.

Ответ:
Багаж болельщика.

Зачет:
"Багаж болельщика".

Источник:
   1. https://www.opentown.org/news/26979/
   2. Л.А. Кассиль. Спортивные рассказы.

Автор:
Ульяна Бабина (Москва)

Вопрос 20:
На съемочной площадке современного фильма "Батальонъ" всё происходило в
действительности: актрис брили наголо, они дрались и стреляли из
рабочего оружия. Хотя все девушки были в одинаковых условиях, а съемки
проходили под Петербургом, актрисы настолько вжились в роль, что, по
мнению автора вопроса, можно было подумать, будто процессом руководил...
Кто?

Ответ:
[Филип] Зимбардо.

Комментарий:
Действия на площадке напоминали условия знаменитого Стэнфордского
эксперимента.

Источник:
Рецензия автора вопроса.

Автор:
Ульяна Бабина (Москва)

Вопрос 21:
Дуплет.
   1. На картинке из детской раскраски белый медвежонок [ПРОПУСК] и
глазеет оттуда по сторонам. Заполните пропуск тремя словами.
   2. Моцарт был вынужден жениться на Констанце Вебер, после того как ее
мать [ПРОПУСК] слухи, что композитор обесчестил девушку. Заполните
пропуск тремя словами.

Ответ:
   1. "... сел на иглу...".
   2. "... пустила по Вене...".

Источник:
   1. http://www.raskraska.com/raskraski/87/56.html
   2. В.Б. Миронов. Народы и личности в истории. Т. 2.
http://www.flibusta.is/b/400344/read

Автор:
Андрей Алдашев (Москва)

Вопрос 22:
Внимание, в вопроса есть замена.
   По сюжету романа Эдвина Арнолда лейтенант Джонс находит на улице
волшебный артефакт и на вопрос, куда он хочет отправиться, отвечает: "Да
хоть ТУДА". Ряд исследователей полагают, что книга Арнольда могла
попасть в руки... К кому?

Ответ:
[Эдгар Райс] Берроуз.

Комментарий:
ТУДА - на Марс. Вообще отношения Берроуза с "антиплагиаторами" больше
строились на почве цикла книг о Тарзане и очевидном сходстве героя с
Маугли, но и во второй по популярности серии, посвященной Джону Картеру
и принцессе Марса, критики смогли разглядеть параллели.

Источник:
http://barros.rusf.ru/article042.html

Автор:
Никита Коровин (Москва)

Вопрос 23:
В 2011 году Банк Москвы сумел удержаться на плаву, благодаря
вмешательству ВТБ. Интересно, что VTB является другим вариантом. Чего
именно?

Ответ:
Сигнала SOS.

Комментарий:
Последовательность точек и тире в сигнале бедствия является непрерывной,
так что можно трактовать ее как совокупность букв азбуки Морзе
по-разному.

Источник:
   1. http://ru.wikipedia.org/wiki/Банк_Москвы
   2. http://ru.wikipedia.org/wiki/SOS

Автор:
Андрей Алдашев (Москва)

Вопрос 24:
Во временном строении в нескольких кварталах от Женевского озера
регулярно ДЕЛАЮТ ЭТО. Примерно в восьми километрах от этого места с 1998
года некая компания тоже, можно сказать, ежегодно ДЕЛАЕТ ЭТО. Назовите
эту компанию.

Ответ:
Форд.

Комментарий:
В рождественские каникулы в Женеве устанавливают цирк-шапито.
Выступления иллюзионистов - традиционная часть цирковой программы. На
ежегодном Женевском автосалоне в числе прочих моделей компания "Форд"
показывает различные "Фокусы".

Источник:
   1. http://www.cirque-de-noel.ch/index.html
   2. http://ru.wikipedia.org/wiki/Женевский_автосалон
   3. http://ru.wikipedia.org/wiki/Ford_Focus

Автор:
Андрей Алдашев (Москва)

Тур:
11 тур. "Брейн" (Москва)

Редактор:
Валерия Комаровская (Коломна)

Вопрос 1:
В американском жестовом языке слово "молоко" показывают кулаком правой
руки, два раза сжимая его особенным образом. Для того чтобы показать
ТАКОЕ молоко, можно провести кулаком правой руки мимо лица ниже бровей,
одновременно показывая им слово "молоко". Какое молоко показывают таким
образом?

Ответ:
Пастеризованное.

Комментарий:
Pasteurized milk - past your eyes, мимо (своих) глаз. Игра слов,
действительно используемая в американском жестовом языке.

Источник:
http://www.lifeprint.com/asl101/pages-signs/m/milk.htm

Автор:
Дарья Беличенко

Вопрос 2:
Несмотря на неоспоримую пользу, приносимую АЛЬФОЙ, от АЛЬФЫ есть и вред:
АЛЬФА прямо или косвенно является причиной примерно 18% парниковых
газов. Кроме того, из-за АЛЬФЫ ежегодно погибает примерно в двадцать раз
больше людей, чем от нападения акул. Назовите АЛЬФУ.

Ответ:
Корова.

Комментарий:
Несмотря на популярное мнение об опасности стать жертвой акул, от
нападения акул погибает примерно один человек в год; из-за коров
погибает около двадцати человек в год.

Источник:
   1. http://www.factroom.ru/facts/1202
   2. https://www.washingtonpost.com/news/wonk/wp/2015/06/16/chart-the-animals-that-are-most-likely-to-kill-you-this-summer/

Автор:
Дарья Беличенко

Вопрос 3:
В британском ситкоме "Компьютерщики" в одной из серии айтишник Моррис
Мос отправляет своим знакомым из IT-сообщества e-mail с описанием
симптомов ПМС у мужчин. Его e-mail очень быстро распространяется,
наводняя собой Интернет. Также этот e-mail приводит к бунтам и
беспорядкам, устроенным компьютерщиками со всего мира, обнаружившими у
себя данные симптомы. Этот эпизод является аллюзией на событие,
произошедшее в 1988 году в Массачусетском университете. Назовите
представителя фауны, упоминающегося в названии, которое получило данное
событие.

Ответ:
Червь.

Комментарий:
В вопросе речь идет о черве Морриса - одном из первых сетевых червей,
распространяемых через Интернет.

Источник:
   1. Сериал "Компьютерщики", s01e06.
   2. http://ru.wikipedia.org/wiki/Червь_Морриса

Автор:
Дарья Коротина

Вопрос 4:
В комедийном мюзикле "Tenacious D: Медиатор судьбы" два рокера ищут
медиатор судьбы, чтобы с его помощью добиться успеха. Этот магический
артефакт находится в музее истории рока. По пути туда между героями
происходит разлад, так как один из них решает провести вечер в компании
трех красоток, вместо того чтобы сразу отправиться в музей. Свой выбор
он оправдывает известной триадой, причем уточняет, что порядок именно
такой. Назовите эту триаду.

Ответ:
Секс, наркотики, рок-н-ролл.

Источник:
Мюзикл "Tenacious D: Медиатор судьбы".

Автор:
Дарья Коротина

Вопрос 5:
На этикетке товара, изготовленного специально для сети магазинов
"ДИКСИ", изображено использование этого товара не по прямому назначению,
а именно в кулинарии. Также не по назначению этот предмет используется
для маникюра или даже для изготовления музыкального инструмента. Для
использования по прямому назначению вы можете приобрести как
классический вариант, так и ароматизированный, который наряду с приятным
запахом имеет еще и антисептический эффект. Назовите этот предмет.

Ответ:
Зубочистка.

Комментарий:
На этикетке зубочистка использована для канапе. Для маникюра зубочистка
используется вместо кисточки для рисования на ногтях. Музыкальный
инструмент, который был упомянут в вопросе, - это рейнстик, изначально
культовый музыкальный инструмент в Чили и Перу, а сейчас очень
популярный среди музыкантов фолка, джаза и других музыкальных жанров.

Источник:
http://ru.wikipedia.org/wiki/Зубочистка

Автор:
Дарья Коротина

Вопрос 6:
В Волшебной стране Волкова резиденцией правителей Фиолетовой страны
является Фиолетовый дворец, в Розовой стране - Дворец Стеллы - Розовый
дворец. А как называется дворец в Стране подземных рудокопов?

Ответ:
Радужный.

Комментарий:
В Стране подземных рудокопов было семь королей, каждому из которых
принадлежало по одной башне во дворце. Башни были выкрашены в семь
цветов радуги.

Источник:
http://ru.wikipedia.org/wiki/Волшебная_страна_(Волков)#.D0.97.D0.B0.D0.BC.D0.BA.D0.B8_.D0.B8_.D0.B4.D0.B2.D0.BE.D1.80.D1.86.D1.8B

Автор:
Дарья Коротина

Вопрос 7:
Литература знает множество различных вальсов: это и "Вальс на прощание"
Милана Кундера, и "Полночный вальс" Джулии Куин. Однако самым
оригинальным является именно ТАКОЙ вальс. ТАКИМ берегом называют бухту
Изтузу в месте впадения реки Дальян в Средиземное море. Просим вас не
медлить и назвать прилагательное, которое мы заменили в вопросе ТАКИМ.

Ответ:
Черепаший.

Комментарий:
"Черепаший вальс" - название романа современной французской писательницы
Катрин Панколь. Бухта, упомянутая в вопросе, называется Черепаший берег,
так как она является одним из основных мест нерестилища черепах
логгерхедов, занесенных в Красную книгу.

Источник:
   1. http://ru.wikipedia.org/wiki/Панколь,_Катрин
   2. http://ru.wikipedia.org/wiki/Черепаший_берег_Изтузу

Автор:
Дарья Коротина

Вопрос 8:
Героиня американского писателя начала XX века, подобно любящему влагу и
плодородную почву цветку, имя которого носит, готова выпить все соки из
своего поклонника. Ее тезка - героиня романа современной французской
писательницы, в противопоставление цветку, не любит быть в тени.
Назовите имя девушек, если известно, что цветок был завезен в Европу из
Китая и Японии в начале XIX века и был назван, по одной из версий, в
честь королевы Голландии.

Ответ:
Гортензия.

Комментарий:
Первая девушка - это Гортензия Бригс, героиня романа Теодора Драйзера
"Американская трагедия". Вторая девушка - это Гортензия Кортес, героиня
романа Катрин Панколь "Желтоглазые крокодилы". Hortensia - слово
латинского происхождения и в переводе на русский означает "садовые
культуры". Королевой Голландии с 1806 по 1810 год была Гортензия де
Богарне.

Источник:
   1. Т. Драйзер. Американская трагедия.
http://www.flibusta.is/b/453956/read
   2. К. Панколь. Желтоглазые крокодилы.
http://www.flibusta.is/b/386009/read
   3. http://ru.wikipedia.org/wiki/Гортензия
   4. http://ru.wikipedia.org/wiki/Богарне,_Гортензия
   5. http://www.lingvo-online.ru/ru/Translate/la-ru/hortensia

Автор:
Дарья Коротина

Вопрос 9:
"Учитель", "Директор" и "Хирург" являются ПЕРВЫМИ. "Пианист", "Таксист"
и "Дворецкий" являются ВТОРЫМИ. А "Наемный убийца" является и ПЕРВЫМ, и
ВТОРЫМ. Назовите в правильном порядке слова, которые мы заменили словами
"ПЕРВЫЙ" и "ВТОРОЙ".

Ответ:
Роман, фильм.

Комментарий:
"Учитель" - роман Шарлотты Бронте, "Директор" - роман Джозефа Файндера,
"Хирург" - роман Тесс Герритсен. "Пианист" - фильм Романа Полански.
"Таксист" - фильм Мартина Скорсезе. "Дворецкий" - фильм Ли Дэниелса.
"Наемный убийца" - это и роман Грэма Грина, и фильм Джона Ву.

Источник:
   1. https://www.livelib.ru/book/1001111336
   2. https://www.livelib.ru/book/1000318746
   3. https://www.livelib.ru/book/1001523306
   4. http://www.kinopoisk.ru/film/355/
   5. http://www.kinopoisk.ru/film/358/
   6. http://www.kinopoisk.ru/film/464446/
   7. https://www.livelib.ru/book/1000701652
   8. http://www.kinopoisk.ru/film/532/

Автор:
Дарья Коротина

Вопрос 10:
Название известной отечественной компании состоит из двух слов. Первое
обозначает род грызунов. Второе созвучно названию романа Валентина
Пикуля. Назовите то, что производит эта компания, если известно, что в
одном из советских фильмов ее продукция считалась величайшей ценностью и
сильно повышала социальный статус обладателей.

Ответ:
Спички.

Комментарий:
Компания называется "Белка-Фаворит". Фильм, упомянутый в вопросе, -
"Кин-дза-дза!". На планете Плюк КЦ, они же спички, являлись наивысшей
ценностью.

Источник:
   1. http://www.belkafavorit.com/
   2. http://ru.wikipedia.org/wiki/Белка
   3. http://ru.wikipedia.org/wiki/Фаворит_(роман)
   4. http://ru.wikipedia.org/wiki/Устройства_и_предметы_мира_%C2%ABКин-дза-дза!%C2%BB

Автор:
Дарья Коротина

Вопрос 11:
В компьютерной игре Эдмунда Макмиллена "The binding of Isaac" [зэ
бАйндинг оф Айзэк] ("Жертвоприношение Исаака") очень много отсылок к
библейской тематике. Например, одной из групп игровых боссов являются
семь смертных грехов. Кого именно автор игры изобразил в качестве
усложненной, так называемой "ультра"-версии босса "Гордыня"?

Ответ:
Себя.

Источник:
   1. Компьютерная игра "The Binding of Isaac".
   2. http://www.edmundm.com

Автор:
Георгий Карпич

Вопрос 12:
В логотипе компьютерной игры "Quake" [куЭйк] ОН один. В пиктограмме
одного из артефактов компьютерной игры "The binding of Isaac" [зэ
бАйндинг оф Айзэк] ИХ два, и длина ИХ составляет около 20,3 сантиметров,
что несколько меньше, чем в названии американской музыкальной группы.
Назовите материал, из которого предлагал делать ИХ русский поэт.

Ответ:
Люди.

Зачет:
По смыслу.

Комментарий:
Речь идет о гвоздях. В логотипе "Quake" один гвоздь, в "The binding of
Isaac" присутствует артефакт "8-inch nails" (восьмидюймовые гвозди).
Группа, разумеется, - "Nine Inch Nails" (девятидюймовые гвозди).
Писатель - Николай Тихонов (а не Маяковский!), который в своей "Балладе
о гвоздях" писал "Гвозди бы делать из этих людей".

Источник:
   1. Компьютерная игра "The binding of Isaac".
   2. http://ru.wikipedia.org/wiki/Nine_Inch_Nails
   3. http://dic.academic.ru/dic.nsf/dic_wingwords/575/

Автор:
Георгий Карпич

Вопрос 13:
Том Хиддлстон - британский актер театра и кино - сказал: "У человека две
жизни, и вторая начинается тогда, когда мы понимаем, что...". Закончите
фразу двумя словами, если известно, что недавно в Саратове ГИБДД провела
мемориальную акцию, названием для которой послужили эти же два слова.

Ответ:
"... жизнь одна".

Источник:
   1. http://www.goodreads.com/quotes/865874-we-all-have-two-lives-the-second-one-starts-when
   2. https://64.mvd.ru/news/item/6766755/

Автор:
Максим Фомин

Вопрос 14:
В мексиканском городе Сьюдад-Хуарес и его окрестностях уже не один
десяток лет происходят массовые изнасилования и убийства женщин.
Подозревается, что многие из них совершались водителями автобусов,
атаковавших остававшихся в одиночестве пассажирок. В 2013 году
неизвестная женщина устроила охоту на таких водителей, застрелив двоих с
разницей в сутки. После этого местная газета получила письмо, в котором
неизвестная пообещала и дальше мстить за всех убитых женщин. Письмо было
подписано именем персонажа античной мифологии. Назовите другую тезку
этого персонажа, известную во всём мире и бывшую объектом пристального
внимания телекамер и светских хроник.

Ответ:
Принцесса Диана.

Зачет:
Леди Диана; Диана Уэльская.

Комментарий:
Неизвестная женщина подписала письмо "Диана, охотница на водителей",
используя имя римской богини-охотницы, также являющейся покровительницей
женщин. А самой известной Дианой в истории была и остается Диана,
принцесса Уэльская, жена британского наследного принца Чарльза.

Источник:
http://www.thisamericanlife.org/diana-hunter-of-bus-drivers/

Автор:
Антон Лазарев

Вопрос 15:
Читая названия участников московского музыкального фестиваля "Most Sad
Gig Ever II", автор вопроса обратил внимание на любопытную
закономерность: названия большей части выступающих групп были выдержаны
в общей стилистике. В концерте, помимо прочих, принимали участие
"Причал", "МАЯК" и "Toluca" (чье название отсылает к вымышленному озеру,
на берегу которого стоит город Сайлент-Хилл). А группа из Петрозаводска
презентовала песни со своего альбома "Меж каменных плит". Как же
называется группа из Петрозаводска?

Ответ:
"Materic".

Комментарий:
Причалы и маяки возводят на границе воды и суши, которая, как правило,
является частью материка, ну а название альбома можно трактовать как
отсылку к материковым плитам.

Источник:
http://www.last.fm/ru/event/4031051+Most+Sad+Gig+Ever+II

Автор:
Антон Лазарев

Вопрос 16:
Классические компьютерные игры жанра "квест" часто славились загадками,
плюющими в лицо логике. Для решения одной из самых известных таких
задачек в игре "Monkey Island 2: LeChuck's Revenge" необходимо
использовать парализованную обезьяну в качестве этого предмета.
По-видимому, разработчики рассчитывали, что игроки вспомнят английское
название разновидности этого предмета, которое и правда напоминает о
приматах. В русском же языке разновидности этого предмета ассоциируются
с парой европейских стран, а увидеть его в повседневной жизни чаще всего
можно по обочинам автодорог. Назовите этот предмет.

Ответ:
Разводной ключ.

Комментарий:
В английском языке одна из разновидностей разводного гаечного ключа
называется "monkey wrench" (впрочем, к обезьянам оно этимологического
отношения не имеет), и в игре при помощи парализованной мартышки
необходимо отвернуть большую гайку. В русском же языке имеются шведские
и французские ключи - последние изображены на знаках "Техническое
обслуживание автомобилей".

Источник:
   1. http://thementalattic.com/2015/04/13/the-weekly-puzzle-monkey-business/
   2. http://en.wikipedia.org/wiki/Monkey_wrench
   3. http://ru.wikipedia.org/wiki/Разводной_ключ

Автор:
Антон Лазарев

Вопрос 17:
На судебном заседании Приморского городского суда в сентябре 2015 года
супружеской паре из Германии было отказано в усыновлении российского
ребенка, поскольку суд предположил, что, являясь гражданами Германии,
они могут сначала СДЕЛАТЬ ПЕРВОЕ, а потом СДЕЛАТЬ ВТОРОЕ. ПЕРВОЕ СДЕЛАЛИ
в 2001 году американский актер, трижды номинированный на премию "Оскар",
и актриса, снявшаяся с ним в нескольких фильмах (например, в фильме 1990
года про автогонки). ВТОРОЕ СДЕЛАЛИ в 2015 году британский актер,
игравший камердинера в комедийном сериале начала 1990-х годов, и
британский стендап-комик. О каких двух действиях идет речь?

Ответ:
Развестись и вступить в однополый брак.

Комментарий:
Актер - Том Круз, актриса - Николь Кидман (фильм "Дни грома"). Актер -
Стивен Фрай (сериал "Дживс и Вустер"), комик - Эллипс Спенсер.

Источник:
   1. http://takiedela.ru/2015/12/adopting-a-child/
   2. http://ru.wikipedia.org/wiki/Круз,_Том
   3. http://ru.wikipedia.org/wiki/Дни_грома_(фильм,_1990)
   4. http://ru.wikipedia.org/wiki/Фрай,_Стивен

Автор:
Дарья Беличенко

Вопрос 18:
Герой романа Людмилы Улицкой "Казус Кукоцкого" Павел Алексеевич Кукоцкий
пытается добиться ИХ легализации, демонстрируя партийному деятелю
проросшую луковицу. В качестве средств совершения этого запрещенного
действия среди прочего он упоминает ножницы и мыльную воду. Что просил
легализовать Кукоцкий?

Ответ:
Аборт.

Комментарий:
Действие романа происходит в период с момента образования СССР и до
середины 60-х годов XX века. Тогда аборты в СССР были запрещены, и
множество женщин прибегали к народным методам, которые приводили к
невозможности иметь детей в будущем, а зачастую и к смерти женщин.
Кукоцкий был врачом-гинекологом, который ежедневно сталкивался с
подобными случаями. Партийному деятелю Кукоцкий демонстрировал
беременную матку с проросшей луковицей. Луковица вводится в шейку матки,
прорастает. Корневая система пронизывает плод, после чего извлекается
вместе с плодом.

Источник:
Л.Е. Улицкая. Казус Кукоцкого.

Автор:
Дарья Коротина

Вопрос 19:
Свою карьеру ОН начал с того, что писал о муравьях. Следующим шагом была
работа об исследованиях потустороннего мира. Одним из ЕГО любимых
женских образов в искусстве является Мэрилин Монро, которая фигурирует в
одной из ЕГО работ. Назовите ЕГО.

Ответ:
[Бернар] Вербер.

Комментарий:
Бернар Вербер - современный французский писатель. В вопросе упоминаются
следующие его произведения: "Муравьи", "День муравья", "Танатонавты".
Мэрилин Монро является одной из героинь романа "Империя ангелов" из
дилогии "Танатонавты".

Источник:
   1. http://ru.wikipedia.org/wiki/Вербер,_Бернар
   2. http://ru.wikipedia.org/wiki/Империя_ангелов

Автор:
Дарья Коротина

Вопрос 20:
В финальной сцене фильма 2006 года по подоконнику пробегает крыса. Это
очень символично, ведь сюжет фильма заключается в том, что участники
двух противоборствующих объединений внедрили друг другу тайного агента и
в каждом пытаются вычислить предателя. Этот фильм является ремейком
другого, русское название которого состоит из двух слов: одно -
прилагательное, другое - шахматный термин. Напишите это название.

Ответ:
"Двойная рокировка".

Комментарий:
В вопросе описан сюжет фильма Мартина Скорсезе "Отступники".

Источник:
http://ru.wikipedia.org/wiki/Отступники

Автор:
Дарья Коротина

Вопрос 21:
Название фильма с Натальей Негодой в главной роли состоит из двух слов,
каждое из которых обозначает музыкальный инструмент. Первое слово входит
в состав фразеологизма со значением "побить", а второе - в состав
фразеологизма со значением "не важно". Напишите название фильма.

Ответ:
"Бубен, барабан".

Комментарий:
В вопросе упомянуты фразеологизмы "дать в бубен" и "по барабану".

Источник:
   1. http://ru.wikipedia.org/wiki/Бубен,_барабан
   2. http://ru.wiktionary.org/wiki/дать_в_бубен
   3. http://ru.wiktionary.org/wiki/по_барабану

Автор:
Дарья Коротина

Вопрос 22:
В одном из советских фильмов АЛЬФА - это прозвище героини, которая
является родственницей сотрудника правоохранительных органов. По иронии
судьбы, именно АЛЬФА становится причиной смерти обидчиков этой героини.
А один из альбомов группы, в названии которой тоже есть АЛЬФА,
называется "Девушка за рулем". Что мы заменили АЛЬФОЙ?

Ответ:
Авария.

Комментарий:
Авария - прозвище девочки Валерии, отец которой является милиционером
(фильм "Авария - дочь мента"). Обидчики - это банда мажоров, которые
совершили групповое изнасилование Валерии. Они заживо сгорают в
автомобильной аварии. Упомянутая в вопросе группа - "Дискотека Авария".
А название ее альбома - это еще одна подсказка, так как, по бытующему в
народе мнению, девушка за рулем - это основная причина автомобильных
аварий.

Источник:
   1. Х/ф "Авария - дочь мента" (1989), реж. Михаил Туманишвили.
   2. http://ru.wikipedia.org/wiki/Дискотека_Авария

Автор:
Дарья Коротина

Вопрос 23:
   <раздатка>
   "Однако я не рекомендовал бы это начинающим, так как здесь легко
впасть в крайность, ПРОПУСК и не получить желаемую позицию в связи с
недостаточной квалификацией".
   Борис Левин "Инородное тело", 1965-1994
   &nbsp;
   "Суп, ПРОПУСК, лапшу прошу подавать мне не в тарелочках: предпочитаю
в мисочках".
   И.А. Бунин "Чаша жизни", 1913
   </раздатка>
   Уважаемые знатоки! Внимание, вопрос!
   Догадавшись, какие однокоренные слова мы пропустили в цитатах,
назовите растение, одна из разновидностей которого появилась в конце
40-х годов XX века как высокоурожайное кормовое растение, однако из-за
его токсичности и способности дичать и внедряться в местную флору от его
выращивания в промышленных масштабах отказались.

Ответ:
Борщевик.

Комментарий:
Мы пропустили в цитатах слова "переборщить" и "борщ". Упомянутая в
вопросе разновидность растения - борщевик Сосновского.

Источник:
   1. http://ru.wiktionary.org/wiki/переборщить
   2. http://ru.wiktionary.org/wiki/борщ
   3. http://ru.wikipedia.org/wiki/Борщевик

Автор:
Дарья Коротина

Вопрос 24:
Этот термин многозначный. Толковый словарь Ушакова дает две группы
значений: в первую входят обозначения людей, имеющих определенные
статусы в обществе, во вторую - названия различных технических
приспособлений. Предмет, именуемый данным термином, является валютой в
послевоенном мире, описанном современным российским автором. Назовите
этот термин.

Ответ:
Патрон.

Комментарий:
Значения термина "патрон", входящие в первую группу, - это, например, "В
древнем Риме - полноправный гражданин, считавшийся покровителем и
защитником своих вольноотпущенников и клиентов"; "Глава, хозяин
предприятия". Значения, входящие во вторую группу: "Часть электрического
осветительного прибора, в которую ввинчивается лампочка для
присоединения ее к сети"; "В токарных и сверлильных станках -
приспособление, служащее для укрепления обрабатываемого предмета (или же
инструмента) во вращающейся части станка". Упомянутый в вопросе
современный российский автор - это Дмитрий Глуховский. Послевоенный мир
он описывает в романах "Метро 2033", "Метро 2034", "Метро 2035".

Источник:
   1. http://dic.academic.ru/dic.nsf/ushakov/922306/
   2. http://ru.wikipedia.org/wiki/Метро_2033

Автор:
Дарья Коротина

Тур:
12 тур. "Непотерянное поколение" (Москва)

Редактор:
Данила Аладин (Пущино)

Вопрос 1:
И для разминки сразу блиц.
   На сайте немецкого журнала "Der Spiegel" есть открытая
онлайн-библиотека классической литературы под названием "Gutenberg".
Есть там и переводы русской классической поэзии. Поскольку это перевод,
он не всегда дословно повторяет написанное, и тем труднее перевести его
обратно на русский с немецкого. Я зачитаю вам три начальных фрагмента
известных произведений русской классической поэзии, переведенные обратно
на русский язык с немецкого, а вам нужно написать названия произведений.
   1. Начнем с простого. "Жили однажды пожилой мужчина и пожилая женщина
на пустынном пляже".
   2. Усложним немного. "Засиделись допоздна три девушки. Мурлыкая,
вертелись колеса их прялочек".
   3. "Царица царя провожала. Путешествие было долгим. И царица очень
грустила, ожидая царя возвращения".

Ответ:
   1. "Сказка о рыбаке и рыбке".
   2. "Сказка о царе Салтане".
   3. "Сказка о мертвой царевне и о семи богатырях".

Комментарий:
   1. "Жил старик со своею старухой у самого синего моря".
   2. "Три девицы под окном пряли поздно вечерком".
   3. "Царь с царицею простился, в путь-дорогу снарядился, и царица у
окна села ждать его одна".

Источник:
   1. http://gutenberg.spiegel.de/buch/das-marchen-vom-fischer-und-dem-fischlein-7593/1
   2. http://gutenberg.spiegel.de/buch/marchen-vom-zaren-saltan-7590/1
   3. http://gutenberg.spiegel.de/buch/marchen-von-der-toten-zarentochter-und-den-sieben-recken-7592/1

Автор:
Артем Демченков

Вопрос 2:
Фрэнк Синатра утверждал, что лучшее время для АЛЬФЫ - за две минуты до
смерти. "АЛЬФА" была впервые поставлена в 1842 году в Александринском
театре. Назовите АЛЬФУ.

Ответ:
Женитьба. Незачет: Свадьба.

Источник:
   1. http://lenta.ru/photo/2015/12/12/sinatra/#9
   2. http://ru.wikipedia.org/wiki/Женитьба_(пьеса)

Автор:
Роман Акасов

Вопрос 3:
Внимание, в вопросе есть замена.
   В романе Мориса Дрюона французский барон издевательски говорит, что
после двух поражений английский король может увидеть бОльшую часть своих
французских владений только во время мочеиспускания. Какое слово мы
заменили в этом вопросе?

Ответ:
Отлива.

Комментарий:
От владений английского короля во Франции, по словам барона, осталась
только узкая полоса вдоль берега моря. При отливе она становится гораздо
больше.

Источник:
М. Дрюон. Французская волчица. http://www.flibusta.is/b/159412/read

Автор:
Алексей Ярцхер

Вопрос 4:
Прослушайте стихотворение Кристины Стрельниковой:
   - Серая мышь, куда ты спешишь?
   - К тетушке кошке, - ответила мышь, -
   Тетушка кошка такая [ПРОПУСК 1].
   Не понимает, что я [ПРОПУСК 2].
   Второй пропуск входит в название культового фильма 1971 года.
Назовите режиссера этого фильма.

Ответ:
[Стэнли] Кубрик.

Комментарий:
Фильм "Заводной апельсин", мышь заводная, а кошка не тупая, что сразу
приходит на ум, а смешная.

Источник:
   1. Кристина Стрельникова "Серая мышь".
   2. http://www.kinopoisk.ru/film/391/

Автор:
Наталья Кузьмина

Вопрос 5:
   <раздатка>
   Swat, smile and repeat.
   </раздатка>
   в розданной вам фразе мы пропустили одну букву. В каком заведении
автор вопроса видела эту надпись?

Ответ:
Спортивный зал.

Зачет:
Фитнес-клуб и т.п.

Комментарий:
Пропущена буква "e" в слове "sweat". Sweat, smile and repeat. Потей,
улыбайся, повторяй.

Источник:
Личный опыт автора вопроса.

Автор:
Алиса Канарская

Вопрос 6:
[Ведущему: "делать это", "не делать ничего" и "делать что-то" должны
быть прочитаны с одинаковой интонацией без выделения.]
   Внимание, в вопросе есть замены.
   В старину люди, которые ДЕЛАЛИ ТО или ДЕЛАЛИ ЭТО, на самом деле
делали что-то. Теперь же ДЕЛАТЬ ТО или ДЕЛАТЬ ЭТО означает "не делать
ничего". Что мы заменили на ДЕЛАТЬ ЭТО или ДЕЛАТЬ ТО?

Ответ:
Точить лясы.

Зачет:
Бить баклуши.

Комментарий:
"Точить лясы" - изготавливать балясины, деревянные распорки для перил. А
"бить баклуши" - изготавливать заготовки для вырезания деревянных ложек.

Источник:
   1. http://ru.wiktionary.org/wiki/точить_лясы
   2. http://ru.wiktionary.org/wiki/бить_баклуши

Автор:
Наталья Кузьмина, Артем Демченков

Вопрос 7:
Прослушайте стихотворение Саши Черного:
   Это было в провинции, в страшной глуши.
   Я имел [ДВА СЛОВА ПРОПУЩЕНЫ]
   Дантистку с телом белее известки и мела,
   А [ДВА СЛОВА ПРОПУЩЕНЫ] -
   Модистку с удивительно нежной душой.
   Заполните пропуски.

Ответ:
"... для души...", "... для тела...".

Источник:
Саша Черный. Ошибка. http://www.stihi-rus.ru/1/chernyiy/51.htm

Автор:
Артем Демченков

Вопрос 8:
После просмотра фильмов "Лес смерти" или "Нападение помидоров-убийц" у
вас может возникнуть некий страх. Этот страх можно назвать неологизмом,
в котором на одну букву меньше, чем в слове, которым ранее в психиатрии
обозначали страх монотонности и однообразности. Напишите этот неологизм.

Ответ:
ГМОфобия.

Источник:
Личный опыт автора вопроса.

Автор:
Иван Беляев, Александр Бречалов

Вопрос 9:
Закончите советский анекдот: "Сталин о Пушкине: "Если бы товарищ Пушкин
жил не в девятнадцатом веке, а в двадцатом, он всё равно бы..."".

Ответ:
"... умер в 37-м".

Зачет:
По смыслу.

Источник:
Ю.Б. Борев. Сталиниада. http://www.flibusta.is/b/166012/read

Автор:
Артем Демченков

Вопрос 10:
Главная героиня фильма "Эльвира Мадиган" - циркачка, танцующая на
канате. Она находится в бегах и продает свой портрет, написанный
каким-то калекой в одном из парижских кафе. Процентщик недорого
оценивает рисунок и спрашивает: "Что это за буквы здесь в углу?".
Напишите эти две буквы.

Ответ:
ТЛ.

Зачет:
TL.

Комментарий:
Режиссер фильма намекает, что рисунок на самом деле стоит намного дороже
цены, которую за него отдал процентщик. TL в кружочке - подпись
художника Тулуза-Лотрека, который, как известно, в юношестве сломал обе
ноги, и они перестали расти. А циркачка, мало того что танцовщица, так
еще и в бегах.

Источник:
Х/ф "Эльвира Мадиган" (1967), реж. Бу Видерберг.

Автор:
Александр Бречалов

Вопрос 11:
Внимание, в вопросе есть замена.
   Прослушайте отрывок из поэмы "Мертвые души": "Счастлив путник,
который после длинной, скучной дороги с ее холодами, слякотью, грязью,
невыспавшимися гробовщиками, бряканьями колокольчиков, починками,
перебранками, ямщиками, кузнецами и всякого рода дорожными подлецами
видит, наконец, знакомую крышу...". Какие два слова мы заменили на одно?

Ответ:
Станционными смотрителями.

Комментарий:
Замена станционных смотрителей на гробовщиков - отсылка к пушкинским
повестям Белкина.

Источник:
Н.В. Гоголь. Мертвые души.
http://www.feb-web.ru/feb/gogol/texts/ps0/ps6/ps6-005-.htm

Автор:
Наталья Кузьмина

Вопрос 12:
В одном из предсезонных хоккейных матчей 2014 года между командами
"Тамба-Бэй Лайтнинг" и "Калгари Флэймз" произошел вполне обычный для
этого вида спорта эпизод. В результате столкновения между вратарем
"Тампы" Беном Бишопом и нападающим "Калгари" Корбаном Найтом первый
получил травму и покинул площадку, а второй был удален со льда. А каким
словом мог бы назвать эту ситуацию однофамилец человека, который занял
место в воротах "Тампы"?

Ответ:
Размен.

Зачет:
Обмен.

Комментарий:
Место в воротах "Тампы" занял Евгений Набоков, но знать это не
обязательно, т.к. Бишоп - это по-английски шахматный слон, а Найт - это
конь. Размен слона на коня является вполне обычной для этого вида спорта
комбинацией. Про писателя Владимира Набокова и шахматы можно и не
упоминать.

Источник:
   1. https://www.nhl.com/player/corban-knight-8475260
   2. https://www.nhl.com/player/ben-bishop-8471750
   3. http://ru.wikipedia.org/wiki/Набоков,_Евгений_Викторович

Автор:
Артем Демченков

Вопрос 13:
Дуплет.
   1. Внимание, в вопросе есть замены.
   Исследователь петербургского городского фольклора историк Наум
Синдаловский упоминает поговорку, возникшую в XIX веке: "ПЕРВЫЙ ВТОРОГО
догоняет, да ТРЕТИЙ мешает". Какие имена мы заменили на "ПЕРВЫЙ",
"ВТОРОЙ" и "ТРЕТИЙ"?
   2. В вопросе есть замены.
   Всё тот же исследователь петербургского городского фольклора Наум
Синдаловский рассказывает, как один известный своими, в том числе
московскими, работами скульптор предложил свои услуги Петербургу. В
результате возникла поговорка: мол, еще не хватало поменять ПЕРВОГО на
ВТОРОГО. Какие фамилии мы заменили на "ПЕРВЫЙ" и "ВТОРОЙ"?

Ответ:
   1. Коля, Петя, Исаакий.
   2. Растрелли, Церетели.

Зачет:
Николай, Петр, Исаакий.

Комментарий:
   1. "Коля Петю догоняет, да Исаакий мешает". Речь о конных памятниках
Николаю I, Петру I и Исаакиевском соборе, расположенном между ними.

Источник:
Н.А. Синдаловский. Очерки петербургской мифологии, или Мы и городской
фольклор. http://www.flibusta.is/b/377727/read

Автор:
Артем Демченков

Вопрос 14:
В рассказе Синдзюро Тобэ говорится о школе фехтования, прославившейся
неким секретным приемом под названием "Клыки дракона". Этот прием
позволял ненадолго задержать меч противника и нанести ему ответный удар,
но использовать его рекомендовалось только в случае настоящей опасности.
В конце рассказа, спасая своего господина, самурай применяет этот прием,
и вокруг него на земле оказываются разбросаны те самые "клыки дракона".
Так что же это на самом деле?

Ответ:
Пальцы.

Комментарий:
Прием состоял в том, чтобы схватить лезвие меча противника одной рукой и
нанести ему смертельный удар другой. Поскольку пальцы почти всегда
оказывались отрубленными, использовать "клыки дракона" рекомендовалось
только в безвыходном положении.

Источник:
Синдзюро Тобэ. Клыки дракона. http://www.flibusta.is/b/234931/read

Автор:
Артем Демченков

Вопрос 15:
Кем приходился персонаж "Анны Карениной" Стива Облонский Константину
Лёвину?

Ответ:
Свояк.

Комментарий:
Свояк - муж сестры жены, но для ответа на вопрос этого знать не нужно,
да и "Анну Каренину" читать не обязательно.

Источник:
   1. https://commons.wikimedia.org/wiki/File:Anna_Karenina_family_tree_RU_(ill).jpg
   2. http://ru.wikipedia.org/wiki/Родство

Автор:
Роман Акасов

Вопрос 16:
В 1951 году молодой сотрудник компании "General Electric" Курт Воннегут
получил письмо, содержавшее помимо адреса отправителя и его подписи
всего одно слово - "Fuck". Мы точно не знаем, сколько таких писем
получил Воннегут, но в том же году он покинул "General Electric". А кто
был отправителем этого короткого, но емкого письма?

Ответ:
Издатель.

Комментарий:
Письмо, полученное Воннегутом, - это отказ в публикации одного из его
рассказов. Точно не известно, сколько отказов он получил, но, после того
как пара рассказов всё же была опубликована, он покинул "General
Electric", чтобы сосредоточиться на писательской деятельности.

Источник:
   1. http://i.onionstatic.com/clickhole/1942/8/original/1280.jpg
   2. http://en.wikipedia.org/wiki/Kurt_Vonnegut

Автор:
Артем Демченков

Вопрос 17:
"Катон отдал приказ иерарху вести "Себек" к входу в бухту на полной
скорости. Барабан под палубой отбивал ритм гребцам, весла опускались и
подымались, разгоняя корабль". Какие две буквы мы пропустили в этом
тексте?

Ответ:
тр.

Комментарий:
По имени Катон можно догадаться, что речь идет о римлянах. Упомянутый
корабль - это римская триера, управлял которой не иерарх, а триерарх.

Источник:
С. Скэрроу. Легион смертников. http://www.flibusta.is/b/351234/read

Автор:
Роман Акасов

Вопрос 18:
Во времена Холодной войны в Голливуде было снято немало фильмов,
действие которых полностью или частично происходит в СССР. Например,
"Парк Горького", "Телефон", "Белые ночи". А в каком городе снималось
большинство "советских" сцен этих фильмов?

Ответ:
Хельсинки.

Комментарий:
Понятно, что снимать в самом Советском Союзе во времена Холодной войны
было невозможно. Города, находящиеся в других странах Варшавского
договора (например, Варшава) тоже не подходили. Зато идеально подходила
столица Финляндии, до 1917 года входившая в состав Российской империи и
сохранившая множество построек того времени. Название фильма "Белые
ночи" также является подсказкой.

Источник:
   1. http://ru.wikipedia.org/wiki/Парк_Горького_(фильм)
   2. http://ru.wikipedia.org/wiki/Телефон_(фильм)
   3. http://ru.wikipedia.org/wiki/Белые_ночи_(фильм,_1985)

Автор:
Артем Демченков

Вопрос 19:
В одной советской песне урожай назван ТАКИМ. Современные слушатели могут
подумать, что он уже гарантированный, но он всего лишь большой. Какое
слово мы заменили на "ТАКОЙ"?

Ответ:
Стопудовый.

Источник:
http://www.sovmusic.ru/text.php?fname=stopudov

Автор:
Роман Акасов

Вопрос 20:
В 2004 году в "Кремниевой Долине" была основана компания "Palantir",
специализирующаяся на анализе большого объема данных. Отличительной
особенностью продуктов компании является совместное принятие решений с
использованием человеческих и машинных ресурсов. В своих презентациях
представители "Palantir" часто ссылаются на известное противостояние
1997 года и последовавший за ним скандал. Назовите обоих участников
этого противостояния.

Ответ:
[Гарри] Каспаров и "Deep Blue".

Зачет:
[Гарри] Каспаров и "IBM".

Комментарий:
После шестой партии матча против компьютера "Deep Blue" компании "IBM"
Гарри Каспаров обвинил противника в мошенничестве, заявив, что в
решающий момент компьютер сделал не свойственный машине ход, а значит,
ему помог профессиональный шахматист. Каспаров потребовал реванша, но
"IBM" не только отказала ему в этом, но и свернула проект "Deep Blue".
На том же принципе помощи человека машине основана работа продуктов
"Palantir".

Источник:
   1. https://www.palantir.com/2012/07/shyam-sankar-speaks-at-tedglobal-2012/
   2. http://www.youtube.com/watch?v=f86VKjFSMJE
   3. http://en.wikipedia.org/wiki/Deep_Blue_versus_Kasparov,_1997,_Game_6

Автор:
Артем Демченков

Вопрос 21:
(pic: 20160316.jpg)
   Перед вами фрагмент современной картины. Как называется эта картина?

Ответ:
"Money Lisa".

Зачет:
"Мани Лиза".

Комментарий:
(pic: 20160317.jpg)
   А на розданном фрагменте ее знаменитая улыбка.

Источник:
http://pikabu.ru/story/money_lisa_3878186

Автор:
Роман Акасов

Вопрос 22:
Некоторые современные ученые полагают, что традиционные представления об
обещанной ИМ награде неверны и получат ОНИ лишь гроздь винограда. Трудно
поверить, но определенная группа людей, погибших в Великой Отечественной
войне, тоже считается ИМИ. Назовите ИХ.

Ответ:
Шахиды.

Комментарий:
Шахид - это мученик за веру. Мусульмане, погибшие в Великой
Отечественной войне, в России считаются шахидами. Официально считается,
что после смерти шахидов ждет общество райских дев гурий. Однако
некоторые ученые говорят о неправильном переводе строк Корана, и слово
"хур", означает всего лишь "белый виноград". Слова "трудно поверить" в
тексте вопроса также являются подсказкой.

Источник:
   1. http://www.pravda.ru/faith/religions/islam/18-08-2004/48135-koran-0/
   2. http://ria.ru/religion/20100622/248984286.html

Автор:
Роман Акасов, Артем Демченков

Вопрос 23:
Не переживайте, осталось два вопроса.
   Происхождение переносного значения одного русского глагола точно не
известно. Но по одной из версий, оно появилось в результате неточного
перевода английского глагола, обозначающего процесс пришвартовывания
корабля к причалу, когда канаты, сбрасываемые с судна на берег,
обматывались вокруг деревянных столбов, чтобы закрепить корабль.
Назовите этот русский глагол.

Ответ:
Закругляться.

Комментарий:
Английский глагол - "wind up". А так как этот вопрос был предпоследним,
нам тоже пора закругляться.

Источник:
http://lenina51.mypage.ru/proishojdenie_slov.html

Автор:
Артем Демченков

Вопрос 24:
10 мая 1923 года в Лозанне в ресторане отеля "Сесиль" был убит советский
дипломат Вацлав Воровский. На ужин он заказал салат, осетрину, кекс,
кофе со сливками и яблоки в красном вине. И уже сидя за столом, получил
пулю в затылок. Убийца - бывший белогвардейский офицер по имени Морис
КонрАди - был признан виновным, но оправдан судом присяжных, как и его
сообщник Аркадий Полунин. После этого в одной из советских газет
появилась такая эпитафия:
   Своих вожделений не скрыли
   В потоке словесной воды,
   Воровскому в сердце вонзили
   Свинцовую...
   Закончите стихотворение тремя словами.

Ответ:
"... точку над i".

Комментарий:
Словесной воды в этом вопросе действительно много, а вот точки над i как
раз и не хватало. И этим вопросом мы ставим точку в нашем пакете.

Источник:
http://tvkultura.ru/video/show/brand_id/20907/episode_id/1254141/

Автор:
Артем Демченков

Тур:
13 тур. "Ветряные мельницы вероятного противника" (Москва)

Редактор:
Александр Берелехис (Уфа)

Вопрос 1:
На соревнованиях "Бегущий город. Москва - 2012" автор вопроса был судьей
на контрольном пункте. На этом пункте участникам выдавалась аудиозагадка
- по "минусовке" требовалось угадать песню. Минусовка звучала непрерывно
в течение многих часов. Многие участники задавали судьям один и тот же
вопрос. Мы не просим воспроизвести этот вопрос. Воспроизведите хотя бы
приблизительно ответ, который судьи давали на него.

Ответ:
"Нам надоело отвечать на этот вопрос".

Зачет:
"Надоело отвечать на такие вопросы", "Надоели такие вопросы" и т.п.

Комментарий:
Мы надеемся, что впечатление от наших вопросов не будет похожим на ответ
на данный вопрос.

Источник:
ЛОАВ.

Вопрос 2:
ИХ появление, главным образом, было спровоцировано развитием железных
дорог, так как привычное на тот момент ориентирование по Солнцу было
чревато опасной путаницей. У вас есть ровно одна минута, чтобы назвать
ИХ двумя словами.

Ответ:
Часовые пояса.

Комментарий:
До введения системы часовых поясов в каждом населенном пункте
использовалось свое местное солнечное время, что создавало полную
неразбериху в расписании поездов. Тогда Британское правительство первым
решилось унифицировать время на территории всей страны - так появилось
среднее время по Гринвичу.

Источник:
http://ru.wikipedia.org/wiki/Часовой_пояс

Вопрос 3:
Если бы объявление о продаже ИКСА увидел, скажем, современник Пушкина,
он подумал бы, что продается человек. Если в поисковике задать запрос
"продам ИКСА", то, в основном, будут выданы объявления о продаже
виртуального товара. На известном сайте частных объявлений продажа
виртуального товара запрещена, а объявлений о продаже ИКСОВ
предостаточно. Что мы заменили на ИКС?

Ответ:
Перс.

Источник:
   1. http://ru.wikipedia.org/wiki/Персы
   2. https://www.google.ru/#newwindow=1&q=продам+перса
   3. https://www.avito.ru/moskva?bt=0&q=перс

Вопрос 4:
Дуплет.
   1. В середине 1990-х годов в Петербурге родилась новая традиция.
Ежегодно 27 мая, в день основания города, проводится "День Петровичей на
Петровской набережной у Домика Петра". С единственным, одним на всех
лозунгом. В качестве этого лозунга взято восклицательное предложение из
четырех слов, представляющее собой небольшую переделку цитаты из А.С.
Пушкина. Воспроизведите этот лозунг.
   2. Согласно фольклору, упомянутая в первом вопросе цитата посвящена
А.С. Пушкиным человеку. Назовите фамилию этого человека.

Ответ:
   1. Любите нас, Петров творенья!
   2. Керн.

Комментарий:
   2. Она ведь Анна Петровна.

Источник:
   1. Н.А. Синдаловский. Словарь петербуржца. Лексикон Северной столицы.
История и современность. http://www.flibusta.is/b/373033/read
   2. Н.А. Синдаловский. Пушкинский круг. Легенды и мифы.
http://www.flibusta.is/b/332391/read

Вопрос 5:
Этому голландскому продукту, который нынче не купить в России, реклама
была не нужна. Но если бы ее захотели сделать, то слоган можно было бы
заимствовать из рекламы продукта, производство которого было налажено в
ЮАР, а потом и в России, выпускниками МХТИ имени Д.И. Менделеева,
добавлявшегося к голландскому продукту отдельными потребителями. Мы не
просим воспроизвести этот слоган. Назовите продукт, выпускавшийся в ЮАР.

Ответ:
Инвайт.

Зачет:
Invite; Инвайт+; Invite+.

Комментарий:
Спирт "Royal" в рекламе не нуждался, но слоган "Просто добавь воды" из
рекламы "Инвайт" ему бы вполне подошел. А некоторые любители выпить при
разведении спирта "Royal" добавляли "Инвайт" для вкуса и цвета.

Источник:
   1. http://www.kushvablog.ru/pomhim/523-yepoxa-spirta-royal.html
   2. http://www.compromat.ru/page_22978.htm
   3. http://www.old-things.org/invite-napitok/
   4. http://www.ko.ru/articles/581
   5. http://www.youtube.com/watch?v=SWe0hTVoIyA
   6. http://foto-history.livejournal.com/8602004.html?thread=136243348#t136243348

Вопрос 6:
На первый взгляд этот объект в некотором роде совмещает в себе
божественное и дьявольское начало. Однако на самом деле его всемирно
известная божественная часть получила наиболее известное из своих
названий в честь простого смертного. Впрочем, в определенном смысле у
него действительно были крылья. Назовите божественную часть.

Ответ:
Анхель.

Зачет:
Водопад Анхель.

Комментарий:
Самый высокий в мире водопад Анхель в Венесуэле, он же Angel Falls,
низвергается с вершины горы Ауянтепуи, что в переводе означает "Дом
дьявола". Свое название водопад получил отнюдь не в честь ангелов, как
можно было бы предположить, будучи знакомым с набожностью
латиноамериканцев, а по имени посетившего его первым американского
летчика Джеймса Энджела. Несмотря на то что в 2009 году Уго Чавес
переименовал водопад в Kerepakupai Meru, это название используется
только в Венесуэле.

Источник:
   1. http://www.britannica.com/place/Angel-Falls
   2. http://ru.wikipedia.org/wiki/Анхель

Вопрос 7:
В романе Виктора Пелевина "Смотритель" главный герой видит по телевизору
то, что кажется ему "странным и, верно, жестоким ритуалом": "человек в
жуткой белой маске и латах, скрытых под просторной рубахой, отбивался
кривой деревянной саблей от таких же нелепых гладиаторов, пытающихся
загнать его в стоящую на льду клетку из сети, растянутой на железной
раме". Что на самом деле показывают по телевизору?

Ответ:
Хоккей.

Зачет:
Хоккейный матч.

Источник:
В.О. Пелевин. Смотритель. Книга 2. Железная бездна.
https://books.google.ru/books?id=yKFyCwAAQBAJ&pg=PT126#v=onepage&q&f=false

Вопрос 8:
(pic: 20160318.jpg)
   Перед вами комикс, две из шести картинок которого заменены
вопросительными знаками. На последней картинке в верхнем ряду можно
видеть, среди прочего, руку и полку. Ответьте по возможности точнее, что
можно увидеть на первой картинке в нижнем ряду.

Ответ:
Грампластинку в проигрывателе.

Зачет:
Играющую пластинку и т.п. Незачет: Упоминание другого носителя или
упоминание пластинки без намека на воспроизведение.

Комментарий:
(pic: 20160319.jpg)
   Прическа меломана напоминает грампластинку, а на то, что он ее
слушает, а не просто рассматривает, намекают ноты на последних двух
картинках.

Источник:
http://phonautograph.tumblr.com/post/35345351735/vinylespassion-lane-nelson-long-days

Вопрос 9:
Закончите четверостишие поэта Николая Федоровича Щербины из
стихотворения "Русская история (Посвящается отечественным
государственным людям)" (1859) двумя словами, начинающимися на одну и ту
же букву:
   Одно мы пред судом народов
   Собой способны доказать,
   Что может шайка идиотов
   Народом...

Ответ:
"... умным управлять".

Источник:
http://az.lib.ru/s/sherbina_n_f/text_0022.shtml

Вопрос 10:
Нобелевскую премию 2015 года по экономике получил американец Энгус Дитон
с формулировкой "за аналитические исследования в области потребления,
бедности и благосостояния". За несколько лет до этого Энгус Дитон вместе
с психологом Даниелем Канеманом прославился забавным исследованием, во
время которого 450 тысячам американцам был задан фактически тот же
вопрос, который адресовался персонажу отечественного произведения 1931
года. Назовите этого персонажа.

Ответ:
[Шура] Балаганов.

Комментарий:
450 тысячам американцам был задан знаменитый вопрос Бендера Балаганову:
"Сколько вам нужно денег для счастья?". Оказалось, что нужно
зарабатывать 75 тысяч долларов в год. Но самое интересное: при
дальнейшем росте дохода уровень удовлетворенности жизнью уже не
меняется.

Источник:
   1. "Вокруг света", 2015, N 12 - С. 132.
   2. И. Ильф, Е. Петров. Золотой теленок.
http://az.lib.ru/i/ilfpetrov/text_0130.shtml

Вопрос 11:
В московском особняке Павла Воиновича Нащокина был "нащокинский домик" -
двухэтажный макет богатого дворянского особняка в футляре из красного
дерева с раздвижными стеклами, размером 2,5 на 2 метра. Среди прочего в
этом домике был рояль красного дерева, на котором жена Павла Воиновича -
Вера Александровна - даже играла. Чем она играла на этом рояле?

Ответ:
[Вязальными] спицами.

Источник:
http://www.den-za-dnem.ru/page.php?article=64

Вопрос 12:
С 1959 по 1975 год в Москве эксплуатировались сочлененные троллейбусы
моделей СВАРЗ ТС-1 и СВАРЗ ТС-2. Один такой троллейбус мог увезти 220
пассажиров (а по факту бывало и больше). Пассажиры прозвали эти
троллейбусы ИКСАМИ. Один из видов ИКСА, патент на который был получен
англичанином Хьюбером Сесилом Бутом в 1901 году, пугал лошадей, и
поэтому его использование на улице было запрещено. Что мы заменили на
ИКС?

Ответ:
Пылесос.

Комментарий:
Упомянутые троллейбусы, как правило, забирали с остановки всех желающих
уехать, за что и получили такое прозвище. "Фырчащий Билли" - пылесос с
бензиновым мотором, работавшим на улице, и 30-метровым шлангом,
заводимым в помещения через окна, - своим шумом пугал лошадей.

Источник:
   1. http://trolley.ruz.net/trollcars/svarz-ts/
   2. http://ru.wikipedia.org/wiki/Пылесос
   3. http://newspaper.moe-online.ru/view/16420.html

Вопрос 13:
Когда автор вопроса, собирая домашнюю стереосистему, установил
CD-проигрыватель на усилитель, он заметил, что прослойка воздуха
недостаточна, и CD-проигрыватель сильно нагревается от исходящего от
усилителя горячего воздуха. В специализированных магазинах предлагались
специальные ножки, однако дешевых в наличии не было, а четыре дорогих
стоили столько, сколько стоил сам усилитель, и по совету с одного из
форумов автор вопроса отправился в спортивный магазин, где купил АЛЬФЫ.
"АЛЬФА" стала предметом внезапной любви пожилого болельщика. Какое слово
мы заменили на АЛЬФУ?

Ответ:
Шайба.

Комментарий:
Автор вопроса использовал детские хоккейные шайбы. А речь идет о любви
старика Хоттабыча к команде "Шайба".

Источник:
   1. http://forums.overclockers.ru/viewtopic.php?f=33&t=356429&p=9416892
   2. ЛОАВ.
   3. Л. Лагин. Старик Хоттабыч. http://www.flibusta.is/b/140813/read

Вопрос 14:
(pic: 20160320.jpg)
   Перед вами герб московского района Очаково-Матвеевское. Ответьте
тремя словами, начинающимися на одну и ту же букву: что из находящегося
в этом районе символизирует рог изобилия?

Ответ:
Предприятия пищевой промышленности.

Источник:
   1. https://ochakovo.mos.ru/about/gerb_rayona/
   2. http://www.heraldicum.ru/russia/subjects/towns/m_rayo16.htm#ocakov

Вопрос 15:
(pic: 20160321.jpg)
   Мы не спрашиваем имени и даже фамилии, которые мы стерли. Ответьте,
что изобрел человек, изображенный на третьем рисунке.

Ответ:
Смайл.

Зачет:
Смайлик; эмотикон.

Источник:
   1. http://pikabu.ru/story/izvestnyie_izobretateli_1214216
   2. http://ru.wikipedia.org/wiki/Фалман,_Скотт

Вопрос 16:
Герой этой поэмы мечтал о репатриации, но потерпел поражение. К тому же,
когда он шел к своей цели, он совершил действие, которое вызвало бы
явное неодобрение у представителей Всемирного фонда дикой природы, если
бы такой в то время существовал. Как называется эта поэма?

Ответ:
"Мцыри".

Комментарий:
Герой вступил в схватку с барсом и убил его.

Источник:
http://ru.wikisource.org/wiki/Мцыри_(Лермонтов)

Вопрос 17:
Дуплет.
   В сообществе "Скверные гимназисты" тексты известных песен
переделывают на манер отрывков русской классической литературы.
   1. Часть одной из песен переделана следующим образом:
   "- Вот-с, полюбуйтесь: границы нашей Империи надежно закрыты на
замке.
   - Ха! Коли уж так, Модест Георгиевич, то ключ от сего замка явно
поломан и выброшен".
   "- К слову, я полагаю дальновидным правителем лишь покойного государя
Николай Павловича. И только его, да-с. Что же касается прочих - сплошь
самодуры да глупцы".
   Рефреном в этом разговоре звучат такие слова:
   "- На всё воля божья, Карл Петрович.
   - На всё воля божья, Модест Георгиевич".
   А какой припев (совпадающий с названием) у этой песни в оригинале?
   2. А это часть другой переделанной песни:
   "Милостивый государь! Извольте наполнить кубок свой божественным
горячительным и опрокинуть его за мое здравие хотя бы и с самим
Нечистым!
   Трактирщик! Графинчик "Столичной" вон тому буйному господину, чтоб
сами небеса пали навзничь!
   Запускаем огни, господа мои, - и пусть напиток сей послужит порохом,
ибо он - орудие самого Нечистого".
   Воспроизведите оригинальное название этой песни.

Ответ:
   1. "Всё идет по плану".
   2. "Russian vodka".

Источник:
   1. https://vk.com/badschoolboys?w=wall-78825554_489
   2. https://vk.com/badschoolboys?w=wall-78825554_926

Вопрос 18:
Ветераны стройки Рублёвской водопроводной станции, построенной в 1903
году, вспоминали: "При забивке свай использовали копёр с чугунной бабой
весом чуть ли не в полтонны. Такую бабу за канаты поднимали человек
тридцать под "Дубинушка, ухнем!". На такую тяжелую работу набирали
работников на Хитровке в Москве, народ пройдохистый, могли сваю забить
не до конца, а верхушку сваи отпилить. Если мастер зазевается, рабочие
могли сдать такую сваю как полностью забитую". Чтобы этого не было,
ввели определенный порядок. Ответьте как можно точнее, что делал мастер,
чтобы рабочие забили все сваи до конца.

Ответ:
Расписывался на верху каждой сваи.

Зачет:
По словам "расписывался на верху" или "ставил подпись на верху".

Источник:
М.В. Данченко. Наследие Кунцева. - М., 2010. - С. 150.

Вопрос 19:
По легенде, Наполеон, оценив красоту храма Успения Пресвятой Богородицы
в Котельниках, назвал его ТАКИМ ИКСОМ и выставил охрану от пожара, хотя
и не от почтения к русской старине, а из намерения разобрать по
кирпичику и перевезти в Париж. Назовите фамилию ТАКОГО поэта, который
через 100 лет написал стихотворение под названием "ИКС".

Ответ:
Мандельштам.

Комментарий:
ТАКОЙ ИКС - Русский Notre Dame.

Источник:
   1. http://www.vesti.ru/doc.html?id=2158868
   2. http://ru.wikipedia.org/wiki/Notre_Dame_(стихотворение)

Вопрос 20:
Комментируя новость о запуске автобусного движения по улице Красная
Сосна, один из участников Форума Общественного Транспорта написал, что
автобусы ДЕЛАЮТ ЭТО. ЭТО СДЕЛАЛ и герой детской песенки. Какая с ним
случилась беда?

Ответ:
Перепутал провода.

Комментарий:
Пользователь написал: "Автобусы залезают на Сосну". А мы с детства
помним слова из песенки Кабалевского: "На сосну медведь полез, /
Осветить задумал лес. / Но случилась тут беда - / Перепутал провода...":
   (aud: 20160002.mp3)

Источник:
   1. http://fot.com.ru/index.php/topic/22373-автобусы-залезают-на-сосну
   2. http://web.archive.org/web/20080623132639/www.tr.ru/news/3811
   3. http://sc384.spb.ru/montr/

Вопрос 21:
При исследовании новгородских берестяных табличек ученый Андрей Зализняк
столкнулся с трудностью перевода на современный русский язык слов "А
замке келиа, двери келиа господарь". Благодаря этой фразе ему удалось
установить, что древненовгородский диалект обособился от "соседних"
диалектов довольно рано. Как переводится слово "келиа" на современный
русский язык?

Ответ:
Целы.

Зачет:
Цел; целые.

Комментарий:
В европейских языках в разное время произошла вторая палатализация,
фонетическое изменение, в результате которого в определенных позициях
"к" была заменена на "ц". Читая эту грамоту, ученые впервые столкнулись
с русским текстом, в котором палатализация еще не случилась.

Источник:
http://tvkultura.ru/video/show/brand_id/20898/episode_id/156387/

Вопрос 22:
В романе Ольги Лукас "Бульон терзаний" один из героев восклицает про
себя: "Гоголь мечтает поставить Пелевина. Полный [ПРОПУСК]!". На месте
пропуска - фамилия известного зарубежного писателя, который жил позже
Гоголя, но раньше Пелевина. О каком писателе идет речь?

Ответ:
Кафка.

Комментарий:
"Гоголь мечтает поставить Пелевина. Полный Кафка!". Кафка описывал
абсурдность бытия, и его фамилия давно ассоциируется с проявлениями
абсурда в реальности.

Источник:
О. Лукас. Бульон терзаний. http://www.flibusta.is/b/396623/read

Вопрос 23:
Дуплет.
   (pic: 20160322.jpg)
   1. На рисунке мы убрали портрет и заменили его вопросительным знаком.
Чей портрет мы убрали?
   (pic: 20160323.jpg)
   2. Американец Грант Снайдер писал: "Я провел несколько лет, поглощая
книги ИКСА. Двенадцать романов, три сборника рассказов, затем биография.
В результате возник этот комикс. Если вам только предстоит встреча с
книгами ИКСА, держите эти карты под рукой - при чтении пригодятся".
Назовите ИКСА, не забывая указать имя.

Ответ:
   1. Пушкин.
   2. Харуки Мураками.

Комментарий:
   1. Основные события биографии А.С. Пушкина представлены в виде
орнамента, выполненного в африканском стиле.

Источник:
   1. http://hiero.ru/2028300
   2. http://www.incidentalcomics.com/2012/06/haruki-murakami-bingo.html

Вопрос 24:
В популярнейшем советском многосерийном мультфильме один из главных
героев двадцать раз совершает действие, ныне регламентируемое
Федеральным Законом N 15-ФЗ от 23.02.2013 г. Раскрутите этот вопрос за
одну минуту и опишите это действие.

Ответ:
Курение [табака].

Комментарий:
В двадцати сериях мультфильма "Ну, погоди!" Волк показан курящим
двадцать раз. Мы надеемся, что вы раскурили этот вопрос, и напоминаем,
что курение во всех помещениях ДС "Ока", включая туалеты, запрещено!

Источник:
   1. http://ru.wikipedia.org/wiki/Ну,_погоди!#.D0.92.D0.BE.D0.BB.D0.BA_.D0.B8_.D1.82.D0.B0.D0.B1.D0.B0.D0.BA.D0.BE.D0.BA.D1.83.D1.80.D0.B5.D0.BD.D0.B8.D0.B5
   2. http://www.rg.ru/2013/02/26/zakon-dok.html

Тур:
14 тур. "Связь без брака" (Ярославль)

Инфо:
Команда благодарит за помощь в подготовке пакета Максима Мерзлякова
(Воронеж) и Наиля Фарукшина (Навои - Москва).

Вопрос 1:
(pic: 20160324.jpg)
   На закрашенном кадре ЭТО можно увидеть минимум один раз. Назовите ЭТО
тремя словами.

Ответ:
Печенье с предсказанием.

Комментарий:
(pic: 20160325.jpg)
   Комикс представляет из себя рекурсию: на бумажке с предсказанием
нарисован он сам. Разумеется, автор не смог нарисовать бесконечность
вложенных комиксов, но первое повторение хорошо различимо.

Источник:
http://comicsia.ru/collections/explosm/i11937.html

Автор:
Андрей Горошников (Ярославль)

Вопрос 2:
Анна Григорьевна Достоевская пишет, что, хотя ее отец и учился в школе
ИКСОВ, но ИКСОМ не сделался, а всю жизнь оставался добрым и простодушным
человеком". Какое слово мы заменили ИКСОМ?

Ответ:
Иезуит.

Источник:
А.Г. Достоевская. Воспоминания.
http://az.lib.ru/d/dostoewskaja_a_g/text_1916_vospominaniya.shtml

Автор:
Александр Судаков (Ярославль)

Вопрос 3:
В повести Эдуарда Успенского Шарик употребляет в отношении Матроскина
оскорбительное слово. Его значения кот понять не сумел, смог только
представить себе упакованных домашних птиц. Напишите это слово.

Ответ:
Куркуль.

Комментарий:
Цитата из повести: "Матроскин не знал, кто такой куркуль. Он только
сумел представить себе целый кулек куриц и больше ничего. Но он понял,
что это что-то очень обидное".

Источник:
Э.Н. Успенский. Тетя дяди Федора, или Побег из Простоквашино.
http://www.flibusta.is/b/281381/read

Автор:
Григорий Ананьин (Ярославль)

Вопрос 4:
Прослушайте цитату из одного романа: "Почему наш поэт решил дать своему
урагану <...> редко употребляемое испанское имя <...>, а не назвал его
Линда или Лоис, неясно". Назовите автора этого романа.

Ответ:
[Владимир] Набоков.

Комментарий:
Поэт назвал ураган "Лолита". Роман Набокова "Бледный огонь" представляет
собой поэму с комментариями к ней. Таким образом, Набоков делает
своеобразную ссылку второго уровня на собственное творчество.

Источник:
В.В. Набоков. Бледный огонь. http://www.flibusta.is/b/223051/read#k_680

Автор:
Александр Судаков (Ярославль)

Вопрос 5:
В одной притче рассказывается, что царь Соломон разрешил спор каменщика,
плотника и землекопа, напомнив, что каждому из них нужен ОН. Кто ОН?

Ответ:
Кузнец.

Комментарий:
Каменщик, плотник и землекоп спорили, кто больше всех сделал для
создания Иерусалимского храма. Форма вопросительной фразы содержит
аллюзию на известный эпизод из фильма "Формула любви". В отличие от
героев фильма, каменщику, плотнику и землекопу кузнец был очень даже
нужен.

Источник:
   1. С.И. Венецкий. В мире металлов.
http://www.flibusta.is/b/224963/read
   2. http://ru.wikiquote.org/wiki/Формула_любви

Автор:
Арсений Глазовский, по идее Юрия Чертова (оба - Ярославль)

Вопрос 6:
Про автомобиль Rolls-Royce Ghost [роллс ройс гОуст] говорили, что это
первый автомобиль марки, в котором водитель фактически поднимается до
статуса хозяина. Так, например, один из аксессуаров "переехал" из торца
задней двери в створ передней. Что это за аксессуар?

Ответ:
Зонт.

Комментарий:
Зонт входит в комплектацию автомобиля уже очень давно, однако раньше
удобство персонала никого не интересовало, поэтому водитель или
охранник, чтобы добраться до зонтика, спрятанного в задней дверце, был
вынужден нырять под дождь.

Источник:
http://www.avtovzglyad.ru/article/2013/08/16/609631-rollsroyce-ghost-siyatelnyiy-dinozavr.html

Автор:
Александр Судаков (Ярославль)

Вопрос 7:
В одном древнем трактате перечислены десять свойств драгоценных камней,
которые должен знать каждый: природа, окраска, блеск, форма, объем,
качество, месторождение, оттенки, цена и ОНИ. Слово "ОНИ" звучит в конце
известного фильма. Назовите ИХ.

Ответ:
Недостатки.

Комментарий:
Фразу "У каждого свои недостатки" можно услышать в конце фильма "В джазе
только девушки".

Источник:
   1. http://mindraw.web.ru/mineral_gems.htm
   2. http://ru.wikiquote.org/wiki/Некоторые_любят_погорячее

Автор:
Арсений Глазовский, по идее Юрия Чертова (оба - Ярославль)

Вопрос 8:
Согласно афоризму, ЕГО специфика в России состоит в том, что ВТОРЫМ тоже
бьют. Назовите ПЕРВЫЙ, а то не получите плюс.

Ответ:
Кнут.

Комментарий:
Имеется в виду метод кнута и пряника.

Источник:
http://bash.im/quote/417156/

Автор:
Андрей Горошников (Ярославль)

Вопрос 9:
Внимание, в вопросе есть замена.
   Политолог Николай СтАриков назвал первую русскую революцию
""ПРИВИВКОЙ" для взрослых". Какие два слова мы заменили словом
"ПРИВИВКА"?

Ответ:
"Праздник непослушания".

Комментарий:
Название повести-сказки Сергея Михалкова было заменено названием его же
детского стихотворения.

Источник:
   1. https://books.google.ru/books?id=Q685AgAAQBAJ&pg=PA222#v=onepage&q&f=false
   2. http://ru.wikipedia.org/wiki/Праздник_непослушания
   3. http://rupoem.ru/mixalkov/na-privivku-pervyj.aspx

Автор:
Владислав Шашкин (Ярославль)

Вопрос 10:
Персонаж Хайнлайна пытается разобраться в истории религии, для чего
предпринимает множество путешествий во времени, но неудачно.
Разочаровавшись, он называет один символ фикцией. Напишите пять букв,
которые мы пропустили в предыдущем предложении.

Ответ:
круци.

Зачет:
крузи.

Комментарий:
Персонаж не может подтвердить существование Иисуса и называет распятие
круцификцией, обыгрывая его латинское название "Crucifix" [круцификс].

Источник:
Р. Хайнлайн. Уплыть за закат. http://www.flibusta.is/b/22119/read

Автор:
Андрей Горошников (Ярославль)

Вопрос 11:
В одном из интервью ОН отметил, что ЕМУ сильно повезло, ведь теперь
деревья выращивают только до 24 дюймов в диаметре, а старые лесопилки
заменяют на современные, с маленькими станочками. Назовите ЕГО.

Ответ:
[Дэвид] Линч.

Комментарий:
Дэвид Линч рассказывает о титрах сериала "Твин Пикс". Через два месяца
после съемок старую лесопилку снесли и построили новую, с маленькими
шустрыми станочками, которые могут работать только с
двадцатичетырехдюймовым бревном.

Источник:
   1. Д. Линч. Интервью. Беседы с Крисом Родли.
http://www.flibusta.is/b/223951/read
   2. http://www.youtube.com/watch?v=IaX3nrWzKEM

Автор:
Александр Судаков (Ярославль)

Вопрос 12:
В 1998 году два американских конгрессмена-консерватора положили текст
Налогового кодекса США в ящик. А что на этом ящике было написано?

Ответ:
"Чай".

Комментарий:
Затем ящик в присутствии журналистов сбросили в залив. Акция
недвусмысленно намекала на знаменитое Бостонское чаепитие.

Источник:
http://ru.wikipedia.org/wiki/Бостонское_чаепитие

Автор:
Владислав Шашкин (Ярославль)

Вопрос 13:
Герой англоязычного фильма находится в здании, окруженном противниками.
Вражеский командир указывает, что герой остался совсем один, на что тот
отвечает: "Там, где вы видите одного человека, я вижу...". Скольких?

Ответ:
Четверых.

Комментарий:
Обыгрывается созвучие фразы "вижу четверых" - "see four" и взрывчатки
C-4. Персонаж заминировал здание, чтобы унести с собой на тот свет
побольше врагов. "Where you see one man, i see four".

Источник:
Hellsing Ultimate Abridged Episode 5.

Автор:
Андрей Горошников (Ярославль)

Вопрос 14:
В 1980 году певец Крис Кельми СДЕЛАЛ ЭТО и начал сольную карьеру. Два
человека как-то раз стали угрожать Майку Тайсону, после того как тот
отказался СДЕЛАТЬ ЭТО. Какие два слова мы заменили словами "СДЕЛАТЬ
ЭТО"?

Ответ:
Оставить "Автограф".

Зачет:
Оставить автограф.

Комментарий:
Крис Кельми оставил "Автограф" - рок-группу, которую он вместе с
Александром СитковЕцким основал в 1979 году. После того как Майк Тайсон
отказался оставить автограф, двое фанатов стали угрожать боксеру и даже
попытались побить его. Впрочем, они не выдержали и одного раунда.

Источник:
   1. http://ru.wikipedia.org/wiki/Кельми,_Крис
   2. http://www.kommersant.ru/doc/399374

Автор:
Владислав Шашкин (Ярославль)

Вопрос 15:
На одной социальной рекламе можно увидеть, например, портреты Леннона,
Пикассо и Эйнштейна, созданные при помощи хаотичного нагромождения
линий. Реклама утверждает, что ОНА не мешает таланту. Постарайтесь
написать, о чем идет речь, так, чтобы мы смогли это разобрать.

Ответ:
ДислексИя.

Комментарий:
Дислексия подразумевает проблемы с пониманием, письмом, почерком. В
вопросе упомянуты известные дислектики.

Источник:
http://www.dailymail.co.uk/news/article-2372565/Dyslexia-artist-Vince-Low-scribbles-portraits-famous-dyslexics-highlight-condition.html

Автор:
Александр Судаков (Ярославль)

Вопрос 16:
Персонаж Лукьяненко скептически спрашивает: "Чего стоит жизнь, начатая
на пепелище и закончившаяся пожаром?". А с кем перед этим сравнивает
влюбленного человека его романтически настроенный собеседник?

Ответ:
С фениксом.

Комментарий:
Собеседник говорит: "Человек, будто птица феникс, сгорает и
перерождается в пламени любви". Перерождается феникс, как известно, из
своего пепла.

Источник:
С.В. Лукьяненко. Купи кота. http://lib.ru/LUKXQN/r_kupi_kota.txt

Автор:
Андрей Горошников (Ярославль)

Вопрос 17:
Статья на сайте susi.ru [суси ру] об одном из деликатесов называется
"ОНА в соевом соусе". Википедия утверждает, что, скорее всего, ЕЕ
происхождение - чисто литературное, поскольку в 1917 году табельным
оружием в русской армии являлся "Наган". Назовите ЕЕ двумя словами.

Ответ:
Русская рулетка.

Комментарий:
Статья рассказывает о рыбе фугу, поедание которой сравнивают с русской
рулеткой. В первом письменном упоминании русской рулетки утверждается,
что в нее играли русские солдаты в 1917 году, но упоминаются пять шансов
на трагедию из шести, в то время как в револьвере "Наган" семь патронов.

Источник:
   1. http://www.susi.ru/fugu/fugu.html
   2. http://ru.wikipedia.org/wiki/Русская_рулетка

Автор:
Андрей Горошников (Ярославль)

Вопрос 18:
Одна из шуток о НЕЙ гласила, что корпорация "Microsoft" отложила выпуск
одного из своих продуктов до первого квартала 1901 года. Мы не просим
вас назвать ЕЕ. Назовите упомянутый продукт.

Ответ:
Windows 2000.

Зачет:
Любой продукт "Microsoft" с числом 2000.

Комментарий:
Речь идет о знаменитой "Проблеме 2000", в результате которой 1999 год во
многих программах должен был смениться на 1900-й.

Источник:
http://old.computerra.ru/offline/1999/281/2249/

Автор:
Андрей Горошников (Ярославль)

Вопрос 19:
Иван Бунин писал, что на родине ОНА - зеленая, но пожелтела от горя,
ублажая трактирную толпу. Википедия утверждает, что "Голубая ОНА" -
неправильный перевод, поскольку не соответствует смыслу произведения.
Назовите ЕЕ.

Ответ:
Канарейка.

Комментарий:
Название известной песни "Blue canary" [блю канАри] можно перевести и
как "Голубая канарейка", и как "Печальная канарейка", но по смыслу
подходит только второе. Дикая канарейка - действительно зеленая, а
домашняя, которую раньше нередко держали в трактирах, чаще всего была
желтой.

Источник:
   1. http://ru.wikipedia.org/wiki/Blue_Canary
   2. http://poetrylibrary.ru/stixiya/kanarejku-iz-za.html

Автор:
Григорий Ананьин (Ярославль)

Вопрос 20:
Героиня одного из комиксов, говоря о своем брате, утверждает, что тот
проиграл бы в драке даже носорогу. Напишите то, что мы пропустили в
предыдущем предложении.

Ответ:
Жуку-.

Зачет:
Рыбе-; птице-; жуку; рыбе; птице.

Комментарий:
Силой ее брат не отличается, поэтому проиграл бы даже жуку-носорогу.

Источник:
Комикс.

Автор:
Андрей Горошников (Ярославль)

Вопрос 21:
Внимание, в вопросе есть замена.
   Персонаж Генрика Сенкевича называет военных врачей "ПОЗВОНОЧНЫМИ",
намекая на используемый ими инструмент. Какое слово мы заменили словом
"ПОЗВОНОЧНЫЕ"?

Ответ:
Ланцетники.

Комментарий:
Ланцет - обычный инструмент военного врача. Замена обусловлена тем, что
ланцетники и позвоночные - подтипы хордовых животных.

Источник:
   1. Г. Сенкевич. Старый слуга. http://www.flibusta.is/b/49085/read
   2. http://ru.wikipedia.org/wiki/Хордовые

Автор:
Григорий Ананьин (Ярославль)

Вопрос 22:
Одна из статей на сайте habrahabr [хабрахАбр] посвящена популярному
способу, позволяющему увеличить срок работы мобильных устройств. В
заголовке статьи упоминаются точки... Над чем?

Ответ:
Над Li.

Зачет:
Ли; литий; Li-ion.

Комментарий:
Статья посвящена популярному мнению, что полная зарядка-разрядка
аккумулятора может увеличить срок работы устройства без подзарядки.
Самыми популярными аккумуляторами для мобильных устройств являются
литиевые.

Источник:
   1. http://habrahabr.ru/post/150661/
   2. http://ru.wikipedia.org/wiki/Литий-ионный_аккумулятор

Автор:
Андрей Горошников (Ярославль)

Вопрос 23:
На одном из комиксов с сайта xkcd.com персонажи любуются страшной
грозой, бушующей вокруг них. Подпись к комиксу гласит, что, согласно
статистике, показатель смертности среди людей, знакомых с ЭТИМ, во много
раз выше, чем среди остальных. Назовите ЭТО двумя словами.

Ответ:
Статистика смертности.

Зачет:
Показатель смертности.

Комментарий:
Персонажи знают, что статистический шанс умереть от удара молнии очень
мал, поэтому не спешат убраться из опасного места.

Источник:
http://xkcd.com/795/

Автор:
Андрей Горошников (Ярославль)

Вопрос 24:
В конце Второй мировой войны в Канаде чеканились так называемые
"победные" пятицентовики. На ободе монеты располагалась надпись "Мы
побеждаем, когда мы работаем охотно". Какая фамилия также присутствует в
описании монеты?

Ответ:
Морзе.

Комментарий:
Монетка небольшая, поэтому фраза была набрана на ободке азбукой Морзе.

Источник:
   1. http://www.staraya-moneta.ru/lib/957/
   2. http://en.wikipedia.org/wiki/Nickel_(Canadian_coin)#Commemorative_nickels

Автор:
Александр Судаков (Ярославль)

Тур:
15 тур. "Благоест" (Коломна)

Редактор:
Валерия Комаровская (Коломна)

Вопрос 1:
(pic: 20160326.jpg)
   Коломенский торговый центр "Глобус" имеет прозвище "ОН". ОН был
построен в 1894 году в европейской столице и пережил масштабную
реконструкцию спустя столетие. Назовите ЕГО двухкоренным словом.

Ответ:
Рейхстаг.

Комментарий:
Коломенская команда приветствует вас вопросом про Коломну и очень
надеется, что это будет единственный вопрос-коломна в пакете.

Источник:
   1. http://www.in-kolomna.ru/userdata/archive/1201872281.pdf
   2. http://ru.wikipedia.org/wiki/Рейхстаг_(здание)

Автор:
Кирилл Третьяков (Воскресенск)

Вопрос 2:
Внимание, в вопросе есть замена.
   Хотя родители этого американского актера, умершего в 2015 году, были
еврейскими мигрантами с Волыни, переводчики на русский язык так и не
определились с правильным склонением его фамилии. Если склонять ее как
украинскую, то выражение "голос этого американского актера" на слух
воспринимается как оксюморон. Назовите этого американского актера.

Ответ:
[Леонард] Нимой.

Комментарий:
Следуя разным вариантам склонения фамилии "Нимой", можно получить
выражения либо "голос Нимоя", либо "голос Нимого". "Голос немого" -
оксюморон.

Источник:
   1. http://ria.ru/spravka/20150227/1050099052.html ("Нимоя")
   2. http://www.gazeta.ru/culture/2015/02/27/a_6429549.shtml ("Нимого")

Автор:
Иван Морозов (Коломна)

Вопрос 3:
Виола, героиня фильма 2005 года, дама за пятьдесят, просит ассистентку
проверить биографию невесты своего сына, особенно ее интересуют любовные
связи избранницы. Получив нужные данные, ассистентка говорит, что у
девушки любовников за всю жизнь было меньше, чем у Виолы за один день на
ферме. Какое слово мы заменили в предыдущем предложении?

Ответ:
Вудсток.

Источник:
   1. Х/ф "Если свекровь - монстр..." (2005), реж. Роберт Лукетич, 35-я
минута.
   2. http://ru.wikipedia.org/wiki/Вудсток_(фестиваль)

Автор:
Валерия Комаровская (Коломна)

Вопрос 4:
ТАКИМИ казаками назывались взятые на учет и ограниченные в правах дети и
внуки руководителей Булавинского восстания. Один ТАКОЙ казак однажды
избавил село от ведьмы, а другой ТАКОЙ казак был обманут лисой. Какое
слово заменено на "ТАКИЕ"?

Ответ:
Сказочные.

Комментарий:
Учет указанной категории казаков велся на основании "сказок" -
документов, в которых царские подданные сообщали известные им факты по
запросу администрации. Также сказочными казаками можно назвать героев
произведений, относящихся к сказочному жанру.

Источник:
   1. http://kazak.academic.ru/1854/
   2. http://cyberleninka.ru/article/n/a-syu-skasku-pisal-po-veleniyu-gosudarya-svoego-skazki-o-sluzhbe-dumnyh-lyudey-v-kontse-xvii-veka
   3. https://sites.google.com/site/russkieskazki/home/skazki/skazki-narodov-rossii/kazaki/kazak-i-vedma
   4. https://sites.google.com/site/russkieskazki/home/skazki/skazki-narodov-rossii/kazaki/kazak-i-lisa-kazaca-skazka

Автор:
Иван Морозов (Коломна)

Вопрос 5:
Мать российского предпринимателя Давида зовут Сильва Ашотовна, а имя его
отца - Ши. Назовите фамилию Давида.

Ответ:
Ян.

Комментарий:
Отец Давида, как можно понять из имени, - китаец. Китайская фамилия, как
правило, состоит из одной морфемы. Мать Давида, судя по отчеству, -
армянка. Современные армянские фамилии обычно заканчиваются на "-ян".
Это намек на то, что у Давида фамилия Ян. (Давид Ян - основатель
компании "ABBYY", создатель программы "FineReader".)

Источник:
   1. http://www.iksmedia.ru/articles/455807-David-YAN-Rabota-vo-sne-i-nayavu.html
   2. http://ru.wikipedia.org/wiki/Китайская_фамилия
   3. http://ru.wikipedia.org/wiki/Армянское_имя

Автор:
Иван Морозов (Коломна)

Вопрос 6:
Согласно шутке из Воскресенской лиги КВН, парень, который привел свою
девушку на первое свидание на одну из сравнительно недавних киноновинок,
теперь имеет на лице [ПРОПУСК]. Пользователь сайта kinopoisk.ru
озаглавил свой негативный отзыв на этот кинофильм "[ПРОПУСК]". Пропуск
одним словом отличается от названия упомянутой киноновинки. Каким?

Ответ:
Красного.

Комментарий:
Речь идет о фильме "50 оттенков серого"; отрицательные рецензии на сайте
"Кинопоиск" имеют красный цвет.

Источник:
   1. Воскресенская лига КВН.
   2. http://www.kinopoisk.ru/user/4227457/comment/2188224/

Автор:
Андрей Волков (Воскресенск)

Вопрос 7:
За свою несообразительность и небольшой рост (149 см) автор вопроса
самоиронично называет себя ИКСОМ. Первые сведения об ИКСЕ получил Генри
Стэнли в 1890 году. Но название, под которым мы знаем ИКС сейчас, он
получил только в июне 1901 года. Назовите ИКС.

Ответ:
Окапи.

Зачет:
Окапи Джонстона.

Источник:
http://ru.wikipedia.org/wiki/Окапи

Автор:
Кристина Шадрина (Коломна)

Вопрос 8:
Один из основоположников анархизма утверждал: "АЛЬФА - это БЕТА". В
главе Уголовного кодекса России присутствует статья "БЕТА", которая
находится в главе "Преступления против АЛЬФЫ". Назовите АЛЬФУ и БЕТУ.

Ответ:
Собственность, кража.

Источник:
   1. http://ru.wikipedia.org/wiki/Прудон,_Пьер_Жозеф
   2. http://www.uk-rf.com/glava21.html

Автор:
Андрей Волков (Воскресенск)

Вопрос 9:
Однажды престарелый византийский император Анастасий собрал на обед трех
своих племянников, а потом предложил им три ложа, чтобы те отдохнули
после трапезы. Однако он никак не мог ожидать, что двое из них улягутся
вместе на одну постель. С какой целью Анастасий собрал племянников?

Ответ:
Выбор наследника.

Зачет:
По смыслу.

Комментарий:
Под подушку одной из кроватей император положил кусок пергамента с
надписью "власть", надеясь, что воля случая поможет ему определиться с
наследником, но эта кровать осталась незанятой.

Источник:
Дж. Норвич. История Византии. http://www.flibusta.is/b/377778/read

Автор:
Иван Морозов (Коломна)

Вопрос 10:
Сергей Довлатов говорил: "Любовь - это... ИКС. Типа - сегодня одна,
завтра другая...". Возможно, у многих из вас был ИКС в детстве. В 2005
году в Японии на выставке "Экспо" был выстроен целый павильон размером с
пятидесятиметровую башню, попав внутрь которого можно было ощутить себя
внутри ИКСА. Назовите ИКС словом греческого происхождения.

Ответ:
Калейдоскоп.

Источник:
   1. С.Д. Довлатов. Зона. http://www.flibusta.is/b/348600/read
   2. http://ru.wikipedia.org/wiki/Калейдоскоп

Автор:
Кристина Шадрина (Коломна)

Вопрос 11:
Согласно информации сайта haqqin.az, противогазы в России производят
лишь в двух городах - Серпухове и Армавире. А вот в Армении противогазы
часто используют не по назначению, например, для укрепления головки
тюльпанов или для срочного ремонта проколотого колеса автомобиля. В
армянских барах пошли еще дальше: там решили заливать в противогазы
кетчуп, майонез и горчицу, а затем прикреплять их к барной стойке, чтобы
клиент мог с легкостью добавить их содержимое в свой хот-дог. Какое
слово мы трижды заменили в вопросе?

Ответ:
Презервативы.

Комментарий:
Резиновое изделие N 2 было заменено на резиновое изделие N 1.

Источник:
http://haqqin.az/news/50993

Автор:
Андрей Волков (Воскресенск)

Вопрос 12:
(pic: 20160327.jpg)
   Объявление, размещенное в петербургском многоквартирном доме,
гласило: "Уважаемые соседи, давайте, пожалуйста, открывать окна не
совсем нараспашку, чтобы голуби не залетали в подъезд!". Кто-то сделал
приписку к этому объявлению, в которой упомянул имя собственное.
Напишите это имя собственное.

Ответ:
Москва.

Комментарий:
(pic: 20160328.jpg)

Источник:
http://pikabu.ru/story/vezhlivost_3336680

Автор:
Кирилл Третьяков (Воскресенск), в редакции Валерии Комаровской (Коломна)

Вопрос 13:
Внимание, в вопросе есть замена.
   Один из вымышленных персонажей, по имени "Икс", родился в Литве, а
второй, по имени "Игрек", - в Черногории. Некоторое время оба этих
персонажа жили на восточном побережье США, но нет сведений о том, что
они когда-либо встречались. Реальные обладатели имен "Икс" и "Игрек"
возглавляли противоборствующие армии в битве при Грументе в 207 году до
н.э. Какие имена заменены в вопросе на "Икс" и "Игрек"?

Ответ:
Ганнибал, Ниро.

Зачет:
Ганнибал, Нерон.

Комментарий:
Вымышленные персонажи, о которых идет речь в вопросе, - серийный убийца
Ганнибал Лектер и сыщик Ниро Вульф. Реальные обладатели этих же имен -
карфагенский полководец Ганнибал Барка и римский консул Гай Клавдий
Нерон. Имя "Ниро" является англоязычным вариантом имени "Нерон".

Источник:
   1. Серия фильмов о Ганнибале Лектере.
   2. Серия книг о Ниро Вульфе.
   3. Тит Ливий. История Рима от основания города. - Кн. XXVII, гл.
41-42. http://ancientrome.ru/antlitr/t.htm?a=1364002700#41

Автор:
Иван Морозов (Коломна)

Вопрос 14:
Сотрудники нашего офиса часто не могут решить, в какое именно кафе
сходить пообедать. Проблему мы решили просто: в коробку побросали
листочки с названиями любимых кафе, и каждый день Его Величество Жребий
определяет место нашего обеда. Эту процедуру мы называем словосочетанием
из двух слов, ставшим очень популярным в 2008 году. Напишите это
словосочетание.

Ответ:
Голодные игры.

Источник:
   1. ЛОАВ.
   2. http://ru.wikipedia.org/wiki/Голодные_игры_(роман)

Автор:
Валерия Комаровская (Коломна)

Вопрос 15:
Когда Сципиону Старшему предложили ПЕРВОЕ, у того уже было ВТОРОЕ. Когда
ВТОРОЕ предложили Петру Великому, у того уже было ПЕРВОЕ. Назовите
ПЕРВОЕ и ВТОРОЕ в правильном порядке.

Ответ:
Царский титул, императорский титул.

Зачет:
По смыслу.

Комментарий:
В эпоху Римской республики "император" - это лишь временный почетный
титул полководца. По рассказу Тита Ливия, после победы римлян при Бекуле
в 208 году до н.э. иберы пришли к Сципиону с целью отдаться под
покровительство римлян и назвали его царем. Сципион сказал иберам, что
для него самым почетным является звание императора, данное ему воинами,
"а царское звание, столь уважаемое у других народов, в Риме ненавистно".
В 1721 году титул императора был поднесен русскому царю Петру Великому.

Источник:
   1. http://cyberleninka.ru/article/n/stsipion-afrikanskiy-i-titul-imperator-v-politicheskoy-sisteme-rimskoy-respubliki
   2. Акт поднесения Государю Царю Петру I титула Императора
Всероссийского и наименования: Великого и Отца Отечества. // Полное
собрание законов Российской империи, с 1649 года. - СПб., 1830. - Т. 6:
1720-1722. N 3840. - С. 444
(http://www.nlr.ru/e-res/law_r/show_page.php?page=444&root=1/6/).

Автор:
Иван Морозов (Коломна)

Вопрос 16:
Внимание, в вопросе есть замена.
   Действие одного телесериала происходит в Канаде начала XX века.
Молодая девушка получает в наследство музей восковых фигур, экспонаты
которого воспроизводят сцены громких убийств того времени. Для
достижения наибольшего реализма в персонажах экспозиций, хозяйка
постоянно сотрудничает с ИКСОМ. Живет ли ИКС в самом Монреале или за
городом, в сериале не упоминается. Известно, что однажды один ИКС
вышвырнул полученное золото, дабы показать некой женщине, что исполняет
свой долг. Назовите эту женщину.

Ответ:
Леди Винтер.

Зачет:
Анна де Бейль; леди Кларик; Шарлотта Баксон; баронесса Шеффилд; графиня
де Ла Фер; Миледи.

Комментарий:
Девушка сотрудничает с палачом, чтобы тот ей отдавал личные вещи
казненных им преступников, а также веревки, на которых их вешал. Как
известно из романа Дюма, когда палач получил от Атоса деньги за казнь
Миледи, он произнес: "А теперь пусть эта женщина тоже знает, что я
исполняю не свое ремесло, а свой долг", после чего швырнул золото в
реку.

Источник:
   1. Телесериал "Наследство сестер Корваль" (2010).
   2. http://www.e-reading.club/chapter.php/81884/69/Dyuma_1_Tri_mushketera.html

Автор:
Валерия Комаровская (Коломна)

Вопрос 17:
Иосиф Бродский говорил: "Умеющий любить умеет ДЕЛАТЬ ЭТО". В
произведении 1880-х годов как раз не ДЕЛАЛИ ЭТОГО. Назовите заглавного
героя произведения или 1987 года, или 2009 года, который прославился на
всю страну, благодаря тому что умел ДЕЛАТЬ ЭТО.

Ответ:
Хатико.

Комментарий:
ДЕЛАТЬ ЭТО - ждать.

Источник:
   1. http://www.world-art.ru/lyric/lyric.php?id=7548
   2. http://ru.wikipedia.org/wiki/Не_ждали_(картина)
   3. http://ru.wikipedia.org/wiki/Хатико

Автор:
Кристина Шадрина (Коломна)

Вопрос 18:
Однажды в сериале "Касл" автор детективов Ричард Касл отправляется в
свое поместье в Хэмптонс, чтобы провести выходные с возлюбленной. Однако
умирающий человек падает в бассейн на заднем дворе дома Касла, и
влюбленные вынуждены оставить свои планы: Ричард приходит на помощь
неопытному шерифу, чтобы с блеском решить загадку преступления. Название
этой серии короче названия сериала, выходившего с 1984 по 1996 год, на
одну букву на языке оригинала, и на две буквы на русском языке. Напишите
название серии на русском или английском языке.

Ответ:
"Он написал убийство".

Зачет:
"Murder, he wrote".

Комментарий:
Серия целиком повторяет развитие сюжета любой серии сериала "Она
написала убийство": популярная писательница детективов Джессика Флетчер
приезжает куда-либо, там тут же случается убийство, и проницательная
писательница с блеском решает загадки, помогая, как правило, недалеким
представителям власти. Здесь всё то же самое, только с писателем во
главе.

Источник:
   1. Сериал "Касл", s05e04.
   2. http://ru.wikipedia.org/wiki/Список_эпизодов_телесериала_%C2%ABКасл%C2%BB#.D0.A1.D0.B5.D0.B7.D0.BE.D0.BD_5_.282012.E2.80.9313.29
   3. http://www.kinopoisk.ru/film/89641/

Автор:
Валерия Комаровская (Коломна)

Вопрос 19:
Внимание, в вопросе есть замены.
   В VII эпизоде эпопеи "Звездные войны" роль персонажа по имени Чукча
играет папуас. В известном романе второй половины XIX века персонаж по
фамилии Чукча отправился в путешествие на плоту вместе с папуасом. А в
биографическом фильме 2012 года одному папуасу довелось сыграть роль
национального героя чукчей. Мы не спрашиваем, какие слова в тексте
вопроса были заменены на "чукчей" и "папуасов". Назовите фамилию
упомянутого национального героя.

Ответ:
Маннергейм.

Комментарий:
"Чукча" в вопросе заменяет слово "финн", а "папуас" - слово "негр". Финн
- чернокожий персонаж VII эпизода "Звездных войн". Гекк Финн и негр Джим
- персонажи романа Марка Твена. Национального героя финнов Карла Густава
Эмиля Маннергейма в фильме "Маршал Финляндии" сыграл чернокожий
кенийский актер Тэлли Савалас Отиэно.

Источник:
   1. Х/ф "Звездные войны: Пробуждение силы" (2015), реж. Джей Джей
Абрамс.
   2. М. Твен. Приключения Гекльберри Финна.
http://www.flibusta.is/b/384870/read
   3. Х/ф "Маршал Финляндии" (2012), реж. Гилберт Лукалиа.

Автор:
Иван Морозов (Коломна)

Вопрос 20:
В эпизоде сериала "Секретные материалы" агент Малдер, увидев ряд
одинаковых строгих черных костюмов, упоминает УИЛЛА СМИТА. УИЛЛ СМИТ
умер в 2003 году в Нэшвилле. Кого мы заменили на УИЛЛА СМИТА?

Ответ:
Джонни Кэш.

Комментарий:
Кантри-певец Джонни Кэш имел прозвище "Человек в черном". Нэшвилл
считается столицей кантри.

Источник:
   1. http://x-files.wikia.com/wiki/Dreamland
   2. http://ru.wikipedia.org/wiki/Кэш,_Джонни
   3. http://ru.wikipedia.org/wiki/Нэшвилл

Автор:
Кирилл Третьяков (Воскресенск)

Вопрос 21:
В одном из рассказов О. Генри повествуется о фиаско великого
престидижитатора Германна. Как-то, во время путешествия, он стал
развлекать друзей и местных жителей своими фокусами: превращал воду в
жидкости разного цвета, жарил омлет в цилиндре и т.д. Бурные овации и
восторг публики вскружили голову Германну настолько, что "выйдя на
открытое пространство", он стал ДЕЛАТЬ ЭТО "из рукавов, из-за воротника
и карманов, причем по полудюжине, не меньше". Такая беспечность чуть не
стоила жизни и самому фокуснику, и его друзьям. Догадавшись, что же он
делал, назовите страну, в одной из провинций которой произошло столь
неудачное выступление.

Ответ:
Австралия.

Комментарий:
ДЕЛАТЬ ЭТО - вытаскивать кроликов.

Источник:
О. Генри. Волосы Падеревского. http://www.flibusta.is/b/306679/read#t131

Автор:
Валерия Комаровская (Коломна)

Вопрос 22:
Герой произведения Виктора Пелевина утверждает, что всегда рекламируются
не вещи, а ОНО, поэтому человек идет в магазин не за вещами, а за НИМ, а
ЕГО в магазине не продают. Ответьте четырьмя словами, больше не надо,
что значит ОНО для женщины, согласно источнику 1995 года.

Ответ:
Был бы милый рядом.

Комментарий:
ОНО - счастье. Как мы помним из творчества Татьяны Овсиенко: "Женское
счастье - был бы милый рядом".

Источник:
   1. В.О. Пелевин. Generation П.
http://pelevin.nov.ru/romans/pe-genp/10.html
   2. http://www.karaoke.ru/artists/ovsienko-tatjana/text/zhenskoe-schaste/

Автор:
Алена Гекк (Ейск), в редакции Валерии Комаровской (Коломна)

Вопрос 23:
В передаче "Давай поженимся" Лариса Гузеева часто дает ценные житейские
советы гостям и участникам программы. В одной из передач она дала
следующий совет излишне темпераментной участнице: "Когда будете выходить
из дома, не забывайте закрывать дверь". Что мы заменили словами "дом" и
"дверь"?

Ответ:
Себя, рот.

Источник:
Передача "Давай поженимся" от 18.06.2009 г.

Автор:
Николай Кирин (Коломна)

Вопрос 24:
Внимание, в вопросе словом "ОНА" заменены два слова на одну букву.
   Известная ОНА была сделана 23 февраля в Восточной Азии, не менее
известная ОНА была сделана 2 мая того же года в Центральной Европе.
Намного менее известная ТРЕТЬЯ ОНА была сделана 27 апреля того же года в
местечке за Полярным кругом. Назовите ТРЕТЬЮ ЕЕ тремя словами на одну
букву.

Ответ:
Фотография финского флага.

Зачет:
Фотография флага Финляндии.

Комментарий:
23 февраля 1945 года - на Иводзиме, 2 мая - в Берлине, 27 апреля - на
финско-норвежской границе. Начали со здания Рейхстага, заканчиваем
упоминанием флага на нем. :-)

Источник:
   1. http://ru.wikipedia.org/wiki/Водружение_флага_над_Иводзимой
   2. http://ru.wikipedia.org/wiki/Знамя_Победы_над_Рейхстагом_(фото_Халдея)
   3. http://fi.wikipedia.org/wiki/Kolmen_valtakunnan_rajapyykki

Автор:
Кирилл Третьяков (Воскресенск)

Тур:
16 тур. "Сборная с. Тупино" (Ступино - Коломна - Егорьевск - Великий
Новгород)

Редактор:
Максим Евланов (Харьков)

Инфо:
Мы не ставили перед собой задачу сделать тур сложным, возможно, многие
факты из вопросов для вас известны - но мы и не стремились обозначить
глубину наших мыслей и нашего внутреннего мира. В нашем пакете мы
постарались собрать все банальности, которыми грешат современные пакеты,
- вашему вниманию будет представлено четыре вопроса по стишкам-порошкам,
шесть вопросов с раздаточным материалом, шесть вопросов по сериалам и аж
девять вопросов с заменами. Эпиграф. Пост в социальной сети "ВКонтакте"
в паблике "Подслушано на ЧГК": "Был на ШЧРе, видел много замечательных
школьников. Господи, сделай, пожалуйста, так, чтобы они не продолжили
играть после того, как выпустятся, не превратились в мерзких пропитых
задротов с грязной головой, которым медаль СтудЧРа дороже, чем мать
родная".

Вопрос 1:
Порошковый дуплет.
   1. Закончите стишок-порошок двумя словами:
   антон бы мог иметь детишек
   цветущий сад и две избы
   вся жизнь антона состояла
   ...
   2. Закончите стишок-порошок неологизмом:
   вобще то шрек веселый малый
   пока оглоблей не огреть
   ну тут уж всякий начинает
   ...

Ответ:
   1. из бы
   2. огреть

Источник:
   1. https://vk.com/sandalporoshki?w=wall-31481258_94856
   2. https://vk.com/sandalporoshki?w=wall-31481258_94924

Автор:
Олег Евстафьев (Ступино)

Вопрос 2:
Внимание, в вопросе есть одна замена.
   В книге Михаила Успенского "Там, где нас нет" герои обсуждают
особенности роста бороды [цитата]: "В лесу главный - леший, в реке -
водяной, в дому - домовой, в бане - банный, в овине - овинный, в поле -
полевой, в любовном деле - ТАКОЙ, во дворе - дворовой, в бороде -
бородовой!" [конец цитаты]. Какое слово мы заменили на "ТАКОЙ"?

Ответ:
Половой.

Источник:
М.Г. Успенский. Там, где нас нет.
http://www.loveread.ec/read_book.php?id=43568&p=37

Автор:
Юлия Кощеева (Коломна)

Вопрос 3:
В совершенно бездарнейшем отечественном сериале "Шаманка" расследуется
покушение на автора популярных детективных романов Дину Александровну
Волину. На вопрос о конкуренции издательств директор издательства
отвечает: "... На то и существуют контракты, но мы Волину не
[ПРОПУСК]..." и извиняется за каламбур. Заполните пропуск глаголом.

Ответ:
Неволили.

Источник:
Телесериал "Шаманка", 6-я серия.

Автор:
Юлия Кощеева (Коломна)

Вопрос 4:
Это слово французского происхождения обозначает распространенный
материал, из которого делают в том числе мебель, шкатулки и многое
другое. В Китае в древности из него делали шлемы. Автор вопроса в шутку
называет этим словом туалетную бумагу. Напишите это слово.

Ответ:
Папье-маше.

Зачет:
Попье-маше. :-)

Комментарий:
"Papier m&acirc;ch&eacute;" в переводе с французского - "жеваная
бумага". Китай - еще и родина бумаги.

Источник:
http://ru.wikipedia.org/wiki/Папье-маше

Автор:
Олег Евстафьев (Ступино)

Вопрос 5:
Порошковый дуплет.
   1. Закончите стишок-порошок двумя словами:
   я так и вижу ту картину
   икар средь птиц на высоте
   с летательным приспособленьем
   ...
   2. Закончите стишок-порошок двумя словами:
   смотрю не понимая смысла
   как прячут в полной темноте
   детей в капусту атеисты
   ...

Ответ:
   1. без те
   2. без те

Источник:
   1. https://vk.com/sandalporoshki?w=wall-31481258_94155
   2. https://vk.com/sandalporoshki?w=wall-31481258_94735

Автор:
Олег Евстафьев (Ступино)

Вопрос 6:
В одной из серий сериала "Саша + Маша" Саше снится эротический сон, в
котором его начинает гладить сначала одна рука Маши, потом другая,
третья, пятая, восьмая. В эпизоде в избыточном количестве присутствует
некий предмет. Дабы избавиться от назойливых ласк, Саша задает Маше
вопрос, после которого поглаживания прекращаются. Представьте, что этот
вопрос задан вам сейчас, и ответьте на него.

Ответ:
[Текущее время с погрешностью в пять минут.]

Комментарий:
На каждой руке Маши наручные часы. Саша задает Маше вопрос: "Который
час?". Маша смотрит на часы, при этом прекращая поглаживания.

Источник:
Телесериал "Саша + Маша".

Автор:
Юлия Кощеева (Коломна)

Вопрос 7:
Внимание, в вопросе есть две замены.
   Один ЖЖ-пост гласит: "На улицах гололед, и в Москве уже родились два
новых вида спорта - ТАКОЕ шатАние и СЯКОЕ пАдание". Напишите в
правильном порядке слова, которые мы заменили на "ТАКОЕ" и "СЯКОЕ".

Ответ:
Фигурное, синхронное.

Зачет:
В правильном порядке.

Источник:
http://vizza.livejournal.com/731492.html

Автор:
Юлия Кощеева (Коломна), Виктор Плотников (Великий Новгород)

Вопрос 8:
Журнал "Витамин здоровья" выходит шесть раз в год. На обложке четвертого
номера журнала за 2015 год изображен ломтик тостерного хлеба, на котором
отчетливо видны следы... От чего?

Ответ:
Купальник.

Комментарий:
В номере есть статья, посвященная правилам загара. Как нетрудно
догадаться, при частоте выхода шесть раз в год четвертый номер попадет
на июль или август.

Источник:
Журнал "Витамин здоровья", 2015, N 4.

Автор:
Юлия Кощеева (Коломна)

Вопрос 9:
Порошковый дуплет.
   1. Назовите того, кто упомянут во второй половине стишка-порошка,
который начинается так:
   пройдя юдОль свою земную
   подходит почтальон к вратам
   ...
   ...
   2. Закончите стишок-порошок одним словом:
   овсянка бэрримор ужасна
   скорее принесите йод
   не лезет в горло да и больно
   ...

Ответ:
   1. Галчонок.
   2. клюет

Зачет:
   1. Хватайка; галчонок Хватайка.

Комментарий:
   пройдя юдОль свою земную
   подходит почтальон к вратам
   оттуда голосом галчонка
   кто там
   Такой орнитологический дуплет.

Источник:
   1. https://vk.com/sandalporoshki?w=wall-31481258_94688
   2. https://vk.com/sandalporoshki?w=wall-31481258_95025

Автор:
Олег Евстафьев (Ступино)

Вопрос 10:
В сериале "Гавайи 5.0" рассказывается история, произошедшая в XIX веке:
банда пиратов захватила дворец короля во время торжественного приема.
Снимая колье с одной из дам, капитан пиратов спрашивает о названии
замечательного аромата, который исходит от дамы. Дама дерзко отвечает,
что это АЛЬФА, и рекомендует попробовать капитану. АЛЬФА как-то помогла
решить проблему с другим ценным предметом. Кому помогла?

Ответ:
Архимеду.

Комментарий:
"Аромат" называется ванна.

Источник:
"Гавайи 5.0", s06e01.

Автор:
Юлия Кощеева (Коломна)

Вопрос 11:
Внимание, в вопросе есть две замены.
   Сайт atkritka.com утверждает, что "в дешевых учебниках по
математическому анализу вместо ТАКИХ логарифмов используются логарифмы
СЯКИЕ". Какое слово мы заменили на "ТАКИЕ" и какие два слова мы заменили
на "СЯКИЕ"?

Ответ:
Натуральные, идентичные натуральным.

Источник:
http://atkritka.com/384367/

Автор:
Юлия Кощеева (Коломна), Виктор Плотников (Великий Новгород)

Вопрос 12:
[Ведущему: не озвучивать дефис в названии канала.]
   Внимание, в вопросе есть две замены.
   Посвященный вязанию канал на YouTube носит название "Специ-фикация".
В предыдущем предложении мы заменили две буквы. Напишите измененное
слово в исходном виде.

Ответ:
Спицы-фикация.

Зачет:
Спицыфикация.

Источник:
http://www.youtube.com/channel/UCn_FwSEZNKdHj3gwCNKLuLg

Автор:
Ольга Ковалёва (Егорьевск)

Вопрос 13:
Согласно стишку-порошку, однажды Илья Муромец в процессе общения стал
намного вежливее. Какие три слова мы заменили словами "стал намного
вежливее"?

Ответ:
Перешел на вы.

Комментарий:
   эй ты кричит илюша змею
   тот приподнял три головы
   илья подумав переходит
   на вы

Источник:
https://vk.com/sandalporoshki?w=wall-31481258_95118

Автор:
Олег Евстафьев (Ступино)

Вопрос 14:
(pic: 20160329.jpg)
   Внимание, в вопросе есть одна замена.
   Розданное вам изображение с сайта adme.ru озаглавлено словами "Какая
неделикатность". Какое слово в предыдущем предложении мы заменили?

Ответ:
Бестактность.

Источник:
(pic: 20160330.jpg)

Автор:
Юлия Кощеева (Коломна), Виктор Плотников (Великий Новгород)

Вопрос 15:
В одном сериале продвинутая в технике девушка-эксперт рассказывает
пожилому собеседнику о браслете для бега: в нем есть GPS, он фиксирует
расстояние, темп и время бега, а еще считает пульс и количество шагов,
датчики движения следят за тем, как пользователь спит, через Bluetooth
он подключается к смартфону и может слать еженедельные e-mail'ы об
успехах пользователя. Пожилой собеседник, выслушав девушку, говорит:
"Если бы тридцать лет назад мне сказали, что можно послать e-mail с
браслета, который оценивает мою активность в постели, я бы спросил...".
Восстановите вопрос пожилого человека.

Ответ:
"Что такое e-mail?".

Зачет:
По смыслу.

Источник:
Сериал "Морская полиция, спецотдел", s13e03.

Автор:
Юлия Кощеева (Коломна)

Вопрос 16:
Вопрос посвящается картине Сальвадора Дали "Постоянство памяти".
   Внимание, в вопросе есть две замены.
   В комментариях к одному из ЖЖ-постов содержится признание в
невежестве: девушка думала, что "мессия" - это ВОЕННОЕ звание. Признание
сопровождается комментарием другого пользователя: не ВОЕННОЕ, а
НАЦИОНАЛЬНОЕ. Слова, которые мы заменили на "военное" и "национальное",
в русском языке отличаются одной буквой. Напишите эти слова.

Ответ:
Армейское, арамейское.

Источник:
http://avva.livejournal.com/1481343.html?thread=29545343

Автор:
Юлия Кощеева (Коломна), Виктор Плотников (Великий Новгород)

Вопрос 17:
   <раздатка>
   Поможем вам быстро и дешево испугаться
   </раздатка>
   Внимание, в вопросе есть одна замена.
   Перед вами рекламный слоган магазина оптики, где мы заменили одно
слово. Напишите его в исходном варианте.

Ответ:
Оправиться.

Автор:
Артем Матухно (Одесса), в редакции Юлии Кощеевой (Коломна) и Виктора
Плотникова (Великий Новгород)

Вопрос 18:
Согласно сериалу "Кости", Бенджамин Франклин якобы очень любил "Нежности
в лесу". А под каким названием эти "нежности" известны нам?

Ответ:
"Секс на пляже".

Источник:
Сериал "Кости", s11e05.

Автор:
Юлия Кощеева (Коломна), в редакции Олега Евстафьева (Ступино)

Вопрос 19:
Рассказывают, что две актрисы с одинаковыми именами и фамилиями
одновременно стали знаменитыми. Ну так бывает. Обе считали, что сменить
имя и тем самым признать себя менее известной должна другая. Ни
многочисленные переговоры продюсеров, ни даже суды не помогали. И вдруг
одна из актрис уступила. Победительница торжествовала ровно до того
момента, когда стало известно, что ее соперница добавила к своей фамилии
одно слово. Какое?

Ответ:
Младшая.

Источник:
http://www.anekdot.ru/id/784450/

Автор:
Юлия Кощеева (Коломна), Виктор Плотников (Великий Новгород)

Вопрос 20:
В своем посте ЖЖ-пользователь Петр Капулянский говорит: "Случайно видел
одну серию Штирлица в цвете. Верните любимому сериалу ИХ!..". ОНИ
появились в 2011 году. Назовите ИХ.

Ответ:
"Пятьдесят оттенков серого".

Источник:
   1. http://petru-her.livejournal.com/4523205.html
   2. http://ru.wikipedia.org/wiki/Пятьдесят_оттенков_серого

Автор:
Юлия Кощеева (Коломна), Виктор Плотников (Великий Новгород)

Вопрос 21:
(pic: 20160331.jpg)
   Внимание, в вопросе есть одна замена.
   Подпись к розданной вам картинке гласит: "Двойной ять". Какое слово в
предыдущем предложении мы заменили?

Ответ:
Дабл.

Источник:
http://pikabu.ru/story/dabl_quotyatquot_3244614

Автор:
Юлия Кощеева (Коломна), Виктор Плотников (Великий Новгород)

Вопрос 22:
Героиня сериала "Мыслить как преступник" вечером ДЕЛАЕТ ЭТО. Утром на ее
работе происходит следующий диалог:
   - Привет, парни, как выгляжу?
   - Отлично.
   - Вчера вечером села на диету и сбросила 84 кг.
   Какие слова мы заменили на "ДЕЛАЕТ ЭТО"?

Ответ:
Расстается с парнем.

Зачет:
Расстается с женихом и т.п.

Источник:
Телесериал "Мыслить как преступник".

Автор:
Юлия Кощеева (Коломна)

Вопрос 23:
Внимание, в вопросе есть две замены.
   ИКС - вежливость, ЕКС - дополнение. Какие слова мы заменили на ИКС и
ЕКС?

Ответ:
КомплИмент, комплЕмент.

Комментарий:
Комплимент - "приятное обращение". Комплемент (например, от шеф-повара)
- дополнение к основному заказу.

Источник:
http://explain-a-word.blogspot.ru/2013/10/blog-post.html

Автор:
Юлия Кощеева (Коломна)

Вопрос 24:
ИКС писал об одном ИГРЕКЕ, что он молчал, когда на него глядели, и что
его называли "индюком". В другом тексте ИГРЕК ИКС твердит "Никогда!".
Назовите ИКСА и ИГРЕКА.

Ответ:
Флобер, попугай.

Комментарий:
   Он говорит: "Jamais!", он всё твердит:
   "Jamais, jamais, jamais, jamais!"
   И плачет по-французски...

Источник:
   1. Г. Флобер. Простая душа. http://www.flibusta.is/b/95332/read
   2. http://www.planetaquarium.com/discography/songs/jamais_pop861.html

Автор:
Владлен Макаров (Великий Новгород)

Тур:
17 тур. "Сомнительные достижения" (Москва - Ивантеевка)

Вопрос 1:
Когда автор вопроса иммигрировал в Россию, его определили в седьмой
класс, где он познакомился с историей о герое, которого часто сравнивают
с Моисеем. Назовите двумя словами, начинающимися на одну и ту же букву,
то, что напомнило автору изображение этого героя в момент его подвига.

Ответ:
Статуя Свободы.

Комментарий:
Речь идет о Данко из "Старухи Изергиль" Горького. Данко вырвал сердце и
поднял его над головой, чтобы осветить дорогу своим спутникам. Больше
вопросов по личному опыту автора вопроса в нашем пакете не будет. :-)

Источник:
Личный опыт автора вопроса.

Вопрос 2:
В японском аниме-сериале "Темный Дворецкий" герои побеждают и
одомашнивают гигантскую демоническую собаку и называют ее ИКС.
Неизвестно, является ли это своеобразной отсылкой к другому
мультипликационному персонажу с Запада. Тем не менее, ответьте, что мы
заменили на ИКС.

Ответ:
Плуто.

Комментарий:
Демоническая собака гораздо больше смахивает на римского бога смерти и
подземного мира, чем дружелюбный пес Микки-Мауса.

Источник:
Аниме-сериал "Темный Дворецкий".

Вопрос 3:
Хорхе Луис Борхес говорил, что 602-я - самая магическая: "Он слышит
начало истории, которая включает в себя всё остальные, а также саму
себя". В русском переводе, однако, речь идет совсем о другом, а концовка
- такая же, как и всегда. Как же называется то, что, таким образом,
слышит человек, упомянутый в цитате Борхеса?

Ответ:
"Тысяча и одна ночь".

Комментарий:
Борхес рассуждает об арабской сказке "Тысяча и одна ночь", говоря о
ночи, когда Шахерезада начала рассказывать шаху его собственную историю
(т.е. собственно произведение "Тысяча и одна ночь"), повторяя с самого
начала все свои сказки, что могло растянуть и без того длинный цикл до
бесконечности. Неизвестно, где Борхес ознакомился с такой вариацией
602-й истории, но в самой распространенной русской версии концовка у нее
есть, как и всегда, - "И Шахерезаду застигло утро, и она прекратила
дозволенные речи".

Источник:
   1. http://ru.wikipedia.org/wiki/Mise_en_abyme
   2. http://www.fairy-tales.su/narodnye/arabskie_skazki/1001noch/5386-rasskaz-o-careviche-i-semi-veziryax-noch-602.html

Вопрос 4:
Считается, что ОН родился в Пенсильвании в 1783 году. Став моряком, ОН
однажды спасся от смерти во время стычки с пиратами, но попал к ним в
плен. В другой раз, сбежав с пиратского корабля, ОН сначала две мили
плыл по открытому морю, а затем долгое время шел на север без карт и
снаряжения. Мы не просим назвать ЕГО имя. Назовите произведение XXI
века, главным героем которого ОН является.

Ответ:
"Выживший".

Комментарий:
Как вы могли понять из описания в вопросе, известному американскому
первопроходцу Хью Глассу несколько раз приходилось выживать в
экстремальных обстоятельствах еще до событий, описанных в фильме
Алехандро Иньярриту "Выживший".

Источник:
http://ru.wikipedia.org/wiki/Гласс,_Хью

Вопрос 5:
Хьюберт Блейн Вольф-старший отказывался платить по квитанциям, выиграл
суд и потребовал извинений у "New York Times" - а всё из-за ошибок,
которые совершали виновные в этом люди. Во избежание подобных ошибок мы
не просим вас назвать то, что мы пропустили в тексте вопроса. Ответьте,
какая особенность Хьюберта стала причиной этих ошибок.

Ответ:
Длинное имя.

Зачет:
Самое длинное имя (по некоторым версиям, это так).

Комментарий:
В полном имени Хьюберта, состоящем из нескольких сотен символов,
постоянно делали ошибки - в том числе в налоговых квитанциях (что
позволяло ему отказываться от их оплаты) и в газетах.

Источник:
http://ru.wikipedia.org/wiki/Вольф%2B585_старший

Вопрос 6:
Один из них - актер, а другой - певец. Каждый из них побеждал в своем
виде соревнований. Родились они в одной стране с разницей примерно в
сорок лет, что дает основания для шуток о деградации Европы вообще и
этой страны в частности. Другие юмористы добавляют к сравнению также
композитора и политика, хотя один из двойки - также политик, а о втором
многие политики, включая Владимира Путина, высказывались. Назовите
обоих.

Ответ:
[Арнольд] Шварценеггер, Кончита Вурст.

Зачет:
[Арнольд] Шварценеггер, Томас Нойвирт.

Комментарий:
Оба родились в Австрии. Шварценеггер побеждал в конкурсе "Мистер
Олимпия", ну и на выборах губернатора Калифорнии, а Кончита - в конкурсе
"Евровидение". Одни шутники говорят о деградации, произошедшей за эти
сорок лет, другие вспоминают Моцарта и Гитлера как другой пример,
говорящий, скорее, о волнообразной смене гениев и спорных личностей в
этой стране.

Источник:
   1. http://ru.wikipedia.org/wiki/Шварценеггер,_Арнольд
   2. http://ru.wikipedia.org/wiki/Кончита_Вурст

Вопрос 7:
Психоаналитики считают, что на творчество этого художника оказала
влияние смерть его матери, труп которой нашли с закрытым тканью лицом.
Назовите этого художника.

Ответ:
[Рене] Магритт.

Комментарий:
На картинах художника лица героев часто оказываются закрытыми тем или
иным образом. В связи с психоаналитиками можно вспомнить и его работу
"Вероломство образов".

Источник:
https://vk.com/arttraffic2?w=wall-52526415_9304

Вопрос 8:
Внимание, в вопросе есть замена.
   В песне Черныша и Барселоны говорится, что "в Новосибирске никогда
[ПРОПУСК], как жаль, что этого не скажешь про людей". В другом городе
обычно [ПРОПУСК] с ноября по апрель. Заполните пропуск тремя словами.

Ответ:
Не разводятся мосты.

Зачет:
Не разводят мосты (с недовольством и тяжелым сердцем).

Источник:
   1. http://megalyrics.ru/lyric/chiernysh-i-barsielona/v-novosibirskie-nie-razvodiatsia-mosty.htm
   2. http://www.ptmap.ru/bridges/razvod

Вопрос 9:
Внимание, в вопросе есть замены.
   На этой картине критики часто подмечают красноту глаз натурщицы.
Возможно, это и послужило причиной ссоры ее предполагаемого
возлюбленного с художником. Догадавшись, что мы заменили в тексте
вопроса, назовите эту картину.

Ответ:
"Происхождение мира".

Комментарий:
На картине "Происхождение мира" Гюстава Курбе в откровенном виде
изображена обнаженная девушка. Некоторые критики заметили красноту ее
половых губ и предположили, что между ней и художником непосредственно
перед написанием картины был сексуальный контакт. Возможно, жених
натурщицы тоже это заметил - во всяком случае, известно, что с Кюрбе они
вскоре поссорились.

Источник:
http://portamur.ru/blogs/pavlov/proishojdenie-mira-gyustav-kurbe.html?PAGEN_1=12

Вопрос 10:
Раньше этот элемент памятника был сделан из искусственного рубина и
часто похищался вандалами, орудующими на Новодевичьем кладбище. Теперь
его изготовили из пластика и намертво закрепили в руках. Мы не
спрашиваем, что это за элемент. Ответьте, на чьей могиле установлен
данный памятник.

Ответ:
[Александра Николаевича] Бакулева.

Комментарий:
На могиле известного кардиохирурга установлен памятник в виде
вырывающихся из-под земли рук, держащих стилизованное сердце.

Вопрос 11:
В 1896 году в штате Род-Айленд некое необычное для того времени
мероприятие собрало большое число зрителей. Однако вскоре люди
заскучали, и кто-то громко попросил СДЕЛАТЬ ЭТО. Ответьте двумя словами,
заканчивающимися на одну и ту же букву, что мы заменили на "СДЕЛАТЬ
ЭТО".

Ответ:
Впрячь лошадь.

Комментарий:
Это была первая гонка на автомобилях, которые в то время ездили еще
слишком медленно.

Источник:
Д. Ергин. Добыча. Всемирная история борьбы за нефть, деньги и власть.
http://www.flibusta.is/b/402892/read

Вопрос 12:
   <раздатка>
   Согласно шутке, написанной в Интернете, однажды к Константину
Сергеевичу в гости пришел известный человек и попросил поставить
[ПРОПУСК].
   </раздатка>
   Недавно в Москве неизвестные развесили плакаты, стилизованные под
рекламирующие [ПРОПУСК], но подразумевающие другого [ПРОПУСК]. Алексей
Навальный вскоре заявил, что не причастен к этому. Восстановите пропуск,
не меняя форму пропущенного слова.

Ответ:
Чайку.

Комментарий:
Чехов просит Станиславского поставить чайкУ. Или чАйку. Из письменного
текста это непонятно. Афиша пьесы "Чайка" была изменена людьми,
вдохновленными нашумевшим расследованием Алексея Навального о Юрии Чайке
и его сыновьях. В частности, на ней упоминались спецслужбы, Браудер и
монолог "Почему чаек не сажают?".

Источник:
   1. http://www.anekdot.ru/id/783851/
   2. https://www.facebook.com/navalny/photos/pcb.1104126232939792/1104126162939799/?type=3

Вопрос 13:
По мнению некоторых искусствоведов, ЕГО творчество - это отражение идей
Платона о том, что Вселенная создана из хаоса путем комбинации
элементов. Например, в диалоге "Тимей" Платон говорит, что "бог устроил
мир как единое видимое живое существо, содержащее все сродные ему по
природе живые существа в себе самом". Назовите ЕГО.

Ответ:
[Джузеппе] Арчимбольдо.

Комментарий:
Картины Арчимбольдо представляют собой портреты людей, составленные из
различных элементов.

Источник:
   1. Платон. Тимей. http://www.flibusta.is/b/166088/read
   2. В. Кригескорте. Джузеппе Арчимбольдо, 1527-1593.

Вопрос 14:
Эдуард Лимонов пишет, что у НЕГО два отца - Герострат и Малевич.
Пользователи Твиттера пишут, что у НЕГО стальные яйца. Назовите ЕГО.

Ответ:
[Петр] Павленский.

Комментарий:
Лимонов приписывает Павленскому одновременно тягу к уничтожению и
пиромании и сомнительные творческие способности. Яйца у него
действительно стальные - и потому что он без проблем прибил их к
брусчатке, и потому что не побоялся поджечь дверь здания ФСБ.

Источник:
   1. http://limonov-eduard.livejournal.com/736703.html
   2. http://www.maximonline.ru/humor/made-in-web/_article/pavlenskiy-fsb/

Вопрос 15:
Согласно Глебу Архангельскому, к ИКСАМ и АЛЬФАМ нужен разный подход.
АЛЬФЫ нужно "есть" сразу, не откладывая на потом, а ИКСЫ - разрезАть на
кусочки, потому что целиком за один раз "проглотить" невозможно. АЛЬФ
можно попробовать, например, в кафе "Ше Андре", а вот ИКСОВ едят лишь в
Африке и некоторых азиатских странах, что, однако, уже угрожает их
популяции. Назовите ИКСОВ или АЛЬФ.

Ответ:
Слоны.

Зачет:
Лягушки.

Комментарий:
В тайм-менеджменте "Лягушки" - это мелкие неприятные дела, которые нужно
сделать быстро, что образно называется "проглотить лягушку", а "Слоны" -
более обстоятельные дела. Лягушками всегда можно полакомиться во
французском ресторане, а вот даже незначительное употребление слонов в
пищу угрожает им не меньше, чем их предкам мамонтам.

Источник:
   1. http://technotes.skycover.ru/2012/02/25/kak-vesti-ezhednevnik/
   2. http://www.chez-andre.com/
   3. http://elhow.ru/zhivotnye/dikie-zhivotnye/edjat-li-slonov

Вопрос 16:
Эту известную притчу можно считать частным случаем концепции ПЕРВОГО. Ее
герои обладали тем же дефектом, что и ВТОРОЙ. Другой ее герой, как и
ПЕРВЫЙ, уже упоминался в этом пакете. ПЕРВЫЙ и ВТОРОЙ были одной
национальности. Назовите ПЕРВОГО, ВТОРОГО и героев этой притчи.

Ответ:
Платон, Гомер, слепые, слон.

Зачет:
Платон, Гомер, слепые мудрецы, слон; те же без слона.

Комментарий:
Слепые, каждый из которых ощупывал слона и представлял его по-своему,
могут быть иллюстрацией того, как мир идей Платона воспринимается
людьми, которые способны видеть лишь его тени на стене пещеры. Гомер,
как и герои притчи, был слеп. Гомер и Платон - эллины.

Вопрос 17:
В одном фильме сперма героя попадает на НЕГО. "Претендентов" на то,
чтобы быть ИМ, проверяли с помощью фрактального анализа - ответвления
теории хаоса, т.к., по мнению физиков, случайное человеческое движение
подчиняется определенным закономерностям. Назовите ЕГО двумя словами,
начинающимися на одну и ту же букву.

Ответ:
Полотно Поллока.

Комментарий:
Брызги спермы органично смотрятся среди брызгов краски на картине
Поллока "Номер 5". Физики считают, что хаотичное разбрасывание краски
Поллоком все-таки имеет особый почерк, благодаря которому можно отличить
полотна его авторства от мистификаций.

Источник:
   1. Х/ф "Клуб "Shortbus"" (2006), реж. Джон Кэмерон Митчелл.
   2. http://lenta.ru/news/2006/02/10/fractal/

Вопрос 18:
На одних пародийных изображениях ОНИ показаны погрязшими в домашних
заботах, потреблении или войне, на других - с более реалистичными
прическами. А кто стал одной из НИХ после совершённой в 2012 году
сделки?

Ответ:
Лея.

Комментарий:
ОНИ - диснеевские принцессы. На изображениях обыгрывается то, что
осталось за кадром - отнюдь не сказочная повседневная жизнь и прически,
испорченные водой, ветром и прочим. Когда Дисней приобрел права на
съемки новых "Звездных войн", принцессу Лею также стало можно считать
диснеевской.

Источник:
   1. http://vev.ru/blogs/disneevskie-princessy-v-neobychnyh-obrazah-chast-1.html
   2. http://abcnews.go.com/blogs/business/2012/10/disney-to-acquire-lucasfilms-for-4-billion/

Вопрос 19:
(pic: 20160332.jpg)
   В XVII-XIX веках участники иногда ДЕЛАЛИ ЭТО в петициях, чтобы было
сложнее определить, кто из них лидер. Постарайтесь СДЕЛАТЬ ЭТО в
раздатке - даже если вы один за столом.

Ответ:
[Росписи игроков (или одного игрока), замыкающиеся в форму круга.]

Источник:
http://en.wikipedia.org/wiki/Round-robin_(document)

Вопрос 20:
Эдуард Лимонов, говоря о том, что Европа становится колонией, пишет, что
ИКС истории вернулся и похож на ИГРЕК. Назовите ИКС и ИГРЕК.

Ответ:
Бумеранг, полумесяц.

Комментарий:
По мнению Лимонова, из-за наплыва мигрантов, пользующихся социальными
гарантиями европейских стран и таким образом разоряющих их, Европа
становится колонией своих собственных бывших колоний - которые являются
преимущественно мусульманскими.

Источник:
http://limonov-eduard.livejournal.com/760797.html

Вопрос 21:
Блиц.
   В сериале "Однажды в сказке" герои сказок попали в наш мир, в город
Сторибрук, но сохранили многое от своих волшебных личностей.
   1. Какой сказочный персонаж в сериале работает официанткой и носит
имя Руби?
   2. Какой сказочный персонаж зовется Синди Гласс и издает ежедневную
газету в Сторибруке?
   3. Какой сказочный персонаж подрабатывает юристом и известен как
Мистер Голд?

Ответ:
   1. Красная Шапочка.
   2. Волшебное Зеркало.
   3. Румпельштильцхен.

Комментарий:
   1. "Руби" переводится как "красный". Красная Шапочка, как и в сказке,
носит еду.
   2. "Гласс" переводится как "стекло, зеркало". Синди Гласс издает
газету с типичным названием "Зеркало Сторибрука".
   3. Румпельштильцхен известен тем, что любил заключать сделки и прял
золото из соломы.

Источник:
Сериал "Однажды в сказке".

Вопрос 22:
В интернет-комиксе супергерои жалуются на своего коллегу, упоминая член,
ядерный взрыв и фотографию с отцом. Назовите имя этого коллеги.

Ответ:
Роршах.

Комментарий:
Роршах - член Хранителей с характерными пятнами на маске, в которых
каждый видел что-то свое.

Источник:
https://vk.com/by_duran?w=wall-25336774_7025

Вопрос 23:
На обложке одного из номеров журнала "Дилетант" изображены Петр I,
Екатерина II, Путин, Сталин. Под что стилизовано это изображение?

Ответ:
Гора Рашмор.

Источник:
(pic: 20160333.jpg)

Вопрос 24:
   <раздатка>
   В сыром подвале под старой церковью разводят они костер и начинают
варить целебное зелье. ИКС добавляет в воду страусиный уксус, экстракт
дикого творога, чугуниевую стружку и лыжную мазь на скипидарной основе,
помешивая это бивнем арктического моржа.
   Великий целитель высыпает в котел пригоршню пчелиных языков и глаз,
приправив их бальзамом "Звездочка" и советской конфетой "Холодок".
   Зелье вспыхивает и начинает бурлить, источая едкий аромат пива
"Очаково".
   ИГРЕК набирает целебный раствор в клизму и вкачивает его в мертвую
бабушку. Ее тело сводит страшная судорога и бабушка восстает из мертвых.
   </раздатка>
   Назовите любого из наших современников, упоминающихся в песне,
фрагмент из которой мы вам раздали.

Ответ:
Геннадий Малахов.

Зачет:
Доктор Попов; Елена Малышева.

Комментарий:
В песне группы "Бездна анального угнетения" "Бабушка вернулась из ада"
производится стеб над современной популярной народной медициной.

Источник:
"Бездна анального угнетения" - "Бабушка вернулась из ада".

Тур:
18 тур. "Почтосинус" (Москва)

Инфо:
Команда благодарит за тестирование и ценные замечания: Наталью Сычёву,
Екатерину Селиванову, Андрея Дукова, Марию Степанову, Ивана
Александрова, Галину Царегородцеву.

Вопрос 1:
Заголовком статьи, посвященной гибели круизного лайнера "Costa
Concordia", стало сложносочиненное предложение, вторая часть которого: -
"..., а у следователей - очень много". Напишите первую часть заголовка
статьи.

Ответ:
"У матросов нет вопросов".

Комментарий:
А у нас для вас еще найдется парочка вопросов.

Источник:
http://www.vesti.ru/doc.html?id=687000

Автор:
Виктория Полякова

Вопрос 2:
Лифт в общежитии МГУ "Дом аспиранта и стажера" на улице Шверника в шутку
называют The Лифт. Что мы заменили на "The"?

Ответ:
Das.

Зачет:
Дас; с любой капитализацией.

Комментарий:
Аббревиатура общежития - ДАС, а в вопросе мы заменили артикль из
английского языка артиклем из немецкого.

Источник:
ЛОАВ.

Автор:
Илья Кузнецов

Вопрос 3:
Название одной шуточной копии картины "Мона Лиза" получается заменой
одной буквы англоязычного названия двумя. Назовите материал,
использовавшийся при создании картины.

Ответ:
Монеты.

Зачет:
Деньги.

Комментарий:
"Money Lisa".

Источник:
http://pikabu.ru/story/money_lisa_3878186

Автор:
Александр Лавренов

Вопрос 4:
Королева из пьесы "Рюи Блаз" очень одинока и чувствует себя во дворце
словно погребенной заживо. Назовите актрису, которую прославила эта
роль.

Ответ:
Сара Бернар.

Комментарий:
Юная Бернар была одержима темой смерти; на известной фотографии она
запечатлена лежащей в гробу.

Источник:
http://ru.wikipedia.org/wiki/Бернар,_Сара

Автор:
Дмитрий Нагорный

Вопрос 5:
По одной из версий, это известное в Европе название появилось из-за
того, что одним из НИХ воспользовались для отравления монахов. ОНИ
описываются в учебниках по химии. Назовите ИХ двумя словами,
начинающимися на одну и ту же букву.

Ответ:
Свойства сурьмы.

Комментарий:
Согласно одной из версий происхождения английского, а также похожих
французского и немецкого названий сурьмы (antimony [Энтимони]), один из
монахов обнаружил слабительное действие сурьмы и предложил своим
собратьям кашу с сурьмой, в результате чего все монахи отравились и
умерли.

Источник:
http://www.chem.msu.su/rus/history/element/Sb.html

Автор:
Рената Дерябина

Вопрос 6:
В одной компьютерной игре главный герой приходит в сознание и узнаёт,
что лишился руки. Он вынужден сменить имя и отправиться в плавание,
чтобы отомстить своим врагам. Назовите его новое имя, восходящее к имени
древнего царя.

Ответ:
Ахав.

Зачет:
Ахаб; Ahab.

Комментарий:
В романе Германа Мелвилла "Моби Дик, или Белый кит" один из главных
героев, капитан китобойного судна "Пекод", носит имя Ахав.

Источник:
Игра "Metal Gear Solid 5".

Автор:
Дмитрий Нагорный

Вопрос 7:
Википедия утверждает, что после подавления восстания 1830-1831 годов
появился целый ряд опер, стихотворений, дум, драм, повестей и рассказов,
посвященных определенной теме. В 2003 году в окрестностях села Исупово
археологами были найдены сорок католических нательных крестов и один
православный. Напишите название статьи Википедии, являющейся источником
этого вопроса.

Ответ:
Иван Сусанин.

Зачет:
Сусанин Иван.

Комментарий:
В 1830-1831 годах было восстание в Польше, и требовалась "идеологическая
поддержка". Православный крестик близ села Исупово, кроме прочего, был
найден разрубленным.

Источник:
http://ru.wikipedia.org/wiki/Иван_Сусанин

Автор:
Виктория Полякова

Вопрос 8:
   <раздатка>
   Книга
   М. Норбекова
   имеет подзаголовок
   "Как избавиться от оков"
   </раздатка>
   В одном из слов на раздаточном материале мы пропустили одну букву.
Назовите одним словом то, что изображено на обложке книги.

Ответ:
Очки.

Комментарий:
(pic: 20160334.jpg)
   Обложка книга немного напоминает таблицу Сивцева.

Источник:
М.С. Норбеков. Опыт дурака, или Ключ к прозрению.

Автор:
Роман Царегородцев

Вопрос 9:
Дуплет.
   1. По одной из версий, прозвище [ПРОПУСК] король получил не за свое
благородство, а за жестокость, неистовство в бою и казнь тысяч
пленников, захваченных при осаде Акры. Заполните пропуск.
   2. В фильме с названием "[ПРОПУСК]" есть киноляп: предмет одежды, с
которым связан примечательный эпизод фильма, не существовал в XIII веке,
а появился лишь несколько столетий спустя. Заполните пропуск.

Ответ:
   1. Львиное сердце.
   2. Храброе сердце.

Комментарий:
   2. Шотландцы в фильме перед боем задирали вверх килты и
демонстрировали врагу причинные места. Согласно разным источникам, килт
появился лишь в XVI-XVII веках.

Источник:
   1. http://ru.wikipedia.org/wiki/Ричард_I_Львиное_Сердце
   2. http://www.tramvision.ru/lapsus/bh.htm

Автор:
Дмитрий Нагорный

Вопрос 10:
Заголовок обзора на сайте slon.ru, рассказывающего о проектах создания
летающих машин, содержит в себе первую часть секретной фразы из
произведения 1979 года. Напишите эту первую часть этой секретной фразы.

Ответ:
Земля, прощай!

Комментарий:
В статье "Земля, прощай" русскоязычное издание цитирует советский
мультфильм "Летучий корабль", сделанный по мотивам русской народной
сказки. Возможно, статья об успешных запусках будет называться "В добрый
путь".

Источник:
   1. https://slon.ru/posts/53215
   2. http://ru.wikipedia.org/wiki/Летучий_корабль_(мультфильм)

Автор:
Роман Царегородцев

Вопрос 11:
Отмечая архетипичность образов Мерлина и короля Артура, журнал "Variety"
отмечает, что они повлияли, в том числе, на ПЕРВОГО и ВТОРОГО. Образ
ПЕРВОГО был, в основном, импровизацией под влиянием личностей ученого
Альберта Эйнштейна и дирижера Леопольда Стоковского. Назовите
произведение, главными героями которого являются ПЕРВЫЙ и ВТОРОЙ.

Ответ:
"Назад в будущее".

Зачет:
"Назад в будущее - 2"; "Назад в будущее - 3".

Комментарий:
Молодой герой Марти Макфлай, получающий силу от опытного, мудрого дока
Брауна напомнили журналу Артура и Мерлина. Да, предыдущий вопрос тоже
связан с этим фильмом.

Источник:
http://ru.wikipedia.org/wiki/Назад_в_будущее_(фильм)

Автор:
Роман Царегородцев

Вопрос 12:
Снимок-победитель фотоконкурса "Земля как искусство" - изображение со
спутника, напоминающее это художественное произведение. Особенное
сходство придают массивные конгрегации зеленовато-белого фитопланктона,
которые циркулируют в темных водах Балтийского моря. Назовите эту
картину и ее автора.

Ответ:
"Звездная ночь", Ван Гог.

Источник:
http://lifeglobe.net/entry/3848

Автор:
Александр Лавренов

Вопрос 13:
По одной из версий, в Стоунхедже древние кельты воплотили свое вИдение
Вселенной, а мистерии, проходившие в нем, символизировали естественный
ход вещей - смены времен года, рождения и смерти. Из-за этого существует
мнение, что ЭТОТ ЧЕЛОВЕК заимствовал основы своих сюжетов в том числе и
из древнекельтских обрядов. Назовите ЭТОГО ЧЕЛОВЕКА.

Ответ:
[Уильям] Шекспир.

Комментарий:
По-видимому, фраза "Весь мир - театр" впервые прозвучала именно на
"подмостках" Стоунхенджа. Существует множество версий "настоящих"
авторов пьес Шекспира.

Источник:
П.Д. Волкова. Мост через бездну. Книга 1. Комментарий к античности.
http://www.flibusta.is/b/412379/read

Автор:
Евгения Дашкина

Вопрос 14:
Стиву Фоссетту удалось СДЕЛАТЬ ЭТО, использовав ТАКОЙ ИКС. Дикеарх,
рассуждая о возможности СДЕЛАТЬ ЭТО, упоминал ИКС. Назовите любого
россиянина, который СДЕЛАЛ ЭТО.

Ответ:
[Иван Федорович] Крузенштерн.

Зачет:
[Юрий Федорович] Лисянский; [Федор Филиппович] Конюхов; [Юрий
Алексеевич] Гагарин.

Комментарий:
СДЕЛАТЬ ЭТО - совершить кругосветное путешествие, ТАКОЙ ИКС - воздушный
шар. Дикеарх, живший в IV веке до нашей эры, считал, что земля имеет
форму шара.

Источник:
   1. http://ru.wikipedia.org/wiki/Фоссетт,_Стив
   2. http://ru.wikipedia.org/wiki/Дикеарх

Автор:
Анна Юдаева

Вопрос 15:
Жители северной Испании с 30-х годов прошлого века называют резиновые
сапоги русским женским именем. Это связано не с персонажем русской
песни, а с заглавной героиней одной малоизвестной оперетты. Назовите
любую из уменьшительных форм имени этой героини.

Ответ:
Катюшка.

Зачет:
Катя, Катька, Катенька, Катюша, Катерина и прочие производные от имени
"Екатерина".

Комментарий:
"Катюшка" - оперетта Пабло Соросабаля. Главная героиня оперетты носила
высокие резиновые сапоги, откуда и пошло прозвище "катюшки". Песня
"Катюша" Блантера и Исаковского была написана позже.

Источник:
   1. http://www.fds-net.ru/showflat.php?Cat=&Number=11321209&fullview=&src=&showlite=#Post11321209
   2. http://etimologias.dechile.net/?katiuskas

Автор:
Алексей Овчинников

Вопрос 16:
Сначала редактор заменил вторую часть этого вопроса. Потом заменил
первую. Потом убрал всё, оставив только ту информацию, которую вы сейчас
услышали. Напишите ответ на этот вопрос, состоящий из двух слов.

Ответ:
Корабль Тесея.

Зачет:
Парадокс Тесея.

Комментарий:
Если в вопросе заменить сначала одну часть, а потом другую - это будет
тот же вопрос или уже другой?

Источник:
ЛОАВ.

Автор:
Евгений Копейка, в редакции Романа Царегородцева :-)

Вопрос 17:
Один пользователь сайта chesspro, комментируя результаты молодых
шахматистов на рождественском турнире "Щелкунчик", назвал имена
ветеранов, которые "участвовали в раздаче". Назовите устойчивое
выражение из трех слов, которое было упомянуто в том же комментарии
ранее.

Ответ:
Получили на орехи.

Комментарий:
Молодые игроки проиграли ветеранам на турнире "Щелкунчик", так что
упоминание орехов неудивительно.

Источник:
http://chesspro.ru/guestnew/looknullmessage/?themeid=16&id=225&page=282#16-225-205166

Автор:
Роман Царегородцев

Вопрос 18:
Дуплет.
   1. В вопросе есть замена.
   Лирический герой песни группы "АЛЬФА" не против введения военного
положения. В следующей строчке упоминается АЛЬФА. Назовите АЛЬФУ двумя
словами.
   2. В вопросе есть замена.
   В куплете песни группы "БЕТА" упоминаются унылые усталые признанья на
крови, а в припеве - АЛЬФ. Назовите АЛЬФа одним словом.

Ответ:
   1. Гражданская оборона.
   2. Гроб.

Комментарий:
   1. В тексте песни группы "Гражданская оборона" есть слова "Да
здравствует гражданская оборона".
   2. Неудивительно, что в творчестве группы "Агата Кристи" встречаются
слова "любовь до гроба". "Гроб" - прозвище группы "Гражданская оборона".

Источник:
   1. http://www.gr-oborona.ru/texts/1056912669.html
   2. https://music.yandex.ru/album/10556/track/111850

Автор:
Илья Кузнецов

Вопрос 19:
В англоязычном комиксе утверждается, что Путин планирует употребить
индейку на День благодарения. Какое слово мы заменили в предыдущем
предложении?

Ответ:
Турцию.

Комментарий:
Слова "turkey" (индейка) и "Turkey" (Турция) в английском языке являются
омофонами. Конфликт с соседним государством пришелся аккурат перед
американским Днем благодарения.

Источник:
http://www.dailymail.co.uk/news/article-3334787/Looks-like-Putin-having-Turkey-Thanksgiving-Internet-pranksters-turn-eye-Russia-s-diplomatic-crisis-brilliant-memes.html

Автор:
Александр Лавренов

Вопрос 20:
В 93-м эпизоде мультсериала "Смешарики" оказывается, что ИКС здесь
является ТАКИМ. Официальная библиография ТАКОГО ИКСА включает в себя
сорок одну книгу и вряд ли будет увеличена. Какие два слова мы заменили
на "ТАКОЙ ИКС"?

Ответ:
Плоский мир.

Комментарий:
В мире Смешариков всё круглое, а сам мир плоский и покоится на черепахе.

Источник:
   1. http://www.youtube.com/watch?v=ptOc6UfJY4c
   2. http://ru.wikipedia.org/wiki/Плоский_мир

Автор:
Роман Царегородцев

Вопрос 21:
В новости, опубликованной на сайте любителей ужасов в январе 2016 года,
самым страшным серийным убийцей был назван ИКС. Митио Каку утверждает,
что ИКС останется непобедимым как минимум до 2100 года. Что мы заменили
на ИКС?

Ответ:
Рак.

Комментарий:
Новость была о смерти Алана Рикмана. Митио Каку, футуролог и
популяризатор науки, считает, что рак останется неизлечимым еще очень
долго.

Источник:
   1. http://horrorzone.ru/page/rip-alan-rikman-1946-2016
   2. М. Каку. Физика будущего. http://www.flibusta.is/b/299229/read

Автор:
Илья Кузнецов

Вопрос 22:
(pic: 20160335.jpg)
   Первое имя персонажа на раздаточном материале - Харли. Назовите
группу, песня которой исполняется в трейлере к фильму "Отряд самоубийц",
одним из героев которого является этот персонаж.

Ответ:
"Queen".

Комментарий:
Полное имя персонажа - Харли Квин (Harley Quinn). Наряд персонажа
напоминает наряд Арлекина.

Источник:
   1. http://ru.wikipedia.org/wiki/Харли_Квинн
   2. http://ru.wikipedia.org/wiki/Отряд_самоубийц_(фильм,_2016)

Автор:
Рената Дерябина

Вопрос 23:
По дороге на марафон автор вопроса оценил блеск гораздо ниже, чем
нищету. Назовите, использовав заимствованное слово, то, чем занимался
автор в пути.

Ответ:
Играл в "Эрудит".

Зачет:
Играл в "Scrabble"; занимался подсчетом очков в "Эрудит" / "Scrabble".

Комментарий:
В игре "Scrabble" нищета стоит 15 очков без удвоений, а блеск - всего 9.

Источник:
ЛОАВ.

Автор:
???

Вопрос 24:
(pic: 20160336.jpg) (pic: 20160337.jpg)
   Восстановите текст, скрытый на втором изображении.

Ответ:
Цена 2 копейки.

Зачет:
2 копейки; две копейки.

Комментарий:
На первом изображении гибрид, составленный из двух автомобилей
"копейка". Второе изображение - стандартная тетрадь времен СССР.

Источник:
   1. http://demotivators.to/p/52434/dve-kopejki.htm
   2. http://jurashz.livejournal.com/1968649.html

Автор:
Александр Лавренов

Тур:
19 тур. "Риббл Страйп" (Санкт-Петербург - Краснодар - Волгоград)

Редактор:
Олег Холодов (Серпухов)

Вопрос 1:
Паоло Мориджиа отмечает: "Арчимбольдо жил достойной и НАСЫЩЕННОЙ жизнью
при императорском дворе". Какое прилагательное с двумя корнями было
заменено на "НАСЫЩЕННОЙ"?

Ответ:
Плодотворной.

Комментарий:
Художник эпохи Возрождения Джузеппе Арчимбольдо в прямом смысле творил
плоды, создавая свои знаменитые картины: то ли портреты, то ли
натюрморты. Мы надеемся, что и игра на нашем туре будет для вас
плодотворной.

Источник:
http://ru.wikipedia.org/wiki/Арчимбольдо,_Джузеппе

Автор:
Максим Киселёв (Санкт-Петербург)

Вопрос 2:
Недавно в ресторане на Ленинградском вокзале появился необычный аппарат.
Люди подходили к нему и пытались произнести определенное слово. В случае
неудачи они получали из аппарата плюшевого медведя, игрушечную пчелу,
накладную бороду. А что они хотели получить?

Ответ:
Пиво.

Зачет:
Бутылку пива.

Комментарий:
Надо было правильно произнести слово "beer", после чего приз - бутылка
"Балтики N 7" - появлялся в специальном проеме. При этом на созвучные
слова вроде "bear", "bee" или "beard" автомат реагировал соответствующим
образом - выдавал плюшевого медведя, игрушечную пчелу или накладную
бороду. В автомате сидел человек, который определял правильность
произношения.

Источник:
http://www.gorobzor.ru/newsline/proisshestviya/associaciya-lyubiteley-piva-podderzhivaet-akciyu-v-tgif-10-04-2014

Автор:
Ольга Мордвина-Щодро (Санкт-Петербург)

Вопрос 3:
Внимание, в вопросе есть замена.
   В 1976 году, после ежегодного съезда американской организации
ветеранов войны, была зафиксирована вспышка тяжелого инфекционного
заболевания. Имя этого заболевания - [ПРОПУСК]. Заполните пропуск.

Ответ:
Болезнь легионеров.

Зачет:
Легионеллёз; питтсбургская пневмония, понтиакская лихорадка (с
отвращением).

Комментарий:
Организация ветеранов войны в США называется Американский Легион. Из-за
заболевания множества членов организации болезнь получила название
Болезнь легионеров. Слова "имя этого заболевания" - намек на цитату из
Евангелия от Марка "Имя мне - легион".

Источник:
http://en.wikipedia.org/wiki/Legionnaires%27_disease

Автор:
Максим Киселёв (Санкт-Петербург)

Вопрос 4:
В современном произведении в образе президента и диктатора Франции
предстает ИКС, который, по словам рассказчика, в какой-то момент понял,
что "люди - более совершенный материал, чем кирпичи". Назовите ИКСА.

Ответ:
Ле Корбюзье.

Комментарий:
Ле Корбюзье (настоящее имя - Шарль-Эдуар Жаннере-Гри) - один из
выдающихся швейцарских архитекторов, много работавший во Франции. Пионер
архитектурного модернизма и функционализма. Вспомним классика: "У
Корбюзье то общее с Люфтваффе, что оба потрудились от души над переменой
облика Европы".

Источник:
   1. http://fai.org.ru/forum/topic/11451-velichayshiy-diktator-ai-rasskaz/
   2. http://ru.wikipedia.org/wiki/Ле_Корбюзье

Автор:
Максим Киселёв (Санкт-Петербург)

Вопрос 5:
На одном гобелене из цикла "Дама с единорогом" можно увидеть зеркало, на
другом - оргАн, на третьем - бонбоньерку. Согласно распространенной
трактовке, пять из шести гобеленов символизируют ИХ. Назовите ИХ.

Ответ:
Пять чувств.

Зачет:
Чувства.

Комментарий:
Зеркало символизирует зрение, оргАн - слух, бонбоньерка - вкус. Обоняние
передается на одной из оставшихся картин изображением обезьяны с
цветком, а осязание - прикосновением дамы к рогу единорога.

Источник:
http://ru.wikipedia.org/wiki/Дама_с_единорогом

Автор:
Мария Подрядчикова (Волгоград)

Вопрос 6:
Петр Рябов говорит о том, что великие открытия Блеза Паскаля, например,
арифметическая счетная машина или теория вероятности, появились
благодаря бытовым потребностям - желанию помочь отцу с расчетами и
интересу к карточной игре. Далее Рябов цитирует первую строчку из
произведения 1940 года. Воспроизведите эту строчку.

Ответ:
"Когда б вы знали, из какого сора...".

Комментарий:
Как и стихи, великие открытия тоже иногда рождаются благодаря не слишком
заметным событиям.

Источник:
   1. П. Рябов. Курс лекций об экзистенциализме. Паскаль. 33-я минута.
   2. http://www.akhmatova.org/verses/verses/397.htm

Автор:
Мария Подрядчикова (Волгоград)

Вопрос 7:
Манильскую бумагу обычно не красят, и она сохраняет естественный
желтоватый цвет волокон. Предмет, который в Америке часто делали из
манильской бумаги, был непохож на аналогичные в СССР, поэтому советским
специалистам казался абстрактным символ, изображающий... Что?

Ответ:
Папку.

Зачет:
Директорию; каталог.

Комментарий:
(pic: 20160338.jpg)
   Как вы можете видеть, стандартный вид американских и советских папок
сильно различался, поэтому для первых пользователей значок папки мог
показаться абстрактным.

Источник:
http://rogix.livejournal.com/156774.html

Автор:
Мария Подрядчикова (Волгоград)

Вопрос 8:
Надеемся, этот вопрос не покажется вам сложным.
   Паскаль Брюкнер замечает, что мудрость Далай-Ламы доступна и понятна
публике по всему миру. Какое слово, вошедшее в обиход в конце XIX века,
при этом упоминается?

Ответ:
Эсперанто.

Комментарий:
Упрощенную восточную мудрость, универсальную и легкую для усвоения
любыми народами, Паскаль Брюкнер назвал "духовным эсперанто" новейшей
истории. Язык эсперанто был создан в конце XIX века Людвигом Заменгофом.
Само слово "эсперанто" означает "надеющийся".

Источник:
   1. http://wildsent.livejournal.com/26882.html
   2. http://ru.wikipedia.org/wiki/Эсперанто

Автор:
Владислав Декалов (Петергоф)

Вопрос 9:
Согласно автору критической статьи, в фильме "Китайский связной" вместо
структуры сюжета - ОНА поединков. Вы сможете увидеть ЕЕ уже в перерыве.
Назовите ЕЕ двумя словами, начинающимися на одну и ту же букву.

Ответ:
Турнирная таблица.

Комментарий:
"Китайский связной" - один из первых фильмов с Брюсом Ли. Как и многие
фильмы о кунг-фу, он чуть более чем полностью состоит из драк. Турнирную
таблицу сегодняшней игры вы сможете увидеть в перерыве.

Источник:
http://vozduh.afisha.ru/cinema/istoriya-gonkongskogo-kino-v-20-filmah/

Автор:
Александр Усков (Краснодар)

Вопрос 10:
Прослушайте список:
   Миревельт, 1617 - Виллема ван дер Меера;
   Баккер, 1670 - Фредерика Рюйша;
   Пикеной, 1619 - Эгбертса;
   Троост, 1728 - Виллема Рюэлла.
   БОльшую часть самого известного варианта занимает Адриаан Адриаанс по
прозвищу Арис Киндт. Какие три слова мы несколько раз пропустили в
тексте вопроса?

Ответ:
Урок анатомии доктора.

Комментарий:
Как известно, аутопсия была популярным сюжетом для картин эпохи позднего
Возрождения и раннего Нового времени. Перечислены и другие известные
полотна различных художников.

Источник:
   1. http://dep_anatom.pnzgu.ru/page/14038
   2. http://ru.wikipedia.org/wiki/Урок_анатомии_доктора_Тульпа

Автор:
Илья Кукушкин (Краснодар)

Вопрос 11:
Событие в ходе восстания ихэтуаней, после которого 222 православных
китайца было канонизировано, также называют ЕЮ в Пекине. Назовите ЕЕ
двумя словами.

Ответ:
Варфоломеевская ночь.

Комментарий:
В ходе восстания было и массовое избиение христиан, после которого
погибших из-за христианской веры причислили к мученикам. Варфоломеевская
ночь - пожалуй, наиболее известное столкновение конфессий, отличающееся
особой жестокостью.

Источник:
   1. http://azbyka.ru/sobor-svyatyx-muchenikov-kitajskix
   2. http://ru.wikipedia.org/wiki/Варфоломеевская_ночь_в_Пекине

Автор:
Мария Подрядчикова (Волгоград)

Вопрос 12:
Синто Бесард ослеп в возрасте 41 года, однако это не помешало ему обойти
на яхте практически весь мир. Статья о нем в журнале "Esquire"
озаглавлена "При любых условиях". В предыдущем предложении мы заменили
несколько слов. Напишите их в исходном виде.

Ответ:
"Не смотря ни на что".

Зачет:
"Несмотря ни на что".

Комментарий:
Несколько цинично, но с верой в безграничные возможности человека.

Источник:
http://esquire.ru/photo/blind-captain

Автор:
Александр Усков (Краснодар)

Вопрос 13:
[Ведущему: не озвучивать капитализацию в слове "Чуму".]
   В энциклопедии "Луркмор" называются атрибуты одного, так сказать,
"мифического персонажа": "кавайность", "склонность к экстремальным
путешествиям" и, собственно, ИКС. Считается, что ИКС переносит Чуму.
Назовите ИКСА абсолютно точно.

Ответ:
Белый конь. Незачет: Бледный конь; конь.

Комментарий:
Статья называется "Принц на белом коне". Действительно, он должен быть
милым (кавайным в терминологии "Луркмора") и стремиться к приключениям.
И, разумеется, иметь белого коня. В "Откровении Иоанна Богослова"
упоминаются четыре всадника Апокалипсиса, первый из которых - всадник на
белом коне - имеет имена Чума и Мор.

Источник:
   1. http://lurkmore.co/Принц_на_белом_коне
   2. http://ru.wikipedia.org/wiki/Четыре_всадника_Апокалипсиса

Автор:
Максим Киселёв (Санкт-Петербург)

Вопрос 14:
Говоря об эсхатологической роли Дмитрия Киселёва на российском
телевидении, автор сообщества "Под корень" особенно отмечает, что тот
является специалистом по скандинавской литературе. Поэтому каждый выпуск
программы "Вести недели", можно сказать, ДЕЛАЕТ ЭТО. Какие два слова,
начинающиеся на соседние буквы, мы заменили на "ДЕЛАТЬ ЭТО"?

Ответ:
Приближать Рагнарёк.

Комментарий:
Раз мы заговорили об Апокалипсисе, то можно вспомнить и о
древнескандинавских представлениях о конце света (что, собственно,
эсхатология и подразумевает). А еще Киселёв - это симулякр.

Источник:
   1. https://vk.com/podkoren?w=wall-60854067_25655
   2. http://ru.wikipedia.org/wiki/Эсхатология

Автор:
Владислав Декалов (Петергоф)

Вопрос 15:
(pic: 20160339.jpg)
   Назовите одиннадцатибуквенным словом то, с чем Жан Бодрийяр призывал
бороться, когда власть, переставая быть собой, превращается в симуляцию.

Ответ:
Вероломство.

Комментарий:
Известна картина Рене Магритта "Вероломство образов" ("Это не трубка").
Вероломство образов власти и мира в целом - одна из центральных тем
самой известной работы Бодрийяра "Симулякры и симуляции" (которой
вдохновлялись братья Вачовски, создавая "Матрицу"). Курящего трубку
Бодрийяра мы не нашли, поэтому на раздатке не Бодрийяр, а Жак Деррида.

Источник:
Ж. Бодрийяр. Симулякры и симуляции.

Автор:
Владислав Декалов (Петергоф)

Вопрос 16:
Изначально автор решил дать это название книге о посещении им Алеппо,
Бейрута, Дамаска, Каира, Константинополя, Медины и Смирны. Напишите это
название из трех слов.

Ответ:
"Семь столпов мудрости".

Комментарий:
Лоуренс Аравийский - английский герой времен арабских восстаний 1910-х
годов - решил написать книгу о семи восточных городах, название для
которой взял из библейской Книги Притч - "Семь столпов мудрости".
Впоследствии от идеи написания этой книги он отказался, но название
сохранил для своей автобиографии.

Источник:
http://ru.wikipedia.org/wiki/Семь_столпов_мудрости

Автор:
Максим Киселёв (Санкт-Петербург)

Вопрос 17:
[Ведущему: выделить голосом слово "непонимание".]
   Игорь Дюшен отмечает обреченность героев Беккета и их трагическое
непонимание собственной судьбы. По его словам, "ужас, а в конце концов и
смерть ждут каждого". Дальнейшая мысль Дюшена почти в точности повторяет
строку из произведения 1968 года. Назовите автора этого произведения.

Ответ:
Иосиф Бродский.

Комментарий:
"Ужас, а в конце концов и смерть ждут каждого. Ирония заключается в том,
что люди знают это только о других". Известна строка Иосифа Бродского из
стихотворения "Памяти Т.Б.": "Смерть - это то, что бывает с другими".

Источник:
   1. Театр парадокса (Ионеско, Беккет и другие): Сб. / Сост. и автор
предисл. И. Дюшен. - М.: Искусство, 1991. - с. 15.
   2. http://www.world-art.ru/lyric/lyric.php?id=7593

Автор:
Владислав Декалов (Петергоф)

Вопрос 18:
В каком экранизированном произведении 1993 года совсем юная девушка
носит браслеты, подклеенные к запястьям липкой лентой?

Ответ:
"Девственницы-самоубийцы".

Комментарий:
Сцена происходит вскоре после неудачной попытки самоубийства, и девушка
скрывает следы глубоких порезов на запястьях. "Девственницы-самоубийцы"
- роман Джеффри Евгенидиса, экранизированный Софией Копполой.

Источник:
Дж. Евгенидис. Девственницы-самоубийцы.
http://www.flibusta.is/b/167172/read

Автор:
Мария Подрядчикова (Волгоград)

Вопрос 19:
Действие фильма "Красотки" происходит в том числе и в России. В одной из
сцен главный герой говорит: "Я думал обо всех девушках, которых знал...
Я говорил себе, что они как [ПРОПУСК]: любопытно узнать, которая будет
последней". Заполните пропуск словом во множественном числе.

Ответ:
Матрешки.

Комментарий:
Оригинальное название фильма - "Les poup&eacute;es russes" - так и
переводится: "Матрешки". Герой пытается найти идеальную девушку, и, как
будто раскрывая матрешки, стремится к той, после которой поиск будет
завершен. Упоминание России здесь вовсе не случайно.

Источник:
http://ru.wikipedia.org/wiki/Красотки_(фильм,_2005)

Автор:
Ольга Мордвина-Щодро (Санкт-Петербург)

Вопрос 20:
(pic: 20160340.jpg)
   На творчество израильского художника Йосля Бергнера очень сильно
повлияло то, что в детстве отец читал ему сказку на ночь. Какое слово мы
заменили в предыдущем предложении?

Ответ:
Кафку.

Комментарий:
Отец Йосля был переводчиком на идиш и переводил именно Кафку, с
творчеством которого и знакомил сына. Художник впоследствии стал
известным иллюстратором писателя, а в отдельных работах воплотил гротеск
и ужас, присущий ему.

Источник:
http://www.culbyt.com/article/textid:569/

Автор:
Анастасия Белозёрова (Санкт-Петербург)

Вопрос 21:
Персонаж одного романа пытается купить билеты до города Принс-Альберт,
но не может сделать этого, так как к билетам необходимо иметь разрешение
на выезд от своего полицейского участка. Для получения разрешения
необходимо было заполнить бланк, отнести в специальную комнату и ожидать
срок до двух месяцев. Назовите короткую фамилию этого персонажа.

Ответ:
К.

Комментарий:
Многие усматривают параллели между персонажами Кафки (К. и Йозеф К.) и
персонажем южноафриканского писателя Джона Кутзее Михаэлем К из романа
"Жизнь и время Михаэля К". Как вы могли заметить, последний также
сталкивался с абсурдностью бюрократического ада.

Источник:
http://en.wikipedia.org/wiki/Life_%26_Times_of_Michael_K

Автор:
Владислав Декалов (Петергоф)

Вопрос 22:
Парапсихологи считают, что если при проведении посвященного телепатии
эксперимента присутствует скептик, его скептицизм будет гасить
проявления пси-энергии. Мартин Гарднер сравнивает это оправдание с НЕЙ.
Назовите ЕЕ.

Ответ:
Уловка-22.

Комментарий:
Таким образом, парапсихологи заведомо ставят себя в выигрышную ситуацию,
где каждый наблюдатель, не верящий эксперименту, не может быть
наблюдателем из-за того, что он эксперименту мешает. Уловка-22 - пример
бюрократической логики, согласно которой каждый, кто объявляет себя
сумасшедшим, им не является, так как только человек в здравом уме может
сделать такое умозаключение.

Источник:
М. Гарднер. Когда ты была рыбкой, головастиком - я...
http://www.flibusta.is/b/312111/read

Автор:
Мария Подрядчикова (Волгоград)

Вопрос 23:
Внимание, в вопросе есть замены.
   По мнению литературоведа Захария Плавскина, герои Бенито Переса
Гальдоса пытаются противопоставить жизни свой далекий от реальности
идеал. Поэтому отнюдь не случайно одного из таких героев называют
"ПЕРВЫМ и ВТОРЫМ". Напишите произошедшее от топонима прилагательное,
которое мы заменили на слово "ВТОРОЙ".

Ответ:
Ламанчский.

Комментарий:
Гальдос - испанец, поэтому текстуальные совпадения с главным испанским
романом совсем не случайны. Одного из персонажей, немного спятившего
священника Назарина, называют "арабским и ламанчским", совсем как
мнимого летописца подвигов Дон Кихота Сида Ахмеда Бен-Инхали.

Источник:
Б. Перес Гальдос. Тристана; Назарин; Милосердие. - Л.: Художественная
литература, 1987. - С. 8.

Автор:
Владислав Декалов (Петергоф)

Вопрос 24:
Надеемся, что наши вопросы вам понравились.
   Прослушайте отрывок из стихотворения Сергея Чекмарёва, комсомольца
тридцатых годов:
   "Я буду там, где должен быть,
   Куда [ДВА СЛОВА ПРОПУЩЕНО],
   Но мне нигде не позабыть
   Сиянья серых глаз".
   Восстановите пропуск.

Ответ:
"... поставит класс...".

Зачет:
"... поставил класс...".

Комментарий:
Вот такое сочетание лирики и патриотического долга. Это вам не лайки
ставить. Верим, что и нашему туру вы поставите "класс", как это делают
пользователи социальной сети "Одноклассники".

Источник:
http://www.peoples.ru/art/literature/poetry/newtime/sergey_checmarev/

Автор:
Мария Подрядчикова (Волгоград)

Тур:
20 тур. "Любовь" (Калуга)

Редактор:
Олег Холодов (Серпухов)

Вопрос 1:
На одном из первенств Оренбурга по шахматам девочка упрямо не хотела
проигрывать, поэтому в эндшпиле ДЕЛАЛА ЭТО. "ДЕЛАЕМ ЭТО" - приличный
вариант одной из речёвок команды "Любовь". Какие три слова мы заменили
словами "ДЕЛАТЬ ЭТО"?

Ответ:
Играть без короля.

Источник:
   1. http://www.onlineon.ru/2013-02-28/article/7692/
   2. ЛОАВ.

Автор:
Алексей Пономарёв (Калуга)

Вопрос 2:
Дуплет.
   1. Говоря о пловцовских навыках, тренер однажды сравнил Сергея
Довлатова с НИМ. Назовите ЕГО одним словом.
   2. Будучи в эмиграции, отличавшийся высоким ростом Сергей Довлатов
однажды сравнил себя с НИМ. Назовите ЕГО двумя словами.

Ответ:
   1. Утюг.
   2. Небоскреб "Утюг".

Источник:
   1. С.Д. Довлатов. Соло на IBM. http://www.flibusta.is/b/129611/read
   2. С.Д. Довлатов. Ремесло. http://www.flibusta.is/b/164788/read

Автор:
Денис Карабанов (Калуга)

Вопрос 3:
Согласно образному выражению Бориса Стругацкого, он продолжал работать
ТАКИМ инструментом и после 12 октября 1991 года. В старину на Руси
массивные восьмикилограммовые кирпичи называли ТАКИМИ. Какими - ТАКИМИ?

Ответ:
Двуручными.

Комментарий:
После смерти брата-писателя в 1991 году, Борис Стругацкий продолжил
пилить бревно литературы двуручной пилой без напарника. Из-за размеров и
массы восьмикилограммовые кирпичи поднимали двумя руками.

Источник:
   1. http://ru.wikipedia.org/wiki/Стругацкий,_Борис_Натанович
   2. http://ru.wikipedia.org/wiki/Стругацкий,_Аркадий_Натанович
   3. http://www.ist-konkurs.ru/raboty/2012/772-historykirpi4

Автор:
Алексей Пономарёв (Калуга)

Вопрос 4:
В фильме "Звуки шума" полиция оцепила брошенный эксцентричными
хулиганами-музыкантами возле посольства автомобиль. Вскоре в автомобиле
был найден он. Напишите шесть букв, которые мы пропустили в тексте
вопроса.

Ответ:
м, е, т, р, о, м.

Источник:
Х/ф "Звуки шума" (2010), реж. Ула Симонссон, Юханнес Шерне Нильссон.

Автор:
Денис Карабанов (Калуга)

Вопрос 5:
Композитор Жан-Батист Люлли, дирижируя оркестром и отбивая тяжелой
тростью ритм, поранил себе ногу и спустя некоторое время умер от
гангрены. В результате осталась незаконченной музыкальная трагедия.
Назовите заглавного героя этой трагедии.

Ответ:
Ахилл.

Источник:
http://ru.wikipedia.org/wiki/Люлли,_Жан-Батист

Автор:
Даниил Марченко (Калуга)

Вопрос 6:
Кот автора вопроса обладает незаурядными акробатическими способностями и
может брать, казалось бы, непреодолимые высоты, цепляясь передними
лапами и затем вытягивая всё тело. Несмотря на то что кот беспородный,
автор вопроса сравнил его... Ответьте двумя словами, начинающимися на
одну и ту же букву: с кем?

Ответ:
Принц Персии.

Комментарий:
Автор сравнил кота с персонажем культовой компьютерной игры, хотя кот и
не персидской породы.

Источник:
ЛОАВ.

Автор:
Денис Карабанов (Калуга)

Вопрос 7:
Аппликатура - способ расположения и порядок чередования пальцев при игре
на музыкальном инструменте. Назовите человека, которому с 1818 года
приходилось пользоваться особой аппликатурой.

Ответ:
Грибоедов.

Комментарий:
Александр Якубович во время дуэли прострелил Александру Грибоедову кисть
левой руки. Грибоедов, как известно, был неплохим пианистом и сочинял
музыку - его вальсы вошли в отечественную классику.

Источник:
А.С. Грибоедов в воспоминаниях современников.
http://az.lib.ru/g/griboedow_a_s/text_0060.shtml

Автор:
Даниил Марченко (Калуга)

Вопрос 8:
Согласно американской шуточной истории философии, Платон СДЕЛАЛ ЭТО,
находясь в пещере. Назовите имя того, кто регулярно ДЕЛАЕТ ЭТО уже более
века.

Ответ:
Фил.

Комментарий:
Платон в пещере увидел свою тень и, подобно сурку Филу, предсказал
долгую зиму.

Источник:
   1. Т. Каткарт, Д. Клейн. Как-то раз Платон зашел в бар... Понимание
философии через шутки. http://www.flibusta.is/b/294585/read
   2. http://ru.wikipedia.org/wiki/Фил_(сурок)

Автор:
Даниил Марченко (Калуга)

Вопрос 9:
[Ведущему: первые два слова вопроса произнести без паузы.]
   "Интересно" - содержание первого в истории комментария на Youtube.
Воспроизведите этот комментарий на русском или английском языках.

Ответ:
Интересно.

Зачет:
Interesting.

Источник:
http://www.youtube.com/watch?v=jNQXAC9IVRw

Автор:
Сергей Осипов (Калуга)

Вопрос 10:
Российский политический плакат 1904 года в сатирическом тоне показывает,
что военные действия японцев против России - не что иное, как стремление
получить финансовую помощь США. Поэтому в названии плаката ПЕРВЫЙ
переименован во ВТОРОЕ. Назовите ПЕРВЫЙ и ВТОРОЕ.

Ответ:
Порт-Артур, Порт-Моне.

Зачет:
Порт-Артур, Портмоне.

Комментарий:
Атака на Порт-Артур - основное событие Русско-японской войны.

Источник:
http://russiahistory.ru/russkie-plakaty-perioda-russko-yaponskoj-vojny/

Автор:
Алексей Пономарёв (Калуга)

Вопрос 11:
Летом 1917 года Корнилов с несколькими соратниками попытались установить
в России военную диктатуру. Рассказывая об этом, Александр Евдокимов
пишет, что общими усилиями основные конкуренты в борьбе за власть
урезонили будущих [ПРОПУСК 1], возомнивших себя [ПРОПУСК 2]. Заполните
каждый из пропусков двумя словами.

Ответ:
Белые генералы, черные полковники.

Зачет:
Белые офицеры, черные полковники.

Источник:
http://www.kprf.ru/history/soviet/147749.html

Автор:
Евгений Черепанов (Калуга)

Вопрос 12:
Говоря о важности независимого мнения, герой телепередачи "Тем временем"
подчеркивает, что "своя колокольня" в лучшую сторону отличается от
АЛЬФЫ. Флобер писал об АЛЬФЕ: "Там порой холодно, зато звезды светят
ярче и не слышишь дураков". Назовите АЛЬФУ четырьмя словами.

Ответ:
Башня из слоновой кости.

Источник:
   1. Телепередача "Тем временем".
   2. http://ru.wikipedia.org/wiki/Башня_из_слоновой_кости

Автор:
Денис Карабанов (Калуга)

Вопрос 13:
Во время "Своей игры" на раздевание калужских знатоков наиболее
неудачливым юношам посоветовали вспомнить эпизод из биографии известного
коллектива. Назовите этот коллектив.

Ответ:
"Red Hot Chili Peppers".

Комментарий:
Имеется в виду эпизод, когда участники группы вышли на сцену в носке на
голое тело.

Источник:
   1. http://dyadya-lyosha.livejournal.com/6564.html
   2. http://ru-tay.ucoz.ru/news/red_hot_chili_peppers/2011-11-16-28

Автор:
???

Вопрос 14:
В песне "Судьба резидента" у страдающего от одиночества героя стерлись
ОНИ. Назовите ИХ.

Ответ:
Линии на руках.

Зачет:
Папиллярные линии.

Источник:
http://www.pripev.ru/Text.aspx?SongId=2923

Автор:
???

Вопрос 15:
В 1958 году Роберт Столлер ввел в науку термин "gen", который
противопоставил термину "sex". Напишите три буквы, которые мы пропустили
в предыдущем предложении.

Ответ:
der.

Зачет:
дер.

Источник:
http://www.krugosvet.ru/enc/gumanitarnye_nauki/sociologiya/GENDERNIE_ISSLEDOVANIYA.html

Автор:
Даниил Марченко (Калуга)

Вопрос 16:
Внимание, в вопросе есть замена.
   Странам, проводящим плохую экономическую политику, а именно
Португалии, Италии, Греции и Испании, в Европе придумали прозвище
"АЛЬФЫ". По преданию, Сатана окропил кровью АЛЬФЫ корни некоего растения
в последнюю очередь. Назовите это растение.

Ответ:
Виноград.

Комментарий:
Аббревиатура PIGS, составленная из первых букв названия стран,
переводится как "свиньи". Существует предание, что, когда Ной посадил
виноградную лозу, Сатана окропил корни винограда кровью овцы, льва,
обезьяны и свиньи, которым и уподобляется пьяница.

Источник:
   1. http://ru.wikipedia.org/wiki/PIGS
   2. http://toldot.ru/tora/articles/articles_1436.html

Автор:
Алексей Пономарёв (Калуга)

Вопрос 17:
Дуплет.
   1. В сериале "Школа" учительнице Валентине Харитоновне ученики дали
кличку. Какую?
   2. В Сериале "Солдаты" старослужащему, однофамильцу известного поэта
и декабриста, солдаты дали кличку. Какую?

Ответ:
   1. Харя.
   2. Рыло.

Источник:
   1. Сериал "Школа".
   2. Сериал "Солдаты".

Автор:
Денис Карабанов (Калуга)

Вопрос 18:
Известно, что заражение АЛЬФОЙ ИКСОВ происходит через зооспоры грибка
Aphanomyces astaci [афаномицес астаци]. Большую роль в распространении
заболевания в России играли заграничные экспортёры, которые продавали
заведомо зараженные ловушки для ИКСОВ. ИКС часто называют АЛЬФОЙ XXI
века. Что мы заменили на ИКС и на АЛЬФУ?

Ответ:
Рак, чума.

Комментарий:
Речь идет о рачьей чуме. Известен случай непосредственного занесения
заразы из Подольской губернии в бассейн реки Оки вместе с транспортом
больных раков.

Источник:
http://ru.wikipedia.org/wiki/Чума_раков

Автор:
Денис Карабанов (Калуга)

Вопрос 19:
Болезнь, которую изучал Жюль БордЕ, имеет характерный симптом: серия
коротких кашлевых толчков заканчивается протяжным звуком. Болезнь
получила название от БАРАША. Какое слово мы заменили на слово "БАРАШ"?

Ответ:
Петух.

Комментарий:
Симптом напоминает крик петуха. "Петух" по-французски - "кок", болезнь -
"коклЮш". Жюль БордЕ открыл возбудителя этой болезни, а его французская
фамилия служит подсказкой. "Бараш" и "петух" - синонимичные понятия
уголовного жаргона.

Источник:
   1. http://www.travelexpress.lt/infekcionnyje-bolezni/kokliush-i-parakokliush.html
   2. http://ru.wikipedia.org/wiki/Опущенный

Автор:
???

Вопрос 20:
Девочки народности "невары" проходят обряд бракосочетания с деревом,
которое символизирует индуистского бога. Брак не мешает впоследствии
выйти замуж за обычного мужчину. В прошлом такой обряд спасал замужних
женщин... От чего?

Ответ:
Сожжения [на погребальном костре мужа].

Зачет:
Самосожжения; сати.

Комментарий:
После смерти мужа женщина номинально оставалась супругой бога.

Источник:
"Вокруг света", 2015, N 3. - С. 35, 48.

Автор:
Алексей Пономарёв (Калуга)

Вопрос 21:
(pic: 20160341.jpg)
   Название группы "АЛЬФА" было услышано основателем после просмотра
фильма, где переводчик перевел причинное место как АЛЬФА. Можно сказать,
что на розданной вами картинке присутствует АЛЬФА. Назовите АЛЬФУ двумя
словами.

Ответ:
Ночная трость.

Источник:
   1. http://www.konstantin-stupin.ru/биография/
   2. http://echo.msk.ru/blog/mihailos/821518-echo/

Автор:
???

Вопрос 22:
Статья о том, какие сложности ожидают представителя ЛГБТ-сообщества при
поиске работы, называется "ТАКИЕ ОНИ". Что мы заменили на "ТАКИЕ ОНИ"?

Ответ:
Нерадужные преспективы.

Источник:
http://ibigdan.livejournal.com/17027932.html

Автор:
Станислав Манихин (Калуга)

Вопрос 23:
Ведущий программы "Человек мира" Андрей Понкратов сообщает, что гейши
появились в Японии в XVII веке, и после этого просит не использовать в
отношении гейш... Какое словосочетание?

Ответ:
Древнейшая профессия.

Зачет:
Старейшая профессия; древняя профессия.

Источник:
"Путешествие дилетанта", телеканал "Россия 2", эфир от 24.07.2014 г.

Автор:
Алексей Пономарёв (Калуга)

Вопрос 24:
Этот вопрос мы адресуем следующим дежурным командам.
   Акция, которая называлась на Украине "мусорной люстрацией", в
иностранной прессе получила название "Trash ПРОПУСК" [трэш пропуск].
Заполните пропуск двумя английскими словами.

Ответ:
Bucket Challenge.

Комментарий:
Этим вопросом мы передаем эстафету трем следующим дежурным командам.

Источник:
http://ru.wikipedia.org/wiki/Ice_Bucket_Challenge

Автор:
Алексей Пономарёв (Калуга)

Тур:
21 тур. "ЗаПущиная наука" (Пущино)

Редактор:
Евгений Кононенко

Вопрос 1:
   <раздатка>
   - Я пришел драться с великаном, а Вы человек ниже среднего роста!
   <...>
   - Но я действительно великан. Хотя в это трудно поверить, но это так.
Просто я [ПРОПУСК].
   - Каким образом?!
   - Я вам могу рассказать.
   - Только покороче.
   - Пожалуйста, если можно, рюмку вина.
   </раздатка>
   Перед вами диалог из фильма "Дом, который построил Свифт". Какой
глагол мы пропустили?

Ответ:
Опустился.

Источник:
Х/ф "Дом, который построил Свифт" (1982), реж. Марк Захаров.

Автор:
Данила Аладин

Вопрос 2:
В игре "Doom 2", вышедшей в 1994 году, на двадцать первой карте можно
найти дробовик. Каким семибуквенным словом называется эта карта?

Ответ:
Нирвана, Nirvana.

Комментарий:
В том же 1994 году лидер группы "Нирвана" Курт Кобейн самоубился при
помощи ружья.

Источник:
http://doom.wikia.com/wiki/MAP21:_Nirvana_(Doom_II)

Автор:
Данила Аладин

Вопрос 3:
Кто дублировал марсианку Ки в русской версии мультфильма "Тайна красной
планеты"?

Ответ:
[Жанна] Агузарова.

Источник:
http://ru.wikipedia.org/wiki/Тайна_красной_планеты

Автор:
Данила Аладин

Вопрос 4:
Согласно недавней шутке, ни Аристотель, ни Фома Аквинский, ни Карл
Поппер не выступали против Платона так последовательно, как это делают
ОНИ. Последняя, третья часть "ИХ" вышла на персональных компьютерах в
ноябре 2009 года. Назовите ИХ.

Ответ:
Дальнобойщики.

Комментарий:
"Платон" - российская система взимания платы с грузовиков, имеющих
разрешенную максимальную массу свыше 12 тонн. "Платон" вызвал известные
нарекания со стороны дальнобойщиков.

Источник:
   1. http://www.anekdot.ru/id/795227/
   2. http://ru.wikipedia.org/wiki/Дальнобойщики_3:_Покорение_Америки

Автор:
Данила Аладин

Вопрос 5:
Один из персонажей телесериала "Сайнфилд" говорит: "Мой отец водил меня
в цирк. Когда появлялись НИКСОНЫ, он кричал им проклятия, возлагая на
них вину за все болезни общества". Какое слово мы заменили на "НИКСОНЫ"?

Ответ:
Слоны.

Комментарий:
Как известно, слон - символ республиканской партии США, а Никсон -
известный республиканец.

Источник:
Сериал "Сайнфилд", s06e06, 17-я минута.

Автор:
Данила Аладин

Вопрос 6:
В сериале "Community" старик, рассуждая о небольшом размере чека,
говорит: "Я над Пёрл-Харбором больше ИХ видел". Назовите ИХ.

Ответ:
Ноли.

Комментарий:
Имеется в виду японский истребитель Mitsubishi A6M Zero.

Источник:
Сериал "Community", s06e13, 10-я минута.

Автор:
Данила Аладин

Вопрос 7:
Под новостью о том, что четверо диггеров и их бомж-проводница задержаны
при попытке пройти в Кремль по канализации один из комментаторов
поместил картинку. Назовите имя любого из пяти персонажей, изображенных
на ней.

Ответ:
Леонардо.

Зачет:
Донателло; Микеланджело; Рафаэль; Эйприл.

Комментарий:
(pic: 20160342.jpg)
   Черепашки-ниндзя - известные жители канализации. :-)

Источник:
https://vk.com/wall-52557389_24938

Автор:
Данила Аладин

Вопрос 8:
Всем известно стихотворение:
   Спорит с викингом раввин.
   Спор заведомо бесплоден:
   - Бог один. И он - не Один!
   - Один - бог! И не один.
   Для этого стихотворения было предложено двустишие-продолжение.
Закончите его тремя словами:
   Раввин и викинг в жарком споре
   Сошлись на том, что...

Ответ:
Сила в Торе.

Зачет:
По слову "Торе".

Источник:
http://mfrid.livejournal.com/372076.html

Автор:
Данила Аладин

Вопрос 9:
Автор вопроса восьмого февраля побывал в клубе "105". В каком
подмосковном городе он расположен?

Ответ:
Дубна.

Комментарий:
8 февраля - День Российской науки. 105-й элемент таблицы Менделеева -
дубний.

Источник:
ЛОАВ.

Автор:
Данила Аладин

Вопрос 10:
Пытаясь использовать популярный мем для рекламы выставки "Ожившие
полотна", ее организаторы столкнулись с официальной претензией
Кристиана, не дававшего разрешения на использование своей фамилии.
Напишите фамилию Кристиана.

Ответ:
Лубутен.

Зачет:
Лабутен.

Комментарий:
Речь идет о клипе группы "Ленинград" на песню "Экспонат", где героиня
собирается на выставку Ван Гога именно на "лабутенах". Организаторы
выставки воспользовались популярностью этого клипа для рекламных акций.

Источник:
http://lenta.ru/news/2016/01/27/louboutin/

Автор:
Евгений Кононенко

Вопрос 11:
Согласно шутке, во время одного из интервью ЕГО спросили, каково это -
родиться бедным, да еще и слепым. ОН ответил с юмором: "Всё могло быть
намного хуже, я мог бы родиться черным". Назовите ЕГО.

Ответ:
Рэй Чарльз.

Комментарий:
Как известно, Рэй Чарльз не был слепым с рождения, а ослеп в детстве,
так что прекрасно знал, что он негр.

Источник:
http://www.musicforums.ru/jazz_funk_arc/1115924133.html

Автор:
Глеб Крутинин

Вопрос 12:
Согласно шутке с сайта bash.im, сосед в два часа ночи пел в караоке хит
2000 года. Назовите эту песню одним словом.

Ответ:
"Хочешь".

Комментарий:
"Хочешь, я убью соседей, что мешают спать?". :-)

Источник:
http://bash.im/abyssbest/20150916

Автор:
Данила Аладин

Вопрос 13:
(pic: 20160343.jpg)
   От вас скрыли название произведения 1985 года. Назовите его автора.

Ответ:
[Патрик] Зюскинд.

Комментарий:
(pic: 20160344.jpg)

Источник:
   1. http://pikabu.ru/story/russkaya_versiya_quotparfyumeraquot_2229845
   2. http://ru.wikipedia.org/wiki/Парфюмер._История_одного_убийцы_(роман)

Автор:
Алексей Дубровский

Вопрос 14:
Так как ЭТО ИМЯ более тридцати лет назад было достаточно малоизвестно в
Европе, то во французском дубляже фильма главного героя называют Пьером
Карденом, а в итальянском - Леви Страуссом. Напишите ЭТО ИМЯ.

Ответ:
Кельвин Кляйн.

Зачет:
Calvin Klein.

Комментарий:
В фильме "Назад в будущее" Марти МакФлая называют Кельвином Кляйном,
увидев надпись на его трусах. В 1985 году Кельвин Кляйн был малоизвестен
в Европе.

Источник:
http://www.adme.ru/vdohnovenie/30-prichin-otpravitsya-nazad-v-buduschee-714660/

Автор:
Данила Аладин

Вопрос 15:
В компьютерной игре "Fallout 4" можно, разобрав завалы в замке, найти
труп генерала Макганна. Назовите несклоняемым словом, что можно найти
рядом с ним.

Ответ:
Амонтильядо.

Источник:
Компьютерная игра "Fallout 4".

Автор:
Данила Аладин

Вопрос 16:
Согласно шутке с сайта bash.im, "Тесла" вывела на тесты новую, МОЩНУЮ,
версию автомобиля". Какое слово мы заменили на "МОЩНУЮ"?

Ответ:
Заряженную.

Комментарий:
"Tesla Motors" известны своими электромобилями. Часто более спортивные,
мощные версии автомобилей называют "заряженными".

Источник:
http://bash.im/quote/434754/

Автор:
Данила Аладин

Вопрос 17:
Дуплет.
   1. Тюрьма, в которой сидел будущий сенатор Джон Маккейн, получила
среди бывших заключенных прозвище "Хан Хилтон". Что мы пропустили в
предыдущем предложении?
   2. Продолжите тремя буквами анекдот: "В Одессе умер Изя. Родственники
решают, как бы подешевле сообщить об этом печальном событии родным в
Израиль. Придумали и послали телеграмму "Изя - ..."".

Ответ:
   1. ой.
   2. "... всё".

Источник:
   1. http://ru.wikipedia.org/wiki/Ханой_Хилтон
   2. http://lurkmore.to/Ой-вей

Автор:
Михаил Шуваев

Вопрос 18:
Главный герой фильма 1947 года указал в анкете, что его зовут Крис
Крингл, дату рождения он назвать не может, возраст его - "чуть старше,
чем его зубы", а в списке ближайших родственников указал восемь кличек.
Какие два слова герой вписал в графу "Место рождения"?

Ответ:
Северный полюс.

Комментарий:
Фильм - "Чудо на 34-й улице". Главный герой - настоящий Санта-Клаус. В
списке ближайших родственников указаны восемь оленей.

Источник:
Х/ф "Чудо на 34-й улице" (1947), реж. Джордж Ситон.

Автор:
Данила Аладин

Вопрос 19:
Репортаж о победе на выборах ультраправого Национального фронта во главе
с МарИн ЛепЕн был озаглавлен "Франция [ПРОПУСК]". Мы не просим заполнить
пропуск двумя словами. Кто был [ПРОПУСК] в песне "Машины времени"?

Ответ:
Птица.

Комментарий:
В репортаже страна была определена как "цвета ультра-Марин". "Машина
времени" использовала образ "птицы цвета ультрамарин".

Источник:
   1. http://www.youtube.com/watch?v=F1MWXJhH9vQ
   2. http://megalyrics.ru/lyric/mashina-vriemieni/ptitsa-tsvieta-ultramarin.htm

Автор:
Евгений Кононенко

Вопрос 20:
По мнению Михаила Гурьева, руководителя специализированной
реставрационной лаборатории Эрмитажа, мировую моду на шедевры
микромеханики, начавшуюся с рококо и сохранявшуюся более столетия, убил
ширпотреб, когда в 1887 году появился ОН. Назовите ЕГО словом с
греческими корнями.

Ответ:
Граммофон. Незачет: Фонограф.

Комментарий:
Шедевры микромеханики - это, прежде всего, музыкальные шкатулки. В
Эрмитаже существует лаборатория реставрации часов и музыкальных
механизмов. Граммофон (от греч. &gamma;&rho;&#940;&mu;&mu;&alpha; -
запись, &phi;&omega;&nu;&#942; - звук) запатентован Эмилем Берлинером в
1887 году. Фонограф появился на десять лет раньше.

Источник:
   1. https://www.1tv.ru/news/2015/12/11/5604-v_ermitazhe_otkrylas_vystavka_gde_predstavleny_proizvedeniya_mehanicheskogo_iskusstva
   2. http://ru.wikipedia.org/wiki/Граммофон

Автор:
Евгений Кононенко

Вопрос 21:
В рекламном ролике американской косметической компании говорится, что
именно "Макс Фактор" превратил Норму в... Закончите слоган одним словом.

Ответ:
Мэрилин.

Комментарий:
Настоящее имя Мэрилин Монро - Норма Джин Бейкер. Слоган компании: "Мы
превратили Норму Джин в Мэрилин".

Источник:
http://www.youtube.com/watch?v=ntBAt6Ekpuo

Автор:
Евгений Кононенко

Вопрос 22:
Обнаружив "ИКС" на задней панели швейной машинки, Маргарет Янг
поделилась находкой с братьями. Через 31 год, в 2004-м, улица
Корпорейшн-Лейн в Мельбурне переименована в ИКС-Лейн, причем в
соответствии с местными правилами ИКС стал на один знак короче.
Воспроизведите ИКС в любом написании.

Ответ:
AC/DC.

Зачет:
ACDC.

Комментарий:
Много ли известных Янгов из Австралии? Маргарет подарила братьям
Малколму и Агнусу обозначение "переменный/постоянный ток" в качестве
названия для организованной ими рок-группы. Улица в Мельбурне была
переименована в честь знаменитого австралийского коллектива, но в
названиях улиц в Австралии не используется знак "/" [слэш].

Источник:
http://ru.wikipedia.org/wiki/AC/DC

Автор:
Евгений Кононенко

Вопрос 23:
Названия некоторых серий: "Холодные закуски", "Основное блюдо",
"Жаркое", "Комплимент от шеф-повара", "Десерт". Назовите заглавного
героя сериала.

Ответ:
[Доктор] [Ганнибал] Лектер.

Комментарий:
Все эпизоды сериала "Ганнибал" (2013) о докторе-людоеде имеют кулинарные
названия.

Источник:
http://ru.wikipedia.org/wiki/Ганнибал_(телесериал)

Автор:
Евгений Кононенко

Вопрос 24:
В "Правде короля Альфреда" есть закон, по которому ЭТО переходило в
собственность семьи погибшего из-за НЕГО. Лесники говорят, что ЭТО -
одна из главных причин гибели бобров. Назовите ЭТО двумя словами.

Ответ:
Упавшее дерево.

Источник:
   1. http://www.rummuseum.ru/lib_t/angl06.php
   2. http://www.zoofacts.ru/17-faktov-o-bobrax/

Автор:
Татьяна Верясова

Тур:
22 тур. "Деннис Бархоппер" (Воронеж - Москва)

Инфо:
Команда благодарит Елену Анциферову (Томск), Андрея Кокуленко (Омск),
Максима Мерзлякова (Воронеж), Константина Науменко (Киев), Алексея и
Марию Трефиловых (Калуга), Евгения Назаренко и Ольгу Поволоцкую (оба -
Воронеж) за тестирование и ценные замечания.

Вопрос 1:
Эти слова произносит герой одного фильма, характеризуя секс-скандал с
участием священников. Эти же слова историк Сесила Клайн использует при
описании обряда, посвященного ацтекской богине Тласолтеотль, которая
олицетворяла очищение от грехов. Напишите эти два английских слова.

Ответ:
Holy Shit.

Комментарий:
Узнав о количестве вовлеченных в скандал священников, герой фильма "В
центре внимания" произносит подходящее к случаю довольно
распространенное ругательство. Богиня Тласолтеотль - покровительница
грешников, также известна как пожирательница грязи или дерьма. В
посвященных ей обрядах использовалось дерьмо. Историк называет его Holy
Shit.

Источник:
   1. Х/ф "В центре внимания" (2015), реж. Том Маккарти, 1:04:41.
   2. http://en.wikipedia.org/wiki/Tlazolteotl

Автор:
Роман Цуркан (Воронеж)

Вопрос 2:
(pic: 20160345.jpg)
   Перед вами коллаж с англоязычного интернет-ресурса. Восстановите
надпись над правой частью коллажа.

Ответ:
Cartesian [bear].

Зачет:
Декартов [медведь].

Комментарий:
Надпись над левой частью - "Polar bear". Обыгрываются названия полярной
и декартовой систем координат. В английском языке белого медведя
называют полярным.

Источник:
http://9gag.com/gag/aVPMGVy

Автор:
Андрей Кудрявцев (Москва)

Вопрос 3:
Внимание, в вопросе есть замена.
   В сериале "Однажды в сказке" Безумного Шляпника зовут Монро, что
является аллюзией на произведение 1967 года. Какое имя собственное мы
заменили словом "Монро"?

Ответ:
Джефферсон.

Комментарий:
Известная в 1960-е группа "Jefferson Airplane" записала в 1967 году одну
из своих самых известных песен - "White Rabbit". Джефферсон и Монро -
президенты США.

Источник:
   1. http://en.wikipedia.org/wiki/List_of_Once_Upon_a_Time_characters
   2. http://www.imdb.com/character/ch0012251/
   3. http://en.wikipedia.org/wiki/White_Rabbit_(Jefferson_Airplane_song)
   4. http://en.wikipedia.org/wiki/List_of_Presidents_of_the_United_States

Автор:
Роман Цуркан (Воронеж)

Вопрос 4:
Согласно шутке, в 2016 году может разыграться семейная драма, ведь после
долгого путешествия ОНА, наконец, доберется до мужа и его возлюбленных.
Другая ОНА известна из истории более чем двухсотлетней давности, где до
создания семьи дело так и не дошло. Назовите ЕЕ.

Ответ:
Юнона.

Комментарий:
В 2016 году к Юпитеру и его спутникам прибудет космический аппарат
"Юнона", запущенный NASA в 2011 году. Многие из спутников планеты Юпитер
(например, Ио, Каллисто, Европа и Ганимед) названы в честь
мифологических персонажей, которые являлись возлюбленными Зевса. "Юнона"
- имя корабля Николая Резанова, чья свадьба с Кончитой так и не
состоялась.

Источник:
   1. http://ru.wikipedia.org/wiki/Юнона_(космический_аппарат)
   2. http://ru.wikipedia.org/wiki/Резанов,_Николай_Петрович

Автор:
Дмитрий Тарарыков (Дубровский)

Вопрос 5:
(pic: 20160346.jpg)
   Мы изменили часть слова, которым в Монреале называют подобные
сооружения. Напишите слово в исходном виде.

Ответ:
Vespasienne.

Комментарий:
Это общественный туалет, который в память о знаменитой фразе "Деньги не
пахнут" называют в честь ее автора - "Веспасьян". Замена: Vespa (осы) на
Apis (пчелы).

Источник:
http://en.wikipedia.org/wiki/Pecunia_non_olet

Автор:
Роман Цуркан (Воронеж)

Вопрос 6:
Иногда властям проще не обращать внимания на проблему. Несмотря на
происходившие в течение последних лет дерзкие налеты, только в 2015 году
в Венеции начали борьбу с НИМИ. В произведении 1970 года заглавный герой
противостоит ИХ обществу с засильем приземленных корыстных интересов.
Назовите ИХ.

Ответ:
Чайки.

Комментарий:
Венеция - морской город. Чайки перекочевали в город с побережья,
поскольку в море уже недостаточно рыбы, а город предоставляет широкие
возможности с точки зрения поиска пищи. В последнее время они совершенно
обнаглели и стали представлять серьезную опасность. Власти города до
определенного момента проблему не замечали и только в 2015 году
принялись за активные действия. Произведение 1970 года - повесть Ричарда
Баха "Чайка по имени Джонатан Ливингстон".

Источник:
   1. http://www.kp.ru/daily/26368.4/3249031/
   2. http://ru.wikipedia.org/wiki/Чайка_по_имени_Джонатан_Ливингстон

Автор:
Дмитрий Тарарыков (Дубровский)

Вопрос 7:
   <раздатка>
   При просмотре отечественной киноленты 1992 года "Гонгофер", средства
на которую выделила известная компания, в кадре неоднократно можно
увидеть ИХ. В тексте этого вопроса также есть ОНИ. Назовите ИХ точно.
   </раздатка>
   При просмотре отечественной киноленты 1992 года "Гонгофер", средства
на которую выделила известная компания, в кадре неоднократно можно
увидеть ИХ. В тексте этого вопроса также есть ОНИ. Назовите ИХ точно.

Ответ:
Три буквы "М".

Зачет:
МММ.

Комментарий:
Деньги на съемку фильма выделил Сергей Мавроди. В кадре время от времени
мелькают "МММ". В тексте вопроса также три раза встречается буква "М".

Источник:
http://ru.wikipedia.org/wiki/Гонгофер

Автор:
Роман Цуркан (Воронеж)

Вопрос 8:
Внимание, в вопросе слова "ИКСовые" и "ИКС" заменяют другие слова.
   Герой Ильдуса Сагитова отправил немного зелени в ИКСовые тени.
Внутренним ИКСом для России Википедия называет Калмыкию и Республику
Алтай. Назовите ИКСы словом английского происхождения.

Ответ:
Офшоры.

Зачет:
Оффшоры; offshore.

Комментарий:
Герой стихотворения отправил немного зелени в офшорные тени. Калмыкию и
Республику Алтай Википедия называет внутренними офшорами.

Источник:
   1. http://www.stihi.ru/2015/03/31/3865/
   2. http://ru.wikipedia.org/wiki/Офшор

Автор:
Роман Цуркан (Воронеж)

Вопрос 9:
В книге Егора Трубникова среди прочих упоминается ютараптор. Назовите
человека, который в аналогичном произведении откровенно признался, что
фантазировал.

Ответ:
[Борис Владимирович] Заходер.

Комментарий:
Егор Трубников написал "Веселую азбуку для маленьких вивисекторов".
Ютараптор - животное на букву "ю". Заходер в своей "Мохнатой азбуке"
писал:
   Откровенно признаю:
   Зверя нет На букву "Ю".
   Это - ЮЖНЫЙ КТОТОТАМ.
   Я его придумал сам!

Источник:
   1. http://mcdowns.livejournal.com/253921.html
   2. http://www.e-reading.club/bookreader.php/96888/Zahoder_-_Mohnataya_azbuka.html

Автор:
Аркадий Илларионов (Воронеж)

Вопрос 10:
Купец Павел Михайлович Третьяков любил повторять поговорку: "Спать долго
- жить долго". Какие две буквы мы пропустили в предыдущем предложении?

Ответ:
с, м.

Комментарий:
"Спать долго - жить с долгом". Третьяков не одобрял долгий сон.

Источник:
http://www.dengy-vsem.ru/prokrizis_vid.php?id=5

Автор:
Роман Цуркан (Воронеж)

Вопрос 11:
DARPA планирует создать дронов под названием "ИКСЫ", которые создавали
бы помехи в функционировании самолета. "ИКСЫ" выиграли пять премий
"Сатурн" в 1985 году. Назовите ИКСОВ.

Ответ:
Гремлины.

Комментарий:
Дроны способны заглушать связь самолета противника. Гремлины известны
как ненавистники техники. "Гремлины" - комедийный фильм ужасов 1984
года, "Сатурн" - американская премия, вручаемая Академией научной
фантастики, фэнтези и фильмов ужасов.

Источник:
   1. http://geektimes.ru/company/robohunter/blog/263224/
   2. http://ru.wikipedia.org/wiki/Эффект_гремлина
   3. http://ru.wikipedia.org/wiki/Гремлины

Автор:
Аркадий Илларионов (Воронеж)

Вопрос 12:
На плакате социальной рекламы ОНИ образуют сочетание три точки - три
тире - три точки. Назовите ИХ двумя словами, начинающимися на одну и ту
же букву.

Ответ:
Капли крови.

Комментарий:
Реклама призывает стать донором, помогая попавшим в беду.

Источник:
http://adsoftheworld.com/media/print/kiev_municipal_blood_donor_centre_sos

Автор:
Аркадий Илларионов (Воронеж)

Вопрос 13:
Дуплет.
   1. В первой половине 2014 года в СМИ появилось ложное сообщение об
отмене десятков ЕГО концертов. Назовите ЕГО фамилию.
   2. В ноябре 2015 года ОН сказал, что хочет переименовать свой
творческий коллектив и взять фамилию жены. Назовите ЕГО фамилию.

Ответ:
   1. Майданов.
   2. Турецкий.

Комментарий:
   1. После Евромайдана появились шутки о том, что у Дениса Майданова
возникнут проблемы из-за фамилии.
   2. Михаил Турецкий разлюбил свою фамилию после резкого ухудшения
отношений России и Турции.

Источник:
   1. http://www.rusradio.ru/news/showbiznes/7669253/
   2. http://lenta.ru/news/2015/11/28/tureckiprotivturcii/

Автор:
Павел Казначеев (Москва)

Вопрос 14:
По замечанию журнала "Русский репортер", после присоединения к России
многие крымчане посетили ИКС. Одним из героев рекламы ИКСА стал Жерар
Депардье. Назовите ИКС.

Ответ:
Фотосалон.

Зачет:
Фотоателье; фотомастерская; фотостудия.

Комментарий:
После присоединения к России многие крымчане стали фотографироваться на
российские документы. В рекламе изображения звезд использовались для
иллюстрации примеров фотографий разных типов, Депардье обозначал паспорт
РФ.

Источник:
   1. "Русский репортер", 2014, N 18.
   2. http://pikabu.ru/story/lyuboe_foto_za_20_minut_2680115

Автор:
Александр Нечаев (Воронеж)

Вопрос 15:
На рекламном плакате портативной игровой консоли PlayStation Vita,
позволяющей использовать обе стороны устройства, изображена ОНА. Одна
китаянка после неудачной пластической операции обнаружила ЕЕ. Назовите
ЕЕ тремя словами.

Ответ:
Грудь на спине.

Комментарий:
Слоган рекламы - "Трогай обе стороны для дополнительного удовольствия".
Китаянка Тянь Хуэ увеличила грудь, но та с каждым днем "уходила" всё
сильнее. Однажды утром Хуэ проснулась и обнаружила одну грудь у себя на
животе, а другую - на спине.

Источник:
   1. http://adsoftheworld.com/media/print/playstation_touch_both_sides
   2. http://www.utro.ru/articles/2014/04/14/1189106.shtml

Автор:
Аркадий Илларионов (Воронеж)

Вопрос 16:
Персонаж Айзека Азимова, описывая пристальный взгляд профессора,
говорит, что почувствовал себя находящимся по ту сторону ЕГО. А каким
словом называют ЕГО в произведении 1881 года?

Ответ:
Мелкоскоп.

Комментарий:
ОН - микроскоп. В "Левше" Лескова микроскоп называют мелкоскопом.

Источник:
   1. А. Азимов. Что в имени? http://www.flibusta.is/b/163026/read
   2. Н.С. Лесков. Левша. http://az.lib.ru/l/leskow_n_s/text_0246.shtml
   3. http://ru.wikipedia.org/wiki/Левша_(сказ)

Автор:
Андрей Кудрявцев (Москва)

Вопрос 17:
Считается, что ставшие названием брэнда ОНИ вдохновили Петра Ершова на
один из эпизодов сказки "Конек-горбунок". Литературный фестиваль других
ИХ проходит ежегодно на севере США. Назовите ИХ двумя словами,
начинающимися на соседние буквы алфавита.

Ответ:
Пять озер.

Комментарий:
Считается, что купаться в пяти озерах, расположенных на границе
Новосибирской и Омской областей, нужно в правильном порядке. Вокруг
Великих озер сформировалась своя небольшая литературная школа, тоже
озерная. "Пять озер" - название известной водки.

Источник:
   1. http://www.5ozer.com/#!/legend/
   2. http://greatlakeswritersfestival.org/

Автор:
Павел Казначеев (Москва)

Вопрос 18:
Несмотря на имя, у АЛЬФЫ, конечно, была не только мать, но и минимум две
дочери, ведь если бы дочь была одна, то АЛЬФОЙ была бы именно она.
Назовите АЛЬФУ двумя словами.

Ответ:
Митохондриальная Ева.

Комментарий:
Митохондриальная Ева - имя, данное в популярной культуре женщине,
которая была последним общим предком всех ныне живущих людей по
материнской линии. Несмотря на аналогию с библейской, митохондриальная
Ева, вероятно, ничем не выделялась на фоне соплеменников.

Источник:
http://ru.wikipedia.org/wiki/Митохондриальная_Ева

Автор:
Дмитрий Тарарыков (Дубровский)

Вопрос 19:
(pic: 20160347.jpg)
   Перед вами фрагмент шуточного рисунка. Назовите фамилию человека, к
которому обращается недовольный рабочий.

Ответ:
Эшер.

Комментарий:
(pic: 20160348.jpg)

Источник:
http://www.newyorker.com/cartoons/a17033

Автор:
Андрей Кудрявцев (Москва)

Вопрос 20:
В одном из рассказов братьев Стругацких ученые испытывают метод
воздействия на сознание человека, названный "акупунктурой". Идея
рассказа пришла Борису Стругацкому, когда тот читал про ИКС. Назовите
ИКС словом неевропейского происхождения.

Ответ:
Пейот.

Зачет:
Пейотль; мескалин.

Комментарий:
Борис Стругацкий был впечатлен образом кактуса, изменяющего сознание, и
выбрал "колючий" термин для своего метода воздействия на человеческий
разум.

Источник:
Б.Н. Стругацкий. Комментарии к пройденному.
http://www.flibusta.is/b/135631/read

Автор:
Павел Казначеев (Москва)

Вопрос 21:
Внимание, в вопросе слово "ТАК" заменяет другое слово.
   В русском переводе одного из рассказов Акутагавы Рюноскэ при описании
банкета используется наречие "ТАК". Скорее всего, перевод не является
буквальным. В названии какого произведения 1894 года содержится
существительное "ТАК"?

Ответ:
"Над вечным покоем".

Комментарий:
В первом случае слово "покоем" - это наречие, во втором -
существительное. "Над вечным покоем" - известная картина Левитана. На
крупных застольях столы часто ставят покоем, т.е. буквой "П". Автор
вопроса не знает японского, но предполагает, что в оригинале
использовалось другое слово, так как в японском языке используются
иероглифы, а не кириллица. Слово "буквальный" - тоже подсказка.

Источник:
   1. Акутагава Рюноскэ. Зубчатые колеса.
http://www.flibusta.is/b/121827/read
   2. http://ru.wikipedia.org/wiki/Над_вечным_покоем

Автор:
Андрей Кудрявцев (Москва)

Вопрос 22:
Можно считать, что в этом вопросе мы пропустили два слова, а можно - что
сделали равноценную замену.
   Статья в журнале "Наука и жизнь", посвященная новым корпусам
Дальневосточного Федерального университета, называется "Университет".
Какие два слова мы пропустили в названии статьи?

Ответ:
"... на Русском".

Комментарий:
Имеется в виду остров Русский, известный в последнее время не только
Дальневосточным университетом, но и, например, уникальным мостом на него
с материка. Можно сказать, что мы сделали равноценную замену, заменив
"университет на русском" на "университет", так как слово "университет"
на русском звучит как "университет".

Источник:
   1. "Наука и жизнь", 2013, N 12. - С. 35.
   2. http://ru.wikipedia.org/wiki/Русский_(остров,_Приморский_край)

Автор:
Андрей Кудрявцев (Москва)

Вопрос 23:
[Ведущему: перед словом "точно" сделать паузу, про себя произнеся
"абсолютно".]
   Юрий Башмет признавался, что при попытках играть на старинных
инструментах так, как предполагалось при их создании, ему ужасно мешал
ИКС. Назовите ИКС точно.

Ответ:
Абсолютный слух.

Комментарий:
Старинные струнные инструменты известных мастеров (Амати, Страдивари,
Гварнери и т.д.) делались для игры в натуральном строе и лучше всего
звучат, конечно, в нем. Юрий Башмет обладает абсолютным слухом,
адаптированным под ныне повсеместно распространенный равномерно
темперированный строй. Для него инструмент, настроенный в другом
(естественном для инструмента) строе, звучит фальшиво.

Источник:
Алексей Насретдинов "Анатомия музыкального слуха".
http://www.youtube.com/watch?v=hsCjtXs7UOU

Автор:
Дмитрий Тарарыков (Дубровский)

Вопрос 24:
Осознавший свое несовершенство герой Евгения Клюева встретил врага. К
счастью, он заметил у себя ЭТО и использовал в качестве оружия. Назовите
ЭТО тремя словами, начинающимися на следующие подряд буквы алфавита.

Ответ:
Бревно в глазу.

Комментарий:
Осознавший свое несовершенство герой видит бревно в своем глазу. Бревно
вполне может стать оружием.

Источник:
Е.В. Клюев. Между двух стульев. http://www.flibusta.is/b/67087/read

Автор:
Павел Казначеев (Москва)

Тур:
23 тур. "Ударная группа" (Зеленоград)

Редактор:
Олег Холодов (Серпухов)

Вопрос 1:
В книге "Английский и американский сленг" выражение "all-and-all",
обозначающее сленговое название ЕЕ, среди русских вариантов перевода
имеет слова "кобра" и "анаконда". А какое название ЕЕ употребила героиня
произведения 1984 года?

Ответ:
Горгона.

Комментарий:
Приведенное выражение - сленговое название жен. Известен диалог из
фильма "Любовь и голуби", в котором героиня Гурченко спрашивает: "Ты к
этой Горгоне?", на что герой Михайлова отвечает: "Нет, я к жене". В
голове Медузы Горгоны, как известно, были змеи.

Источник:
   1. Т.Е. Захарченко. Английский и американский сленг.
   2. Х/ф "Любовь и голуби" (1984), реж. Владимир Меньшов.

Автор:
Константин Крюков

Вопрос 2:
Вернувшись в город после длительного отсутствия, герой Вальтера Мёрса
видит следующее объявление: "Если ты у меня под юбкой, ты, должно быть,
ИКС". ИКС из комикса "Зеленый фонарь" создал гипно-луч. Догадавшись, что
мы заменили на ИКС, назовите двумя словами увлечение, захватившее город
за время отсутствия героя.

Ответ:
Кукольный театр.

Комментарий:
ИКС - кукловод. Гипно-луч использовался для контроля сознания.

Источник:
   1. http://en.wikipedia.org/wiki/List_of_minor_DC_Comics_characters#Puppeteer
   2. Walter Moers, "Das Labyrinth der Tr&auml;umenden B&uuml;cher".

Автор:
Екатерина Данина

Вопрос 3:
Кто и когда впервые создал такой рисунок, неизвестно. Основными
техниками его выполнения являются этчинг и питчинг, а основными фигурами
- "яблоко", "сердечко" и "листочек". Высшим пилотажем считается
"Тадж-Махал", придуманный австралийцем Джоржем Саббадсом. Мировой рекорд
на сегодняшний день - семь роз, "распустившиеся"... Где?

Ответ:
В чашке кофе.

Зачет:
В чашке капучино.

Источник:
   1. http://photoshtab.ru/2012/02/capuchino/
   2. http://coffee-klatsch.ru/latte-art1.html
   3. http://www.youtube.com/watch?v=DTwx4i8j5V0

Автор:
Екатерина Данина

Вопрос 4:
ТАКОЙ ОН похож на обман, ЭТАКИЙ ОН проплывал над героями, унося их в
ЭТАКУЮ даль. А вот героиня повести 1970 года могла вызывать СЯКОГО ЕГО,
Назовите эту героиню.

Ответ:
Арахна.

Комментарий:
Речь шла о песнях "Синий туман" и "Сиреневый туман", а также о сказке
А.М. Волкова "Желтый туман".

Источник:
   1. http://megalyrics.ru/lyric/vdobrynin/sinii-tuman.htm
   2. http://a-pesni.org/dvor/sirentuman.php
   3. http://ru.wikipedia.org/wiki/Жёлтый_Туман

Автор:
Валерий Клементьев

Вопрос 5:
Есть на Волге между Нижним Новгородом и Чебоксарами небольшой город
Козьмодемьянск, который в 2014 году принимал участие в Межрегиональном
фестивале сатиры и юмора. Приглашенный в город большой мастер Анатолий
Евгеньевич в течение пяти часов обучал 18 своих "подмастерьев" азам
своего мастерства. Назовите это мастерство.

Ответ:
Шахматы.

Комментарий:
Козьмодемьянск стал прототипом Васюков в знаменитом романе И. Ильфа и Е.
Петрова "Двенадцать стульев". "Гроссмейстер" в переводе с немецкого -
"большой мастер", а им на турнире с участием шахматистов из Ижевска,
Казани, Йошкар-Олы, Чебоксар, Козьмодемьянска и других городов России
стал чемпион мира Анатолий Карпов (кстати, две партии закончились
вничью).

Источник:
http://www.kmkmuzey.ru/index.php/arh1/694-shahimat

Автор:
Валерий Клементьев

Вопрос 6:
На балу у Сатаны Воланд в разговоре с головой Берлиоза произносит мысль
о том, что ОНИ - "самая упрямая в мире вещь". С 1978 года ОНИ вместе со
своими напарниками по средам радуют своих почитателей и при этом даже
ставят рекорды. Назовите напарников.

Ответ:
Аргументы.

Комментарий:
Речь шла о фактах (факт - самая упрямая вещь) и их напарниках -
аргументах. А следовательно, и о еженедельнике "Аргументы и факты",
который попадал даже в Книгу рекордов Гиннесса.

Источник:
   1. М.А. Булгаков. Мастер и Маргарита.
http://books.rusf.ru/unzip/xussr_av/bulgam01.htm?34/50
   2. http://ru.wikipedia.org/wiki/Аргументы_и_факты

Автор:
Валерий Клементьев

Вопрос 7:
Некоторые считают, что китайский язык очень простой. Необязательно учить
все иероглифы, ведь зачастую можно догадаться о значении слова,
посмотрев на составляющие его части. Например, дословный перевод
известной пары антонимов выглядит следующим образом: "ИКС хозяина" и
"ИКС гостя". Назовите пару антонимов, если известно, что они являются
прилагательными.

Ответ:
Субъективный и объективный.

Комментарий:
"Взгляд хозяина" и "взгляд гостя". Подсказки: "посмотреть на..." и
вступление "некоторые считают...".

Источник:
http://www.forum-tvs.ru/lofiversion/index.php/t121882-50.html

Автор:
Ирина Табункова

Вопрос 8:
Финляндия - страна победившего гендерного равенства. Финские женщины
первыми в Европе получили право голоса, активно работают во всех сферах,
включая полицию и армию, имеют большую долю в руководстве страны и даже
допущены до получения духовного сана, вплоть до епископа. Считается, что
причина такого положения кроется в том числе и в самом финском языке,
так как в нем отсутствует ЭТО. Назовите ЭТО.

Ответ:
Род.

Комментарий:
Местоимения "он" и "она" в финском языке обозначаются одним словом.

Источник:
http://pinkboy.ru/osobennosti-finskogo-ravnopravija-muzhchina-i-zhenschina-raznye-i-ravnye

Автор:
Екатерина Батракова

Вопрос 9:
До начала XX века, чтобы СДЕЛАТЬ ЭТО, двум людям требовалось почти
семьдесят лет. Сейчас же на это уходит всего два года. Что мы заменили
на "СДЕЛАТЬ ЭТО"?

Ответ:
Съесть пуд соли.

Источник:
https://health.mail.ru/news/kak_est_menshe_soli/

Автор:
Екатерина Данина

Вопрос 10:
Джеймс Рольф охарактеризовал игру "Desert Bus", в которой игроку
необходимо доехать от Лос-Анджелеса до Лас-Вегаса, как самую худшую игру
всех времен. Создатели игры в свое оправдание заявили, что хотели
создать игру, похожую на реальную жизнь. В этой игре нет соперников,
испытаний, временнЫх рамок и много чего еще. Также в игре нет ЕЕ. ОНА,
как известно, обычно является крайней. Назовите ЕЕ двумя словами.

Ответ:
Кнопка "пауза".

Комментарий:
На стандартной клавиатуре она правая в верхнем ряду.

Источник:
   1. "Angry Video Game Nerd", серия 119.
http://www.youtube.com/watch?v=RFi2vcseEz8
   2. http://en.wikipedia.org/wiki/Penn_%26_Teller%27s_Smoke_and_Mirrors#Desert_Bus

Автор:
Екатерина Данина

Вопрос 11:
Статья в журнале "Cosmopolitan", отвечающая на вопрос читательницы
"Почему он не хочет жениться?", начинается словами "Люди вообще-то редко
хотят жениться, обычно они хотят [ПРОПУСК], но это уже совсем другие
люди". Заполните пропуск словом-исключением.

Ответ:
"... замуж...".

Источник:
"Cosmopolitan", 2010, N 3.

Автор:
Наталья Самсонова

Вопрос 12:
Гитарист группы "The Offspring" Кевин Вассерман в одном из интервью
заявил, что в какой-то мере даже любит некомпетентных журналистов,
потому что при встрече с ними предвкушает, как будет ДЕЛАТЬ ЭТО.
Догадавшись, что мы заменили на "ДЕЛАТЬ ЭТО", назовите прозвище Кевина
по-русски или по-английски.

Ответ:
Лапша.

Зачет:
Noodles.

Источник:
Эфир радио "Ультра" от 22.10.2015 г.

Автор:
Екатерина Данина

Вопрос 13:
При подготовке к погружению молодые подводники должны пройти огонь, воду
и медные трубы. Огонь означает, что они должны уметь справиться с
пожаром на борту, вода означает ликвидирование течи забортной воды. А
какую деталь подлодки подразумевают под медными трубами?

Ответ:
Торпедный аппарат.

Комментарий:
В экстремальной ситуации через него осуществляется эвакуация экипажа.

Источник:
Журнал "Discovery", декабрь 2015 - январь 2016 г.

Автор:
Константин Крюков

Вопрос 14:
Автор вопроса называет это изобретение "деньгами на ветер" из-за счетов
за электричество, приходящих после использования этого прибора. Мы не
просим назвать этот прибор. Назовите героя, который явно не платил по
счетам за использование этих самых "денег на ветер".

Ответ:
Карлсон.

Комментарий:
Вентилятор.

Источник:
ЛОАВ.

Автор:
Наталья Самсонова

Вопрос 15:
Прослушайте миниатюру Андрея Яхонтова "Современный Германн" из цикла
"Монологи": "Графиня! Откройте мне номер своей карты". В этой миниатюре
мы пропустили три буквы. Восстановите их.

Ответ:
сим.

Источник:
http://www.mk.ru/culture/2011/03/11/571873-monologi.html

Автор:
Валерий Клементьев

Вопрос 16:
Древние строители возводили Стоунхендж из каменных глыб, добытых в горах
Пресели за 260 километров от места строительства. Исследователи
Лондонского королевского колледжа сделали открытие, что эти камни при
ударе молотком издают звук, напоминающий удар колокола. Поэтому и
появилось предположение, что Стоунхендж - это гигантский он. Какие шесть
букв мы пропустили в последнем предложении?

Ответ:
к, с, и, л, о, ф.

Зачет:
к, с, и, л, ф, о.

Источник:
"Geo", 2014, N 12.

Автор:
Валерий Клементьев

Вопрос 17:
В ЖЖ-блоге oskanov в одной из записей утверждается, что это наверняка
знакомое вам с детства прозвище можно перевести как "Король Лев".
Догадавшись, какое это прозвище, назовите того, благодаря кому оно вам
известно.

Ответ:
Киплинг.

Комментарий:
"Шер" в переводе с индийского - это лев.

Источник:
http://oskanov.livejournal.com/420758.html

Автор:
Константин Крюков

Вопрос 18:
Согласно одной из версий, этот персонаж древнееврейского эпоса получил
то, что хотел, лишь от карийского фикуса, так как все остальные деревья
обвиняли его в грехе и ослушании. Я не спрашиваю, что он хотел получить
у деревьев. Ответьте, в качестве чего он хотел это использовать.

Ответ:
Одежда.

Зачет:
По смыслу.

Источник:
   1. http://ru.wikipedia.org/wiki/Инжир
   2. Библия.

Автор:
Наталья Самсонова

Вопрос 19:
Король Марокко Хасан II запретил ЭТО произведение 1970 года в своей
стране. По некоторым данным, он просто обиделся. Назовите ЭТО
произведение.

Ответ:
"Белое солнце пустыни".

Комментарий:
Обиделся, что гарем в фильме был больше, чем у него.

Источник:
https://www.film.ru/articles/bylo-povere-chto-cherez-podushku-pulya-ne-probivaet

Автор:
Константин Крюков

Вопрос 20:
(pic: 20160349.jpg)
   Дуэт американских музыкантов Пола Саймона и Артура Гарфанкеля
известен под названием "Simon & Garfunkel". А начинали они выступать для
других подростков в 1957 году, вдохновленные образом братьев Эверли, под
названием, получившим свою известность с 1940 года. Напишите это
название.

Ответ:
"Tom & Jerry".

Источник:
   1. http://en.wikipedia.org/wiki/Simon_%26_Garfunkel
   2. http://en.wikipedia.org/wiki/Tom_and_Jerry

Автор:
Константин Крюков

Вопрос 21:
Альпинист Анатолий Букреев не очень хорошо владел английским языком. Во
время восхождения на Эверест в 1996 году он получил прозвище, которое
долго не мог понять, считая, что его называют шоколадкой. А какую деталь
экипировки Анатолия высмеивало это прозвище?

Ответ:
Обувь.

Комментарий:
Прозвище было "Sneakers", которое Анатолий по созвучию принял за
"Snickers". Дело в том, что до решающих высот он носил легкие ботинки с
небольшими шипами, в отличие от многих других альпинистов. Слово
"sneakers" является обозначением легкой обуви - теннисных туфель,
шиповок и т.д.

Источник:
А.Н. Букреев, Г. Вестон де Уолт. Эверест: смертельное восхождение.
http://www.flibusta.is/b/422421/read

Автор:
Константин Крюков

Вопрос 22:
До начала музыкальной карьеры басист группы "Nickelback" Майк Крюгер
имел другую профессию. Какую?

Ответ:
Официант.

Комментарий:
Фраза "Here's your nickel back" ("Вот ваши пять центов сдачи") стала
своего рода визитной карточкой музыканта. Из нее и получилось название
группы.

Источник:
http://ru.wikipedia.org/wiki/Nickelback

Автор:
Екатерина Данина

Вопрос 23:
24 июля 1915 года в Чикаго затонул пассажирский пароход "Eastland". По
одной из версий, причиной катастрофы стало то обстоятельство, что на
борту было слишком много ИХ. Назовите ИХ.

Ответ:
Шлюпки.

Комментарий:
После катастрофы "Титаника", усугубленной недостаточным количеством
шлюпок на борту судна, в США был принят "Акт о моряках", обязывающий в
том числе разместить на суднах количество шлюпок, достаточное для
эвакуации 75% пассажиров. Суда на Великих озерах строились без расчета
на дополнительный вес, вследствие чего шлюпки, добавленные после
принятия данного Акта, добавили веса и так перегруженному судну, в
результате чего оно пошло ко дну прямо около причальной стенки.

Источник:
   1. http://en.wikipedia.org/wiki/SS_Eastland
   2. http://ru.wikipedia.org/wiki/Крушение_%C2%ABТитаника%C2%BB

Автор:
Константин Крюков

Вопрос 24:
На протяжении последних тринадцати лет Синтаро Хаяси делает ИКСЫ.
Получив один, два, три, а иногда и четыре ИКСА, ее клиенты не только
быстрее находят новую работу, но и могут идеально показать один из самых
известных фокусов. Назовите ИКС двумя словами, начинающимися на
одинаковую букву.

Ответ:
Протезы пальцев.

Источник:
http://mirfactov.com/zhenshhina-vyirastila-dlya-yakudza-300-paltsev/

Автор:
Ирина Табункова

Тур:
24 тур. "Андреевские заразы" (Москва)

Редактор:
Антон Волосатов (Ивантеевка), Екатерина Лобкова (Москва), Сергей
Терентьев (Санкт-Петербург); куратор - Олег Холодов (Серпухов)

Инфо:
Команда благодарит Константина Сахарова за ценные замечания.

Вопрос 1:
Название американской танцевальной песни перевели на русский одной из ее
строчек - "Я люблю кофе...". В оригинале название состоит из двух
созвучных слов. Напишите их.

Ответ:
"Java Jive" [чтецу: джАвэ джайв].

Комментарий:
Java - это известный сорт кофе, а jive - танец.

Источник:
   1. http://artistherd.ru/slova-kompozicii-java-jive/
   2. Перевод названия - по изданию: Популярная зарубежная музыка для
детского хора. - СПб.: Композитор, 2013. - С. 3.

Автор:
Антон Волосатов (Ивантеевка)

Вопрос 2:
В 1928 году место СлЕтса занял ДжЕки, который уже не был немым: при его
появлении включался фонограф. Назовите компанию, которая работала со
Слетсом и Джеки.

Ответ:
Metro-Goldwyn-Mayer [чтецу: мЭтро гОлдвин мЭйер].

Зачет:
MGM.

Комментарий:
Слетс и Джеки - львы, сменявшиеся на знаменитой заставке.

Источник:
http://www.adme.ru/tvorchestvo-reklama/istoriya-lva-iz-legendarnoj-zastavki-metro-goldwyn-mayer-335455/

Автор:
Екатерина Лобкова (Москва)

Вопрос 3:
В 1979 году компания "Омега" представила устройство, предназначенное для
спортивных соревнований. Рядом с устройством устанавливали динамики, а
само оно срабатывало по достижении определенного уровня давления.
Ответьте одним словом: что это устройство должно было определять?

Ответ:
Фальстарт.

Комментарий:
Часовая компания "Омега" разработала стартовые колодки, позволяющие с
точностью до сотой или тысячной доли секунды зафиксировать время старта
спортсмена. Стартуя, спортсмен создает давление на колодку, срабатывает
датчик, который передает сигнал на таймер, фиксирующий разницу между
временем отрыва ноги и выстрелом. Звук от стартового пистолета доходит
до дальней дорожки на несколько сотых позже, чем до ближайшей. Для того
чтобы бегуны находились в равных условиях, у каждой колодки стоял
динамик, передающий звук выстрела.

Источник:
   1. http://ru.wikipedia.org/wiki/Фальстарт
   2. http://www.findpatent.ru/patent/223/2234132.html
   3. http://www.nkj.ru/archive/articles/11393/

Автор:
Сергей Терентьев (Санкт-Петербург)

Вопрос 4:
На сайте отеля на час "Подушкин" есть изображение часовой и минутной
стрелки. Назовите в любом порядке то, в виде чего они выполнены.

Ответ:
Зеркало Венеры, щит [и копье] Марса.

Зачет:
Символ Венеры, символ Марса; женский символ, мужской символ (в любом
порядке).

Комментарий:
(pic: 20160350.jpg)
   Часы означают, что отель специализируется на почасовой сдаче номеров
для известных целей. Счастливые часов не наблюдают!

Источник:
http://www.podushkin.ru/hotel/7/

Автор:
Андрей Волыхов (Москва)

Вопрос 5:
[Ведущему: кавычки не озвучивать.]
   В историческом романе говорится, что, несмотря на ряд последующих
побед, ИХ "насест" долго не могли найти. Назовите ИХ двумя словами.

Ответ:
Золотые орлы.

Зачет:
Орлы легионов; орлы легионеров; римские орлы.

Комментарий:
Насестом Роберт Грейвз называет место, куда германцы спрятали отобранные
у римлян после битвы в Тевтобургском лесу штандарты легионов с
позолоченными орлами. Но если вы подумали про Карры и парфян, то тоже
неплохо.

Источник:
Р. Грейвз. Я, Клавдий. http://www.flibusta.is/b/175287/read

Автор:
Дмитрий Сахаров (Тутаев)

Вопрос 6:
Некоторое время в США ОНИ представляли собой уменьшенные копии
спортивных снарядов. Новичкам одного из департаментов ИХ часто вручали
со словами: "Это тебе вместо мозгов". Назовите ИХ.

Ответ:
Полицейские дубинки.

Зачет:
Резиновые дубинки; резиновые палки; полицейские палки.

Комментарий:
Дубинки вручали полицейским, не обладавшим навыками единоборств. В итоге
большинство полицейских без особых затей, как пещерные люди, били палкой
по головам преступников. Одной из самых популярных дубинок была так
называемая "Billy club" [бИлли клаб], которая представляла собой
небольшую бейсбольную биту. Дубинами, кстати, также называют глупых
людей. :-)

Источник:
Алекс Левитас. "Это тебе вместо мозгов...". // Журнал "Мастер-ружье",
2002, N 65. - С. 82-85.

Автор:
Сергей Терентьев (Санкт-Петербург)

Вопрос 7:
Переводчик Максим Немцов использовал слегка измененное слово
"барабУлька". Вместо какого слова?

Ответ:
Рыбка-бананка.

Зачет:
Bananafish; селедка (есть и такой перевод).

Комментарий:
Немцов известен своими оригинальными переводами зарубежной прозы. Так, у
него "Над пропастью во ржи" стала "Ловцом на хлебном поле", а переводя
"A Perfect Day for Bananafish" [э пёфэкт дэй фо бананафиш] Немцов
использовал неологизм "банабулька" вместо привычной нам рыбки-бананки.
Барабулька - реально существующая рыбка из отряда окунеобразных.

Источник:
http://n2.bg.ru/society/slovar_2008-16230/

Автор:
Дмитрий Сахаров (Тутаев)

Вопрос 8:
По мнению ХАрма де Блея, во время природных бедствий только
взаимовыручка спасает людей, которые раньше могли даже не общаться с
соседями. Географ называет разрушительную мощь природы ИМ. "ОН" -
подзаголовок книги Дэниела АлЕфа. Чья биография приведена в этой книге?

Ответ:
[Сэмюэла] Кольта.

Комментарий:
Блей называет стихийные бедствия Великим Уравнителем. Аналогичное
прозвище закрепилось и за известным оружейником.

Источник:
   1. Harm de Blij. The Power of Place. P. 255.
   2. http://www.amazon.com/Samuel-Colt-Equalizer-Titans-Fortune-ebook/dp/B0024FARAC/

Автор:
Екатерина Лобкова (Москва)

Вопрос 9:
Внимание, в вопросе слово "ТАКОЙ" заменяет другое слово.
   Артем Зырянов назвал ТАКИМ звено, где играли хоккеисты Ларионов,
Макаров и ГарпенлОв. Что ТАКОЕ появилось во второй половине XIX века?

Ответ:
Стиль [в архитектуре].

Комментарий:
ТАКОЙ - это псевдорусский. Гарпенлов - швед по национальности, несмотря
на традиционный для русских фамилий суффикс "-ов". Болельщики НХЛ
считали, что в звене играли три россиянина. Заметим, что более
правильной транскрипцией фамилии является "Гарпенлёв".

Источник:
http://www.sports.ru/hockey/1032134344.html

Автор:
Дмитрий Сахаров (Тутаев)

Вопрос 10:
Один из лозунгов пикета, направленного против экспонатов кощунственного
содержания, гласил, что, например, на яблоки ОНИ нашлись. Назовите ИХ.

Ответ:
Бульдозеры.

Комментарий:
Лозунг пикета является отсылкой к "Бульдозерной выставке" и заодно
напоминает об уничтожении санкционных продуктов.

Источник:
http://www.nsktv.ru/news/obshchestvo/v_novosibirske_proshel_piket_protiv_vystavki_skulptury_kotorykh_my_ne_vidim_230820151712/

Автор:
Сергей Терентьев (Санкт-Петербург)

Вопрос 11:
В песне Юрия Гарина вагоны считают себя ЕЮ столичного метро. Впрочем, в
последнее время сравнение стало менее точным. Назовите ЕЕ двумя словами,
начинающимися на парные согласные.

Ответ:
Голубая кровь.

Комментарий:
Сейчас более популярны другие цвета вагонов.

Источник:
http://www.bards.ru/archives/part.php?id=28960

Автор:
Андрей Волыхов (Москва)

Вопрос 12:
В начале фильма, действие которого происходит во Франции, персонаж
говорит, что, для того чтобы завладеть ключом от квартиры, ему пришлось
задабривать консьержку. Напишите слово, которое мы заменили в предыдущем
предложении.

Ответ:
Умасливать.

Комментарий:
Это фильм "Последнее танго в Париже". Широко известна другая его сцена,
где масло используется не по прямому назначению. Так что фраза героя
Марлона Брандо об умасливании выглядит намеком на предстоящую сцену.

Источник:
Х/ф "Последнее танго в Париже" (1972), реж. Бернардо Бертолуччи, 9-я
минута.

Автор:
Сергей Терентьев (Санкт-Петербург)

Вопрос 13:
Дуплет.
   1. Назовите африканскую страну, посольство которой располагается в
доме 14/9 [четырнадцать дробь девять] по Коробейникову переулку.
   2. Назовите балканскую страну, посольство которой располагается в
доме 16/10 [шестнадцать дробь десять] по Коробейникову переулку.

Ответ:
   1. Кот-д'Ивуар.
   2. Хорватия.

Комментарий:
Александр Коробейников известен своей любовью к слонам и к шахматам.

Источник:
   1. Карта Москвы.
   2. http://cotedivoire.ahibo.com/AmbM.htm
   3. http://ru.mfa.hr/ru/veleposlanstvo0/

Автор:
Екатерина Лобкова, Андрей Волыхов (Москва)

Вопрос 14:
Герой фильма - андроид, обладающий большим умом, но ограниченным
временем жизни, - хочет встретиться с создателем, чтобы продлить свои
дни. Символично, что во время встречи двое героев воспроизводят ЕЕ.
Назовите фамилию любого из ЕЕ участников.

Ответ:
[Адольф] Андерсен.

Зачет:
[Лионель] Кизерицкий.

Комментарий:
ОНА - это "Бессмертная партия" романтического периода шахмат. Андроид
совершает дерзкий гамбит ферзя и обыгрывает создателя. Забавно, что к
2019 году, в котором происходит фильм, эту классику уже успели
подзабыть. Соседний вопрос мог вам помочь.

Источник:
   1. https://www.quora.com/Why-cant-Tyrell-see-checkmate-in-the-film-Blade-Runner
   2. http://www.youtube.com/watch?v=sQuI3onaW3Y

Автор:
Антон Волосатов (Ивантеевка)

Вопрос 15:
   <раздатка>
   Долговечный вариант
   </раздатка>
   В качестве доступной альтернативы Бретт и Кейт МакКей предлагают
следующий долговечный вариант: два конца проволоки, связанные морским
узлом. Ответьте двумя словами: вариант чего?

Ответ:
Обручального кольца.

Зачет:
Помолвочного кольца.

Комментарий:
Проволока, разумеется, нужна золотая или серебряная, но в любом случае
выйдет достаточно красиво и при этом дешевле бриллиантов (которые, как
известно, навсегда). Кроме того, такая форма кольца традиционно
используется моряками. Розданная фраза отсылает к сетевой забаве -
подставлять любые подходящие по размеру словосочетания на место
"обручального кольца" в одноименной песне.

Источник:
   1. http://www.artofmanliness.com/2014/06/11/5-alternatives-to-the-diamond-engagement-ring/
   2. https://twitter.com/neprostoe

Автор:
Антон Волосатов (Ивантеевка)

Вопрос 16:
Писатель Уильям Гибсон прославился романами в киберпанк-стилистике, но
потом переключился на проблемы современного мира. В анонсе нового романа
Гибсона издательство "Азбука" пишет, что "главный визионер современности
наконец вернулся...". Закончите фразу тремя словами.

Ответ:
"... назад в будущее".

Комментарий:
Новый роман "Периферийные устройства" - снова о будущем.

Источник:
http://sibkron.livejournal.com/396222.html

Автор:
Руслан Хаиткулов (Москва)

Вопрос 17:
Писатель Гор Видал был известен как бескомпромиссный критик американской
политической системы, участник демонстраций и защитник прав человека, а
от его произведений консерваторов просто тошнило. Название одного из
этих произведений прямо говорит, что Видал ДЕЛАЕТ ЭТО. Ответьте двумя
словами: что именно делает?

Ответ:
Раскачивает лодку.

Комментарий:
Гор Видал был видной пятой колонной и раскачивал лодку в Штатах, отчего
тошнило местных крыс.

Источник:
http://ru.wikipedia.org/wiki/Видал,_Гор

Автор:
Руслан Хаиткулов (Москва)

Вопрос 18:
Как мы определили бы жанр документа, который в средневековой Руси
называли сговОрной полюбовной прутИвной записью?

Ответ:
Брачный контракт.

Комментарий:
СговОрная - договор, полюбовная - добровольная, прутИвная (противная) -
составленная для двух сторон.

Источник:
http://www.philol.msu.ru/faculty/press-center/events/lomonosov-readings-2010/

Автор:
Екатерина Лобкова (Москва)

Вопрос 19:
(pic: 20160351.jpg)
   Говоря о положении мотогонщиков, комментатор в шутку использовал
прилагательное, которое пишется через дефис. Напишите его.

Ответ:
Коленно-локтевая.

Зачет:
В любой форме.

Комментарий:
Мотогонщики нередко касаются асфальта в поворотах не только коленом, но
даже локтем.

Источник:
Трансляции MotoGP 2015 года.

Автор:
Андрей Волыхов (Москва)

Вопрос 20:
   <раздатка>
   Однажды он, пролечив головную боль
   полгода, с курорта скучного возвратится -
   и к милой Мими с мимозами разлетится,
   ан поздно! Она не Мими уж, а ми-бемоль.
   </раздатка>
   Александр Шапиро, комментируя приведенный отрывок из песни Михаила
Щербакова, вспоминает музыкальный жаргон, в котором "бемоль"
обозначает... Что?

Ответ:
Беременность.

Комментарий:
Помимо характерной формы знака, слово "бемоль" начинается на "б" и
заканчивается на "ь".

Источник:
http://m-sch.livejournal.com/540191.html

Автор:
Андрей Волыхов (Москва)

Вопрос 21:
Характеризуя отсталость России в XIX веке, Георгий ГловЕли пишет:
"Слишком много ЕГО в земледелии, слишком мало ЕГО в промышленности".
Назовите ЕГО одним словом.

Ответ:
Пар.

Комментарий:
В земледелии до тех самых пор существовала устаревшая система трехполья,
где одно из полей каждый год оставалось под паром, а вот в
промышленности паровых машин было еще мало.

Источник:
Г. Гловели. Экономическая история. - М.: Юрайт, 2014.

Автор:
Руслан Хаиткулов (Москва)

Вопрос 22:
Один из фанатов группы "Peking Duk" [пикИн дак] прошел за кулисы,
предъявив охраннику удостоверение личности и смартфон. Ответьте: что
фанат сделал непосредственно перед этим?

Ответ:
Отредактировал статью Википедии о группе "Peking Duk".

Зачет:
По смыслу.

Комментарий:
Молодой человек зашел со смартфона в Википедию, где в посвященной группе
статье обозначил себя родственником одного из музыкантов. Предъявив
охране новую версию статьи, фанат совершенно легально проследовал в
гримерку.

Источник:
https://tjournal.ru/p/peking-duk-fan

Автор:
Антон Волосатов (Ивантеевка), по наводке Руслана Хаиткулова (Москва)

Вопрос 23:
Греческое имя Ирина означает "мир". Персонаж шекспировской пьесы,
посчитав, что оно происходит из английского языка, назвал этим именем...
Что?

Ответ:
Шпагу.

Зачет:
Меч; клинок.

Комментарий:
Пистоль, персонаж "Генриха IV", считал, что имя происходит от
английского слова "iron" [айрн] - железо.

Источник:
А. Азимов. Путеводитель по Шекспиру. Английские пьесы.
http://www.flibusta.is/b/402786/read

Автор:
Сергей Терентьев (Санкт-Петербург)

Вопрос 24:
Участие в средневековых спектаклях оплачивалось в зависимости от
сложности роли. Так, например, за роль одного из апостолов полагалось 18
пенсов, а человеку, который за сценой ДЕЛАЛ ЭТО, - 4 пенса. Какие два
слова, восходящие к одному корню, мы заменили словами "ДЕЛАЛ ЭТО"?

Ответ:
Пел петухом.

Комментарий:
Иисус предсказал Петру, что тот отречется от Него до того, как пропоет
петух. Когда петух пропел, Петр вспомнил об этих словах и горько
раскаялся. Петь петухом - несложная задача, так что платили за нее
немного.

Источник:
http://istoriya-teatra.ru/books/item/f00/s00/z0000025/st001.shtml

Автор:
Сергей Терентьев (Санкт-Петербург)

Тур:
25 тур. "Самсон" (сборная)

Вопрос 1:
Александра Новикова иногда называют королем русского шансона. Говоря о
манере Новикова держать микрофон, один из членов жюри шоу "Точь-в-точь"
обратил внимание на ИКС. Назовите ИКС одним словом.

Ответ:
Мизинец.

Комментарий:
Если человек держит чашку с оттопыренным мизинцем, это могут счесть
признаком аристократизма.

Источник:
http://www.youtube.com/watch?v=vUW_i6xfc3Y

Автор:
Александр Марков (Москва)

Вопрос 2:
[Ведущему: читать текст очень быстро.]
   Я откупорю пробочку,
   А рядом кореша.
   Пивка хотит и водочки
   Их русская душа.
   Сегодня с загуделого
   Играем и поем,
   А завтра дело сделаем -
   И снова заживем.
   Название песни Михаила Круга, отрывок из которой вы только что
прослушали, пишется через дефис. Автор вопроса нашел в Интернете четыре
различных варианта этого названия. Напишите любой из них.

Ответ:
"Зек-рэп".

Зачет:
"Зек-реп"; "Зэк-реп"; "Зэк-рэп".

Комментарий:
К слову, этот вопрос - оммаж
(http://db.chgk.info/question/monos14.3/10).

Источник:
   1. http://www.mkrug.ru/txt/024.html
   2. Поиск по аудиозаписям ВКонтакте по запросам "Зек-реп", "Зэк-реп",
"Зэк-рэп".

Автор:
Александр Марков (Москва)

Вопрос 3:
РататОск в скандинавской мифологии - белка, бегающая по мировому древу.
Музыкант, взявший псевдоним Рататоск, в дебютном альбоме использовал ЕЕ.
ОНА выпускается, в том числе, под маркой "Stradivarius" [страдивАриус].
Назовите ЕЕ.

Ответ:
[Музыкальная] пила.

Комментарий:
Пила, как и белка, имеет отношение к дереву, хотя в скандинавских мифах
ясень Иггдрасиль грызет дракон Нидхёгг, а не Рататоск. На музыкальной
пиле играют смычком, отсюда брэнд "Stradivarius".

Источник:
   1. http://ru.wikipedia.org/wiki/Иггдрасиль
   2. https://ratatosk.bandcamp.com/album/cest-la-vie-tragique
   3. http://www.bahcoworld.com/blog/2012/09/stradivarius-musical-saw-by-bahco/

Автор:
Александр Марков (Москва)

Вопрос 4:
На афише одного мюзикла изображено разбитое яйцо, на котором нарисован
британский флаг. Название этого мюзикла - четырехзначное число. Напишите
это число.

Ответ:
1776.

Комментарий:
Из разбитого яйца выглядывает белоголовый орлан - символ США. Декларация
о независимости США была подписана 4 июля 1776 года.

Источник:
http://en.wikipedia.org/wiki/1776_(musical)

Автор:
Александр Марков (Москва)

Вопрос 5:
Религиовед Дмитрий Узланер называет ЕГО социальным лифтом. ЕГО иногда
называют шестым... Чем?

Ответ:
Столпом ислама.

Комментарий:
ОН - джихад. Для террористов-смертников лифт, вероятно, наиболее
эффективен.

Источник:
http://www.the-village.ru/village/city/city-news/175671-chto_novogo_religioved

Автор:
Юрий Разумов (Минск)

Вопрос 6:
Искусствовед отмечает холодные, бледные цвета на картине Сезанна,
названием которой стал ОН. Что не рекомендуют делать в НЕМ?

Ответ:
Говорить о веревке.

Комментарий:
ОН - дом повешенного.

Источник:
http://www.musee-orsay.fr/en/collections/works-in-focus/painting/commentaire_id/la-maison-du-pendu-8824.html?cHash=702eaca4ea

Автор:
Юрий Разумов (Минск)

Вопрос 7:
В 1977 году компания "Onitsukatiger" провела ребрендинг, использовав
латиноязычный девиз. Первое слово этого девиза - "Anima". Напишите
современное название этой компании.

Ответ:
ASICS.

Комментарий:
Сокращение латинской фразы "В здоровом теле здоровый дух" (Anima sana in
corpore sano) стало названием компании "ASICS".

Источник:
http://en.wikipedia.org/wiki/ASICS

Автор:
Евгений Пашковский (Москва)

Вопрос 8:
Фанаты подсчитали, что Губке Бобу должно быть не меньше 31 года, ведь в
одном из фильмов он говорит, что его 374 раза подряд признавали ИКСОМ.
Какие слова мы заменили ИКСОМ?

Ответ:
[Лучший] работник месяца [в "КрастиКрабс"].

Комментарий:
Даже если Губка Боб работал в "КрастиКрабс" с рождения, то ему должно
быть не меньше 31 года и 2 месяцев. Правда, в его водительских правах
указано, что в 2016 году ему должно исполниться тридцать. Time paradox!

Источник:
https://www.reddit.com/r/plotholes/comments/2342tq/the_spongebob_movie_spongebob_is_over_50_years_old/

Автор:
Александр Марков (Москва)

Вопрос 9:
   <раздатка>
   Скифы
   &nbsp;
   Мильоны - вас. Нас - тьмы, и тьмы, и тьмы.
   Попробуйте, сразитесь с нами!
   Да, скифы - мы! Да, азиаты - мы,
   С раскосыми и жадными очами!
   &nbsp;
   Для вас - века, для нас - единый час.
   Мы, как послушные холопы,
   Держали щит меж двух враждебных рас
   Монголов и Европы!
   &nbsp;
   Века, века ваш старый горн ковал
   И заглушал грома, лавины,
   И дикой сказкой был для вас провал
   И Лиссабона, и Мессины!
   &nbsp;
   Вы сотни лет глядели на Восток
   Копя и плавя наши перлы,
   И вы, глумясь, считали только срок,
   Когда наставить пушек жерла!
   &nbsp;
   Вот - срок настал. Крылами бьет беда,
   И каждый день обиды множит,
   И день придет - не будет и следа
   От ваших Пестумов, быть может!
   &nbsp;
   О, старый мир! Пока ты не погиб,
   Пока томишься мукой сладкой,
   Остановись, премудрый, как Эдип,
   Пред Сфинксом с древнею загадкой!
   &nbsp;
   Россия - Сфинкс. Ликуя и скорбя,
   И обливаясь черной кровью,
   Она глядит, глядит, глядит в тебя
   И с ненавистью, и с любовью!...
   &nbsp;
   Да, так любить, как любит наша кровь,
   Никто из вас давно не любит!
   Забыли вы, что в мире есть любовь,
   Которая и жжет, и губит!
   &nbsp;
   Мы любим всё - и жар холодных числ,
   И дар божественных видений,
   Нам внятно всё - и острый галльский смысл,
   И сумрачный германский гений...
   &nbsp;
   Мы помним всё - парижских улиц ад,
   И венецьянские прохлады,
   Лимонных рощ далекий аромат,
   И Кельна дымные громады...
   &nbsp;
   Мы любим плоть - и вкус ее, и цвет,
   И душный, смертный плоти запах...
   Виновны ль мы, коль хрустнет ваш скелет
   В тяжелых, нежных наших лапах?
   &nbsp;
   Привыкли мы, хватая под уздцы
   Играющих коней ретивых,
   Ломать коням тяжелые крестцы,
   И усмирять рабынь строптивых...
   &nbsp;
   Придите к нам! От ужасов войны
   Придите в мирные объятья!
   Пока не поздно - старый меч в ножны,
   Товарищи! Мы станем - братья!
   &nbsp;
   А если нет - нам нечего терять,
   И нам доступно вероломство!
   Века, века вас будет проклинать
   Больное позднее потомство!
   &nbsp;
   Мы широко по дебрям и лесам
   Перед Европою пригожей
   Расступимся! Мы обернемся к вам
   Своею азиатской рожей!
   &nbsp;
   Идите все, идите на Урал!
   Мы очищаем место бою
   Стальных машин, где дышит интеграл,
   С монгольской дикою ордою!
   &nbsp;
   Но сами мы - отныне вам не щит,
   Отныне в бой не вступим сами,
   Мы поглядим, как смертный бой кипит,
   Своими узкими глазами.
   &nbsp;
   Не сдвинемся, когда свирепый гунн
   В карманах трупов будет шарить,
   Жечь города, и в церковь гнать табун,
   И мясо белых братьев жарить!...
   &nbsp;
   В последний раз - опомнись, старый мир!
   На братский пир труда и мира,
   В последний раз на светлый братский пир
   Сзывает варварская лира!
   </раздатка>
   По мнению Андрея Синявского, в одной из строк стихотворения "Скифы"
Александру Блоку пришлось прибегнуть к инверсии, чтобы избежать наглой
рифмы. Напишите эту строку в таком виде, чтобы эта рифма становилась
неизбежной.

Ответ:
Перед пригожею Европой.

Комментарий:
Очень мало частей тела рифмуются с "Европой".

Источник:
   1. http://www.polemics.ru/articles/?articleID=14314
   2. http://www.stihi-rus.ru/1/Blok/147.htm

Автор:
Дмитрий Карякин (Москва)

Вопрос 10:
Блиц.
   (pic: 20160352.jpg)
   1. Назовите то, что обозначается розданным символом.
   2. Назовите язык, занимающий 13-е место по распространенности в
Интернете.
   3. Назовите одним словом группу пищевых добавок с обозначениями от
E410 до E419.

Ответ:
   1. Драм.
   2. Фарси.
   3. Камеди.

Источник:
   1. http://ru.wikipedia.org/wiki/Символ_драма
   2. http://w3techs.com/technologies/overview/content_language/all
   3. http://ru.wikipedia.org/wiki/Пищевые_добавки

Автор:
Илья Богданов (Москва)

Вопрос 11:
В своей "Истории западной философии" Бертран Рассел пишет, что имена
двух основателей этого учения почти всегда упоминаются вместе, поэтому
выделить вклад каждого из них очень сложно. Назовите это учение.

Ответ:
Атомизм.

Комментарий:
Основатели учения о неделимых элементах Демокрит и Левкипп практически
неотделимы друг от друга.

Источник:
Б. Рассел. История западной философии.
http://www.flibusta.is/b/104472/read

Автор:
Егор Игнатенков (Москва)

Вопрос 12:
   <раздатка>
   Volgograd
   </раздатка>
   В восьмидесятых годах чиновники из ГДР сменили советские автомобили
на европейские. Вскоре после этого закрытый поселок восточногерманской
элиты получил прозвище, которое мы вам раздали. Какую букву мы заменили
в этом названии?

Ответ:
V.

Комментарий:
Были закуплены машины марки "Volvo".

Источник:
http://en.wikipedia.org/wiki/Waldsiedlung

Автор:
Михаил Новосёлов (Москва)

Вопрос 13:
   <раздатка>
   Ich weine um Chinas: __ __ __
   Я плачу по китайцам: __ _____
   </раздатка>
   Перед вами строчка из современной песни Шорша Камеруна и ее перевод
на русский. Восстановите то, что мы пропустили в любой из розданных вам
строчек.

Ответ:
Ai wei wei.

Зачет:
По-русски.

Комментарий:
Имя китайского художника Ай Вэйвэя звучит как звуки плача - особенно
если читать дифтонг "ei" на немецкий лад. Сейчас Ай Вэйвэй живет в
Берлине.

Источник:
Schorsch Kamerun "Ich meine ich weine".

Автор:
Антон Пинчук (Москва)

Вопрос 14:
Шейх Ахмед бин Саид аль-Мактум, руководитель компании, занимающейся
строительством искусственного архипелага Джузур аль-Алам, стал ИКСОМ в
2008 году. Произведение "ИКС" было написано в 1970, а не в 1993 году.
Назовите это произведение.

Ответ:
"The man who sold the world".

Зачет:
"Человек, продавший мир".

Комментарий:
Искусственный архипелаг у побережья эмирата Дубаи имеет форму карты мира
и называется Мир. Песню "The man who sold the world" написал Дэвид Боуи,
хотя, к сожалению, наиболее известна версия в исполнении группы
"Nirvana".

Источник:
   1. http://en.wikipedia.org/wiki/The_World_(archipelago)
   2. http://en.wikipedia.org/wiki/Nakheel_Properties
   3. http://en.wikipedia.org/wiki/The_Man_Who_Sold_the_World

Автор:
Дмитрий Карякин (Москва)

Вопрос 15:
В одной из новелл "Декамерона" к путешествующему аббату присоединяется
красивый юноша. Добравшись до ближайшего селения, аббат решает бросить
якорь. Какое слово в тексте вопроса мы заменили двумя?

Ответ:
Пристать.

Комментарий:
Аббат оказывается девушкой, скрывающейся под видом монаха. В
"Декамероне", с сожалением отмечает автор вопроса, вообще мало геев.

Источник:
Дж. Боккаччо. Декамерон. http://www.flibusta.is/b/7795/read

Автор:
Юрий Разумов (Минск)

Вопрос 16:
Когда Георг ПлАчек предложил использовать ИКС для замедления нейтронов,
Ферми ответил, что это просто смешно. Назовите ИКС.

Ответ:
Гелий.

Источник:
Richard Rhodes. Making of the Atomic Bomb.
https://books.google.ru/books?id=aSgFMMNQ6G4C&pg=PA300#v=onepage&q&f=false

Автор:
Юрий Разумов (Минск)

Вопрос 17:
ОН встречается как в воспоминаниях о детстве известного писателя, так и
в заглавии романа человека, который считал этого писателя коллективным
псевдонимом группы коммунистов. Назовите ЕГО.

Ответ:
Высокий замок.

Комментарий:
Детство Станислава Лема прошло во Львове, а Филип Дик, автор романа
"Человек в высоком замке", всюду искал коммунистический заговор.

Источник:
   1. http://ru.wikipedia.org/wiki/Высокий_замок_(роман)
   2. http://www.culture.pl/ru/article/filip-kindred-dik-stanislav-lem-mirovoy-zagovor-kommunistov
   3. http://ru.wikipedia.org/wiki/Человек_в_высоком_замке

Автор:
Дмитрий Карякин (Москва)

Вопрос 18:
Когда Стивен Вольфрам придумывал название программы, которая будет верно
служить ученым всего мира, он в шутку рассматривал и имя, вероятно
имеющее скандинавское происхождение. Напишите это имя.

Ответ:
Игорь.

Комментарий:
Помощников безумных ученых в западной поп-культуре часто называют
Игорями, этот образ сложился из нескольких классических фильмов ужасов.
Программу Вольфрам в итоге назвал просто "Mathematica". Никакой
фантазии.

Источник:
   1. http://www.mathematica25.com/data/uploads/1987_NamingMathematica_image.jpg
   2. http://en.wikipedia.org/wiki/Igor_(character)
   3. http://ru.wikipedia.org/wiki/Игорь

Автор:
Александр Марков (Москва)

Вопрос 19:
Внимание, в вопросе есть замены.
   "Лос-Анджелес Таймс" писала, что благодаря детищу Ирвина и Бёрта
вкусы американцев стали гораздо шире, чем белый, красный и коричневый.
Напишите любое из слов, которые мы заменили в предыдущем предложении.

Ответ:
Ваниль.

Зачет:
Клубника; шоколад; прилагательные, образованные от этих слов.

Комментарий:
До того как Ирвин Роббинс и Бёрт Баскин создали свою сеть кафе с
мороженым, для которой Роббинс разработал сотни новых вкусов,
ассортимент мороженого был невелик и ограничивался, в основном,
ванильным, шоколадным и клубничным.

Источник:
http://www.latimes.com/local/obituaries/la-me-robbins7-2008may07-story.html

Автор:
Александр Марков (Москва)

Вопрос 20:
В Германии ОН стал известен как Аль Кацоне. Под каким именем ОН наиболее
известен в России?

Ответ:
Толстопуз.

Комментарий:
Преступник-рецидивист Fat Cat, главный противник Чипа и Дейла, в
Германии получил кличку, схожую с именем другого преступника.

Источник:
http://de.wikipedia.org/wiki/Chip_und_Chap_%E2%80%93_Die_Ritter_des_Rechts

Автор:
Евгений Пашковский (Москва)

Вопрос 21:
(pic: 20160353.jpg)
   Acumin [акумИн] - новый шрифт от "Adobe" [адОб]. Когда дизайнер
Роберт СлИмбах увидел шрифт Helvetica [гельвЕтика] в печати, он был
удивлен, так как гельветика изначально была шрифтом для заголовков,
вывесок и дорожных знаков. Он решил заново изобрести нео-гротеск,
соединив нейтральность с человечностью и дав издателям возможность
подобрать хороший нео-гротеск для печати. Акумин содержит десять
степеней насыщенности, лигатуры и настоящие курсивные гарнитуры. Джеффри
Зельдман сравнивает наслаждение акумином с наслаждением очень вкусной
ЕЮ. Назовите ЕЕ.

Ответ:
Вода.

Комментарий:
Гельветика и Акумин - подчеркнуто нейтральные шрифты, поэтому логично
сравнение с водой. В тексте вопроса много "воды" - ненужной для взятия
информации. А раздатку мы раздали вам просто потому, что на акумин можно
смотреть бесконечно, так же, как на текущую воду.

Источник:
https://medium.com/@zeldman/a-helvetica-for-readers-f5dd1e6e80ef

Автор:
Александр Печёный (Москва)

Вопрос 22:
ИКСОМ в этом вопросе мы заменили другое слово.
   Во время Второй мировой войны в Европе наблюдался дефицит бензина. По
мнению Джея Стивенса, именно это вынудило известного ученого
воспользоваться ИКСОМ. Назовите этого ученого.

Ответ:
Альберт Хофманн.

Комментарий:
Из-за дефицита бензина, по предположению Стивенса, изобретатель LSD
воспользовался велосипедом. Знаменитый "день велосипеда" произошел 19
апреля 1943 года.

Источник:
Дж. Стивенс. Штурмуя небеса. http://www.flibusta.is/b/123642/read

Автор:
Михаил Новосёлов (Москва)

Вопрос 23:
Спасаясь от наводнения, персонажи фильма "Шаркнадо" отправляются в
городок, чей побратим - Канны. Назовите этот городок.

Ответ:
Беверли-Хиллз.

Комментарий:
Спасаясь от наводнения, логично двигаться наверх.

Источник:
   1. Х/ф "Шаркнадо" (2013), реж. Энтони Ферранте.
   2. http://en.wikipedia.org/wiki/Beverly_Hills,_California

Автор:
Юрий Разумов (Минск)

Вопрос 24:
Внимание, в вопросе дважды пропущены одни и те же несколько слов.
   Креационисты считают, что бог не [ПРОПУСК], потому что он ее создал.
Назовите любую сущность, которая [ПРОПУСК] в произведении конца XX века.

Ответ:
Бледный бармен с дрожащей рукой.

Зачет:
Дыма табачного пленный; та, что согласна ехать со мной; счетчик такси,
похожий на пульс; звезды - следы трассирующих пуль; черствый коньяк
разбавленных лет; сжатые нервно колени; губы твои; тусклый рассвет в
холодном окне; время конвейерной лентой; всё, что сейчас происходит во
мне; а также любой осмысленный набор слов из песни "Вселенная", которую
исполняет Александр Иванов.

Комментарий:
По мнению креационистов, бог не является частью вселенной. По мнению
автора текста песни "Вселенная" Александра Шаганова, частью вселенной
является много чего.

Источник:
   1. Nicholas Everitt, "The non-existence of God".
   2. http://www.1tv.ru/prj/dostrespub/vypusk/9884

Автор:
Александр Марков (Москва)

Тур:
26 тур. "Гидрою контры убит Чаушеску" (Раменское)

Редактор:
Данила Аладин (Пущино)

Вопрос 1:
По воспоминаниям Бориса СелЕннова, в первом видеоролике снялся сын
главного редактора радиостанции "Юность", взлетевший затем до статуса
известного дизайнера. В более позднем ролике можно было увидеть два
знака "меньше либо равно". Назовите телепередачу, в которой можно было
видеть эти ролики.

Ответ:
"До шестнадцати и старше".

Комментарий:
Как вы, наверное, помните, наш пакет на прошлом "Окском марафоне"
начался с вопроса про телепередачу "Марафон-15". Первая заставка
известного советского и российского молодежного шоу "До шестнадцати и
старше" представляла собой видеоролик, в котором тинейджер бежал за
камерой по взлетной полосе аэродрома. В более поздней заставке число 16
появлялось на экране в окружении двух знаков "меньше либо равно",
обыгрывая название передачи.

Источник:
   1. http://www.youtube.com/watch?v=egHej1j1Dgk&t=4m10s
   2. http://www.youtube.com/watch?v=fQUDNTcNCRM

Автор:
Виктор Бойков (Раменское)

Вопрос 2:
Приносим свои извинения за то, что не смогли подготовить к этому вопросу
раздаточный материал.
   Назовите фамилию девушки, чья фотография 25 июня 2015 года в 14:00
появилась в сообществе "Selfie Armenia" [сэлфи армения].

Ответ:
Сорян.

Комментарий:
Мы не зря извинились перед вами в начале вопроса. Похожее на армянскую
фамилию слово "сорян" - элемент русскоязычного молодежного сленга,
образованный от английского "sorry" ("извини"). Редакторская группа
также заранее приносит извинения всем, чьи ожидания этот пакет не
оправдает.

Источник:
   1. https://vk.com/selfie_armenia?w=wall-70772966_38863
   2. http://teenslang.su/id/18715

Автор:
Виктор Бойков (Раменское)

Вопрос 3:
Услышав о внедрении нового КОМПЛЕКСА ТЕХНИЧЕСКИХ СРЕДСТВ МОНИТОРИНГА,
позволяющего лучше контролировать подвижной состав, автор вопроса понял,
почему перевозчик использует как минимум два ИХ. Назовите ИХ двумя
словами.

Ответ:
Оттенки серого.

Комментарий:
Комплекс технических средств мониторинга, или КТСМ, - автоматизированная
система, применяемая, в частности, на российских железных дорогах.
Аббревиатура КТСМ на слух практически неотличима от буквосочетания БДСМ,
которым обозначается психосексуальная субкультура ролевых игр,
господства и подчинения, суть которой щедро раскрывается в кинофильме
"50 оттенков серого". В раскраске подвижного состава ОАО "РЖД" также
можно встретить несколько оттенков серого.

Источник:
   1. http://www.vse-lekcii.ru/zheleznodorozhnyj-transport/ats/ktsm
   2. http://ru.wikipedia.org/wiki/БДСМ

Автор:
Виктор Бойков (Раменское)

Вопрос 4:
Герой юмористической миниатюры спрашивает у коллег, что за цирк они при
нем устроили, и интересуется, не хотят ли они получить ПРОПУСК.
Возможно, ПРОПУСК вошел в название из-за того, что несколькими годами
раньше Ги освоил выдувание огня. Заполните пропуск двумя словами.

Ответ:
Дю Солей.

Зачет:
Du Soleil. Незачет: Солнце; Солнца.

Комментарий:
Цирковое шоу "Cirque du Soleil" ("Цирк Солнца"), название которого
перекликается с тем, что можно получить от недовольного собеседника, был
основан канадским артистом Ги Лалиберте. За несколько лет до основания
цирка Лалиберте в качестве уличного артиста совершил путешествие по
Европе, в ходе которого приобрел опыт участия в файер-шоу.

Источник:
   1. http://www.youtube.com/watch?v=LOE0x4MJV_w&t=1m24s
   2. http://en.wikipedia.org/wiki/Cirque_du_Soleil

Автор:
Виктор Бойков (Раменское)

Вопрос 5:
   <раздатка>
   Совести нет у ПЕРВЫЙ ПРОПУСК,
   Нет ума у ПЕРВЫЙ ПРОПУСК.
   Выльют пусть грязи хоть ушат -
   Стерпит ВТОРОЙ ПРОПУСК.
   </раздатка>
   Перед вами отрывок из пародийной песни, написанной в марте 2012 года
после обострения ситуации. Что мы заменили на ПЕРВЫЙ ПРОПУСК, а кого -
на ВТОРОЙ ПРОПУСК?

Ответ:
МАГАТЭ, Ахмадинежад.

Комментарий:
В начале 2012 года Исламская Республика Иран заявила об успешности своей
ядерной программы, что вызвало виток напряженности в ее отношениях с
Международным агентством по атомной энергии (МАГАТЭ) и западными
государствами. Махмуд Ахмадинежад - президент Ирана в 2005-2013 годах,
ускоривший развитие иранской ядерной программы.

Источник:
   1. http://www.youtube.com/watch?v=AkN2JG6M8vM&t=1m22s
   2. http://ru.wikipedia.org/wiki/Ядерная_программа_Ирана

Автор:
???

Вопрос 6:
Внимание, в вопросе есть замена.
   Согласно остроумному замечанию, руководитель КНДР Ким Чен Ын очень
любит суши-бары, так как там он может съесть по нескольку АЛЬФ в день. А
в произведении 1976 года АЛЬФА была... Чем?

Ответ:
Отелем.

Комментарий:
Шутят, что северокорейский лидер любит есть роллы "Калифорния", т.к. тем
самым наносит урон своему внешнеполитическому противнику в лице США.
Музыкальная композиция "Отель "Калифорния"" была записана группой
"Eagles" в 1976 году.

Источник:
   1. http://www.youtube.com/watch?v=ABNiW33ozK0&t=2s
   2. http://ru.wikipedia.org/wiki/Калифорния_(ролл)
   3. http://ru.wikipedia.org/wiki/Hotel_California_(альбом)

Автор:
Виктор Бойков (Раменское)

Вопрос 7:
По случаю переезда из Цинциннати в основанный в 1755 году город Шарлотт
американская агропромышленная компания выпустила рекламный плакат с
изображением раскрытого банана, стилизованного под АЛЬФУ. Какое слово мы
заменили на АЛЬФУ?

Ответ:
Корона.

Комментарий:
Город Шарлотт в Северной Каролине назван в честь Шарлотты
Мекленбург-Стрелицкой - жены английского короля Георга III. В связи с
этим город имеет прозвище "Queen City" ("Город Королевы").

Источник:
   1. http://www.bestadsontv.com/ad/41313/Charlotte-NC-The-Queens-Banana
   2. http://www.adweek.com/agencyspy/wednesday-odds-and-ends-134/27687
   3. http://en.wikipedia.org/wiki/Charlotte,_North_Carolina

Автор:
Евгений Кравченко (Москва)

Вопрос 8:
На известной картине эпохи Ренессанса изображены две женщины. При этом
старая возбужденно поднимает подол. Назовите имя молодой.

Ответ:
Даная.

Комментарий:
В древнегреческой мифологии Даная - дочь царя Аргоса, заключенная в
темницу вместе с приставленной к ней пожилой служанкой. Возжелавший
Данаю бог Зевс проник к ней в темницу в виде золотого дождя, увидев
который алчная служанка попыталась собрать золото в свой подол.

Источник:
   1. http://ru.wikipedia.org/wiki/Даная_(картина_Тициана,_1553%E2%80%941554)
   2. http://ru.wikipedia.org/wiki/Даная

Автор:
Илья Худеев (Раменское)

Вопрос 9:
   <раздатка>
   Лимонову удалось и обновить язык политики, и снабдить ее особым
культурным измерением, придать ______ ощущение _____.
   </раздатка>
   Перед вами цитата из книги Феликса Сандалова о жизни московского
андеграунда 90-х годов. Слова, которые мы в ней пропустили, можно
сказать, встречаются в самом известном из НИХ, появившемся на десять лет
раньше описываемых в книге событий. Назовите ИХ двумя словами,
начинающимися на две одинаковые или на две соседние буквы.

Ответ:
Russian Reversal.

Зачет:
Русские перевертыши.

Комментарий:
По мнению Сандалова, лимоновцы смогли "придать партии ощущение party".
Russian Reversal, или "русские перевертыши", - комедийный прием
построения фраз, в которых субъект и объект меняются местами. В 1980-х
годах "русские перевертыши" стали популярны в США благодаря одесскому
эмигранту Якову Смирнову, противопоставлявшему в своих юмористических
зарисовках советский и американский образы жизни. Один из самых
известных "перевертышей" Смирнова звучал так: "In Los Angeles, you can
always find a party. In Soviet Russia, the Party can always find YOU!"
("В Лос-Анджелесе вы всегда найдете тусовку. В Советской России партия
всегда найдет ТЕБЯ!").

Источник:
   1. http://magazines.russ.ru/znamia/2016/7/lef-xxi-veka.html
   2. https://lurkmore.co/Russian_Reversal

Автор:
Виктор Бойков (Раменское)

Вопрос 10:
В 1864 году выздоравливавший после ранения военный, желая скоротать
время, начал бросать иголку на лист линованной бумаги. Ответьте, что он
пытался таким образом определить как можно точнее.

Ответ:
Значение числа Пи.

Зачет:
Число Пи; Пи; &pi;.

Комментарий:
Указание на точность здесь относится к попытке участника Гражданской
войны в США капитана Фокса как можно точнее определить число Пи через
количество пересечений иголкой линий, длину иглы и расстояние между
линиями. Метод Монте-Карло, основанный на тех же принципах, появился
почти сто лет спустя.

Источник:
http://ru.wikipedia.org/wiki/Метод_Монте-Карло

Автор:
Евгений Кравченко (Москва)

Вопрос 11:
В Древней Греции Гекате, богине ночи и мрака, часто приносили в жертву
ТАКИХ ИХ. ТАКОГО ЕГО ранил "старый корабельный друг", впоследствии также
подвергшийся кровопусканию, которое совершил... Кто?

Ответ:
[Доктор] [Дэвид] Ливси.

Комментарий:
В жертву Гекате, в частности, приносили черных псов, т.к. они
ассоциировались с ее подземной свитой. В романе Роберта Стивенсона
"Остров сокровищ" пират по кличке Черный Пес был ранен своим подельником
Билли Бонсом, у которого сразу после схватки случился апоплексический
удар. Посетивший больного доктор Дэвид Ливси пустил Бонсу кровь в
медицинских целях.

Источник:
   1. http://en.wikipedia.org/wiki/Hecate#Cross-cultural_parallels
   2. Р.Л. Стивенсон. Остров сокровищ.

Автор:
Илья Худеев (Раменское)

Вопрос 12:
Рассказывая о путешествии по Италии, Марк Твен пишет, как в одном из
музеев чуть не ослеп, разглядывая бесконечные кары исторических картин.
Какое слово мы слегка изменили в предыдущем предложении?

Ответ:
Акры.

Комментарий:
Акр - единица площади из английской системы мер, используемой также и в
США. Вполне логично, что американец Твен употребил в описании близкий
ему термин.

Источник:
   1. М. Твен. Простаки за границей.
http://www.flibusta.is/b/389718/read
   2. http://ru.wikipedia.org/wiki/Акр

Автор:
Сергей Филин (Красноармейск)

Вопрос 13:
В фильме "Черчилль идет на войну" дворецкий, прочитав название книги,
решил, что ее главный герой оказался в тюрьме для гомосексуалистов.
Назовите российскую музыкальную группу, выпустившую в 2000 году альбом,
название которого всего несколькими буквами отличается от названия
данной книги.

Ответ:
"Агата Кристи".

Комментарий:
Дворецкий Букингемского дворца обнаруживает сочинение Адольфа Гитлера
"Mein Kampf" и пытается прочесть ее название на английский манер "Me in
camp F" ("Я в лагере Ф"), что и наводит его на столь подозрительные
мысли. Екатеринбургский музыкальный коллектив "Агата Кристи" выпустил в
2000 году известный альбом под названием "Майн Кайф?".

Источник:
   1. Х/ф "Черчилль идет на войнцу" (2004), реж. Питер Ричардсон,
0:05:54.
   2. http://ru.wikipedia.org/wiki/Моя_борьба
   3. http://ru.wikipedia.org/wiki/Майн_Кайф%3F

Автор:
Виктор Бойков (Раменское)

Вопрос 14:
(pic: 20160354.jpg)
   Перед вами часть шуточного постера, появившегося во второй половине
2015 года и сопровождавшегося подписью "В нашей истории бывали
информационные поводы и похуже!". Назовите то, чему он был посвящен,
сложносоставным неологизмом англо-немецкого происхождения.

Ответ:
Дизельгейт.

Зачет:
Dieselgate.

Комментарий:
На раздаточном материале Адольф Гитлер и другие деятели Третьего рейха
рассматривают скрытый нами макет автомобиля "Фольксваген Жук". В
сентябре 2015 года была раскрыта афера немецкого концерна "Фольксваген",
оснащавшего свои автомобили программным обеспечением, занижавшим
количество вредных выхлопных газов, выбрасываемых в воздух дизельными
двигателями. По аналогии с Уотергейтским скандалом, приведшим в 1974
году к импичменту президента США Ричарда Никсона, ситуация с
"Фольксвагеном" получила в прессе наименование "Дизельгейт". Рудольф
Кристиан Карл Дизель (1858-1913) - немецкий инженер и изобретатель,
создатель модели поршневого двигателя внутреннего сгорания, названного
его именем.

Источник:
   1. https://hahahahah.dirty.ru/chestno-govoria-u-nas-byvali-informatsionnye-povody-i-pokhuzhe-982573/
   2. http://royaldutchshellplc.com/1938/07/06/hitler-during-his-visit-to-the-vw-factory-in-1938/
   3. http://ru.wikipedia.org/wiki/Дело_Volkswagen
   4. http://ru.wikipedia.org/wiki/Уотергейтский_скандал

Автор:
Виктор Бойков (Раменское)

Вопрос 15:
Любящие свою родину, но не очень грамотные герои мини-сериала "Польска
милицейка" наклеивают на свой автомобиль дополнительные буквы, из-за
чего может показаться, что речь в сериале идет о голландских стражах
порядка. Назовите модель этого автомобиля.

Ответ:
[Volkswagen] Polo.

Зачет:
[Фольксваген] Поло. Незачет: Volkswagen; Фольксваген.

Комментарий:
Герои подклеили к слову "Polo" на багажнике служебного Volkswagen'а
латинские буквы N и D, в результате чего у них получилось слово
"Polond", с точностью до одной грамматической ошибки повторяющее
название Польши на английском языке.

Источник:
   1. http://www.youtube.com/watch?v=vInaxNIj-hc&t=58s
   2. http://en.wikipedia.org/wiki/Poland

Автор:
Виктор Бойков (Раменское)

Вопрос 16:
Внимание, словом "ИКС" в вопросе заменено другое слово.
   В романе Германа Мелвилла священник говорит, что рассказанная им
история - "лишь тончайшая нить, вплетенная в ИКС Писания". Сын другого
священника, говоря о людях, упомянул ИКС... Где?

Ответ:
Над пропастью.

Зачет:
Между животным и сверхчеловеком.

Комментарий:
ИКСОМ мы заменили слово "канат". Американский писатель Герман Мелвилл
известен своими произведениями о море и мореходстве. Неудивительно, что
в его романе "Моби Дик" священник сравнивает Священное Писание с
канатом. Немецкий писатель и философ Фридрих Ницше, родившийся в семье
лютеранского пастора, в своей книге "Так говорил Заратустра" сравнил
человека с канатом, натянутым над пропастью между животным и
сверхчеловеком.

Источник:
   1. Г. Мелвилл. Моби Дик, или Белый Кит.
http://www.flibusta.is/b/166245/read
   2. Ф. Ницше. Так говорил Заратустра.
http://www.flibusta.is/b/406494/read
   3. http://ru.wikipedia.org/wiki/Ницше,_Фридрих

Автор:
Илья Худеев (Раменское)

Вопрос 17:
Внимание, в вопросе есть замены.
   Известный автогонщик после 1996 года получил прозвище "ТАКОЙ ОН".
Предпочитавший особую форму одежды русский военачальник носил прозвище
"СЯКОЙ ОН". Назовите родившегося в 1936 году итальянца, которого
спортивный комментатор однажды назвал ТАКИ-СЯКИМ ИМ.

Ответ:
[Сильвио] Берлускони.

Комментарий:
Немецкий автогонщик Михаэль Шумахер, выступая в ярко-красных цветах
итальянской гоночной команды "Феррари", получил прозвище "красный барон"
по аналогии с немецким летчиком-асом Первой мировой войны Манфредом фон
Рихтгофеном. Русский военачальник времен Гражданской войны Петр Врангель
носил прозвище "черный барон" за ношение черной казачьей черкески.
Итальянский олигарх Сильвио Берлускони является владельцем футбольного
клуба "Милан", выступающего в красно-черных цветах.

Источник:
   1. http://ru.wikipedia.org/wiki/Шумахер,_Михаэль
   2. http://ru.wikipedia.org/wiki/Врангель,_Пётр_Николаевич
   3. http://ru.wikipedia.org/wiki/Берлускони,_Сильвио
   4. ЛОАВ.

Автор:
Виктор Бойков (Раменское)

Вопрос 18:
Чешский теннисист Радек Штепанек за свою 20-летнюю одиночную карьеру
сумел выиграть только пять турниров высокого уровня. При этом Штепанек
известен своими любовными похождениями: в разные годы он встречался с
Мартиной Хингис, Николь Вайдишовой и Петрой Квитовой. По утверждению
посетителя сайта sports.ru, Штепанек входил в ПЕРВЫЙ ПРОПУСК чаще, чем
во ВТОРОЙ ПРОПУСК. Назовите в правильном порядке аббревиатуру, входящую
в ПЕРВЫЙ ПРОПУСК, и аббревиатуру, входящую во ВТОРОЙ ПРОПУСК.

Ответ:
WTA, ATP.

Зачет:
В правильном порядке.

Комментарий:
Любители тенниса поговаривают, что любвеобильный Штепанек чаще входил в
топ-10 Женской теннисной ассоциации (WTA), чем в топ-10 мужской
Ассоциации теннисистов-профессионалов (ATP).

Источник:
   1. http://www.sports.ru/tribuna/blogs/glaz_naroda/712061.html
   2. http://ru.wikipedia.org/wiki/Женская_теннисная_ассоциация
   3. http://ru.wikipedia.org/wiki/Ассоциация_теннисистов-профессионалов

Автор:
Георгий Горев (Москва)

Вопрос 19:
В канун одного из праздников западноевропейские футболисты записали
видеоролик, в котором использовались не только привычные музыкальные
инструменты, но и спортивный инвентарь. Название этого ролика одной
буквой отличается от названия произведения 1858 года. Воспроизведите
название ролика.

Ответ:
"Jingle Balls".

Зачет:
"Джингл Боллс".

Комментарий:
Незадолго до Рождества игроки дортмундского клуба "Боруссия" выпустили
видео, в котором с определенными интервалами били футбольными мячами по
подвешенным перед ними колоколам, звон которых складывался в известную
мелодию. "Jingle Bells" - популярная рождественская песня Джеймса Лорда
Пьерпонта, написанная в 1858 году.

Источник:
   1. http://www.youtube.com/watch?v=RKUrWXG7QH4
   2. http://ru.wikipedia.org/wiki/Jingle_Bells

Автор:
Виктор Бойков (Раменское)

Вопрос 20:
Фанаты пермского "Амкара" на одном из своих баннеров изобразили ЕГО,
угрожающе поднявшего руку. Автор вопроса, ожидавший увидеть ЕГО
изображение на атрибутике другой команды Премьер-лиги, решил, что ошибка
возникла из-за низкого качества нелицензионной копии. Назовите ЕГО имя и
фамилию.

Ответ:
Фредди Крюгер.

Комментарий:
По всей видимости, болельщики "Амкара", выступающего в красно-черных
цветах, решили изобразить на своем баннере Фредди Крюгера, т.к.
предполагали, что его свитер имеет красно-черную раскраску. На самом
деле Крюгер носил свитер в красную и зеленую полоску, поэтому его образ
уместнее было бы использовать фанатам московского "Локомотива".
Демонстрировавшийся в СССР и Российской Федерации пиратские копии
фильмов франшизы "Кошмар на улице Вязов" зачастую не могли похвастаться
хорошим качеством записи, поэтому черный и грязно-зеленый цвета на них
вполне можно было перепутать.

Источник:
   1. http://fanat1k.ru/news-44377-10-luchshih-bannerov-chempionata-rossii-2010.php
   2. http://ru.wikipedia.org/wiki/Фредди_Крюгер

Автор:
Виктор Бойков (Раменское)

Вопрос 21:
Певица Земфира однажды назвала ЕЕ своей тезкой. Какие слова фигурировали
вместе с НЕЙ в названии произведения 1961 года?

Ответ:
Тревоги нашей.

Комментарий:
Уменьшительно-ласкательный вариант имени Земфира может быть созвучен
слову "зима". "Зима тревоги нашей" - последний роман американского
прозаика Джона Стейнбека, опубликованный в 1961 году.

Источник:
   1. https://music.yandex.ru/album/81432/track/732413
   2. http://ru.wikipedia.org/wiki/Зима_тревоги_нашей

Автор:
Виктор Бойков (Раменское)

Вопрос 22:
В своих воспоминаниях о музыкальной группе "Интеграл" Бари Алибасов
рассказывает, как в 1965 году их "джаз-квинтет" превратился в
"бит-квинтет". Это время Алибасов называет "эпохой разгибания ИХ".
Назовите ИХ словом, появившимся не ранее 1842 года.

Ответ:
Саксофоны.

Комментарий:
В 60-х годах XX века советские власти проводили активную борьбу с
"тлетворными западными влияниями", в рамках которой джаз был признан
"идеологическим оружием буржуазии" и фактически оказался под запретом.
Саксофон - музыкальный инструмент, изобретенный Адольфом Саксом в 1842
году, - является важным элементом джазового ансамбля.

Источник:
   1. http://www.na-nax.com/alibasov/101-2010-06-02-17-36-51/2863-group-na-naalibasov-vladimir-politovvyacheslav-zherebkin-.html
   2. http://ru.wikipedia.org/wiki/Саксофон

Автор:
Виктор Бойков (Раменское)

Вопрос 23:
По словам Сергея, пощечина общественному вкусу была задумана им еще в
школе, когда он узнал, что АЛЬФА, приводя в негодность станки и гайки,
мешает строительству коммунизма. Мы не просим восстановить два слова,
замененные нами на АЛЬФУ. Назовите фамилию или псевдоним Сергея.

Ответ:
Троицкий.

Зачет:
Паук.

Комментарий:
Во время подготовки к экзамену по химии юный неформал, уже познавший
некоторые тяготы взаимодействия с советской системой, решил назвать свою
музыкальную группу "Коррозией металла" в пику тогдашней официальной
музыкальной идеологии. В ходе своей бурной андеграундной карьеры Сергей
Троицкий получил прозвище "Паук".

Источник:
   1. С. Троицкий. 9 лет трэш-угара Коррозии Металла.
http://www.flibusta.is/b/216801/read
   2. http://ru.wikipedia.org/wiki/Коррозия#.D0.9A.D0.BE.D1.80.D1.80.D0.BE.D0.B7.D0.B8.D1.8F_.D0.BC.D0.B5.D1.82.D0.B0.D0.BB.D0.BB.D0.BE.D0.B2

Автор:
Виктор Бойков (Раменское)

Вопрос 24:
   <раздатка>
   Изредка разве это провождение времени нарушится каким-нибудь
нечаянным случаем, когда, например, все ______ целым домом, от мала до
велика. Других болезней почти и не слыхать было в дому и деревне...
   &nbsp;
   ... Но ____ случался частенько. Тогда все валяются вповалку по
постелям; слышится оханье, стоны; один обложит голову огурцами и
повяжется полотенцем, другой положит клюквы в уши и нюхает хрен, третий
в одной рубашке уйдет на мороз, четвертый просто валяется без чувств на
полу. Это случалось периодически один или два раза в месяц, потому что
тепла даром в трубу пускать не любили...
   </раздатка>
   Надеемся, что нынешний "Окский марафон" доставит вам только
положительные эмоции.
   Перед вами фрагменты произведения, опубликованного в 1859 году, в
котором мы пропустили глагол и существительное. Восстановите любое из
пропущенных слов.

Ответ:
Угар.

Зачет:
Глагол "угорать" или "угореть" в любой форме.

Комментарий:
В этих отрывках из романа Ивана Гончарова "Обломов" говорится о
последствиях отравления угарным газом из-за чрезмерной экономии и
попытках сохранить тепло при топке печей в деревенских избах.
   Редакторская группа прощается с вами и желает всем командам больше
добра, трэша и угара.

Источник:
   1. И.А. Гончаров. Обломов.
http://az.lib.ru/g/goncharow_i_a/text_0020.shtml
   2. http://ru.wiktionary.org/wiki/угар
   3. https://lurkmore.co/Паук#.D0.A2.D0.B0.D0.BA_.D0.B3.D0.BE.D0.B2.D0.BE.D1.80.D0.B8.D0.BB_.D0.9F.D0.B0.D1.83.D0.BA

Автор:
Виктор Бойков (Раменское)


FreeBSD-CVSweb <freebsd-cvsweb@FreeBSD.org>